You are on page 1of 162

1/4/22, 11:53 AM PLATING EXAM ON BLOOD VESSELS: Attempt review

Dashboard My courses Patho 2A 1st Sem SY 2021 MD2-A,B,C,D,E,F,G,H,I,J PLATING EXAMS

PLATING EXAM ON BLOOD VESSELS

Started on Monday, 22 November 2021, 12:09 PM


State Finished
Completed on Monday, 22 November 2021, 12:28 PM
Time taken 19 mins 2 secs
Marks 15.00/20.00
Grade 75.00 out of 100.00

Question 1
Correct

Mark 1.00 out of 1.00

Composition of fibrous cap

a. cell debris
b. cholesterol crystals
c. smooth muscle cells 
d. calcium

The correct answer is: smooth muscle cells

Question 2
Correct

Mark 1.00 out of 1.00

A 35-year old male, heavy cigarette smoker presents with severe pain and Raynaud phenomenon on the left
extremity. These findings are compatible with

a. Goodpasture disease
b. Buerger disease 
c. Wegener granulomatosis

The correct answer is: Buerger disease

Question 3
Incorrect

Mark 0.00 out of 1.00

A 40-year old male with tertiary syphilis has a spherical outpouching involving a portion of the aorta. This finding is
compatible with

a. Pseudo-aneurysm
b. Fusiform aneurysm 
c. Saccular aneurysm
d. Dissection

The correct answer is: Saccular aneurysm

https://moodle.feu-nrmf.ph/mod/quiz/review.php?attempt=1300894&cmid=98714 1/6
1/4/22, 11:53 AM PLATING EXAM ON BLOOD VESSELS: Attempt review

Question 4

Correct

Mark 1.00 out of 1.00

A 30-year old male with redness and pain of the left leg was diagnosed with lymphangitis. Which is the most
probable cause of this condition?

a. malignant tumor
b. heavy cigarette smoking
c. Streptococcal infection 
d. Filariasis

The correct answer is: Streptococcal infection

Question 5
Correct

Mark 1.00 out of 1.00

A 20-year old male with segmental transmural necrotizing inflammation of renal artery was diagnosed with
Polyarteritis Nodosa. This condition is associated with the presence of

a. Antiendothelial cell antibodies


b. PR3-ANCAs
c. HBsAg-HBsAb complexes 
d. MPO-ANCAs

The correct answer is: HBsAg-HBsAb complexes

Question 6
Incorrect

Mark 0.00 out of 1.00

The following agents have vasoconstricting effects, EXCEPT

a. leukotrienes 
b. prostaglandins
c. thromboxane
d. catecholamines

The correct answer is: prostaglandins

Question 7
Correct

Mark 1.00 out of 1.00

A 60-year old female with hypertension has homogenous, eosinophilic thickening of the walls with luminal
narrowing of renal vessels. These findings are compatible with

a. Hyperplastic arteriolosclerosis
b. Hyaline arteriolosclerosis 
c. Fibromuscular intimal hyperplasia

The correct answer is: Hyaline arteriolosclerosis

https://moodle.feu-nrmf.ph/mod/quiz/review.php?attempt=1300894&cmid=98714 2/6
1/4/22, 11:53 AM PLATING EXAM ON BLOOD VESSELS: Attempt review

Question 8

Correct

Mark 1.00 out of 1.00

A 70-year old male with hypertension was diagnosed of having a blood-filled channel within the wall of the
ascending aorta due to the separation of the laminar planes of the media by blood. This finding is compatible with
what type of dissection?

a. DeBakey type III


b. DeBakey type II 
c. DeBakey type I
d. DeBakey type IV

The correct answer is: DeBakey type II

Question 9
Correct

Mark 1.00 out of 1.00

This is a principal point of physiologic resistance of blood flow

a. arteriole 
b. capillary
c. venule
d. aorta

The correct answer is: arteriole

Question 10
Incorrect

Mark 0.00 out of 1.00

Classic Kaposi sarcoma is highly associated with

a. Solid organ transplantation


b. Bartonella infection
c. Malignancy
d. HIV infection 

The correct answer is: Malignancy

Question 11
Incorrect

Mark 0.00 out of 1.00

A 4-year old girl presents with fever, edema of the hands and feet, erythema of the palms and soles,
desquamative rash and cervical lymph node enlargement. Treatment for this condition is

a. TNF-antagonist 
b. cyclophosphamide
c. antibiotics
d. intravenous immunoglobulin

The correct answer is: intravenous immunoglobulin

https://moodle.feu-nrmf.ph/mod/quiz/review.php?attempt=1300894&cmid=98714 3/6
1/4/22, 11:53 AM PLATING EXAM ON BLOOD VESSELS: Attempt review

Question 12

Correct

Mark 1.00 out of 1.00

An 80-year old female with congestive heart failure was diagnosed with DVT. Pain was elicited when her calf
muscle was squeezed. This is referred to

a. Moses sign
b. Homan sign 
c. Trousseau sign

The correct answer is: Homan sign

Question 13
Correct

Mark 1.00 out of 1.00

Congenital vascular anomaly with focal irregular thickening in medium and large muscular arteries and
described as STRING OF BEADS

a. Arteriovemous fistula
b. Anomalous coronary artery origin
c. Fibromuscluar dysplasia 
d. Berry aneurysm

The correct answer is: Fibromuscluar dysplasia

Question 14

Correct

Mark 1.00 out of 1.00

A 45-year old male experiences frequent fatigue, weight loss and fever. On PE, he has reduced blood pressure and
weak pulses in the carotids and upper extremities. These findings are compatible with

a. Goodpasture disease
b. Churg-Strauss syndrome
c. Takayasu arteritis 
d. Giant cell arteritis

The correct answer is: Takayasu arteritis

Question 15
Correct

Mark 1.00 out of 1.00

Aortic dissection is unusual in the presence of

a. substantial atherosclerosis 
b. severe hypertension
c. double-barelled aorta
d. Marfan syndrome

The correct answer is: substantial atherosclerosis

https://moodle.feu-nrmf.ph/mod/quiz/review.php?attempt=1300894&cmid=98714 4/6
1/4/22, 11:53 AM PLATING EXAM ON BLOOD VESSELS: Attempt review

Question 16

Correct

Mark 1.00 out of 1.00

An 18-year old female was diagnosed with Raynaud disease. This condition is often associated with

a. cold temperature 
b. tobacco smoking
c. viral infection
d. SLE

The correct answer is: cold temperature

Question 17
Correct

Mark 1.00 out of 1.00

Major cause of thoracic aortic aneurysm

a. Myocardial infarction
b. Marfan syndrome
c. Hypertension 
d. Atherosclerosis

The correct answer is: Hypertension

Question 18
Correct

Mark 1.00 out of 1.00

In the development of atheromatous plaques, the cornerstone of response-to-injury hypothesis is

a. endothelial cell injury 


b. hemodynamic disturbance
c. accumulation of lipoproteins
d. smooth muscle cell recruitment and proliferation

The correct answer is: endothelial cell injury

Question 19
Incorrect

Mark 0.00 out of 1.00

Renin is released in response to

a. high catecholamine level


b. low nitric oxide level 
c. low prostaglandin level
d. high aldosterone level

The correct answer is: high catecholamine level

https://moodle.feu-nrmf.ph/mod/quiz/review.php?attempt=1300894&cmid=98714 5/6
1/4/22, 11:53 AM PLATING EXAM ON BLOOD VESSELS: Attempt review

Question 20

Correct

Mark 1.00 out of 1.00

A 2-year old girl presents with a light pink flat lesion on the neck. A bigger lesion was present at birth and
significantly regressed over time. These findings are compatible with

a. Nevus flammeus 
b. Pyogenic granuloma
c. Spider telangiectasia
d. Port wine stain

The correct answer is: Nevus flammeus

Previous activity
◄ PLATING EXAM ON RBC AND BLEEDING DISORDERS

Jump to...

Next activity
LABORATORY REPORTING VISUAL MATERIAL (CYCLE 1) ►

Stay in touch

http://www.feu-nrmf.ph

+632 8983 8338 / +639176272623

info@feu-nrmf.ph

 Data retention summary

 Get the mobile app

https://moodle.feu-nrmf.ph/mod/quiz/review.php?attempt=1300894&cmid=98714 6/6
1/4/22, 11:53 AM PLATING EXAM ON RBC AND BLEEDING DISORDERS: Attempt review

Dashboard My courses Patho 2A 1st Sem SY 2021 MD2-A,B,C,D,E,F,G,H,I,J PLATING EXAMS

PLATING EXAM ON RBC AND BLEEDING DISORDERS

Started on Friday, 19 November 2021, 7:05 PM


State Finished
Completed on Friday, 19 November 2021, 7:29 PM
Time taken 24 mins 3 secs
Marks 15.00/20.00
Grade 37.50 out of 50.00 (75%)

Question 1
Correct

Mark 1.00 out of 1.00

True of Polycythemia vera EXCEPT:

a. WBC count is increased


b. Erythropeoitin level is low
c. Erythropoeitin level is high 
d. RBC count is elevated

The correct answer is: Erythropoeitin level is high

Question 2
Correct

Mark 1.00 out of 1.00

Hemoglobin electrophoresis results are as follows: Alpha=absent Beta=absent, Delta=absent , Gamma = present,
identify this hemoglobin ?

a. fetal hemoglobin
b. Gowers hemohlobin
c. adult hemoglobin
d. Barts hemoglobin 

The correct answer is: Barts hemoglobin

Question 3
Incorrect

Mark 0.00 out of 1.00

Patient presented with hemarthroses with no note of petechial hemorrhages. The lab exams revealed: Platelet
count is normal, Bleeding time and Pro-time are also normal but with prolonged Clotting time & PTT consistent
with:

a. DIC
b. Hemophilia A
c. Von Willebrand disease
d. Bernard-Soulier disease 

The correct answer is: Hemophilia A

https://moodle.feu-nrmf.ph/mod/quiz/review.php?attempt=1295489&cmid=98440 1/6
1/4/22, 11:53 AM PLATING EXAM ON RBC AND BLEEDING DISORDERS: Attempt review

Question 4

Correct

Mark 1.00 out of 1.00

Increased OFT in hereditary spherocytosis means that hemolysis is seen in this NaCl concentration

a. 0.1percent
b. greater than 0.5 percent 
c. less than 0.2 percent
d. less than 0.5 percent

The correct answer is: greater than 0.5 percent

Question 5
Correct

Mark 1.00 out of 1.00

The mechanism of anemia is increased red cell destruction EXCEPT:

a. G6PD deficiency
b. Iron deficiency anemia (IDA) 
c. Hereditary Spherocytosis
d. Sickle cell disease

The correct answer is: Iron deficiency anemia (IDA)

Question 6
Correct

Mark 1.00 out of 1.00

A purely vegetarian diet may lead to this type of anemia:

a. microcytic hypochromic
b. normocytic hypochromic
c. normocytic normochromic
d. macrocytic hyperchromic 

The correct answer is: macrocytic hyperchromic

Question 7
Correct

Mark 1.00 out of 1.00

The mechanism of anemia in patient with G6PD deficiency is:

a. chronic blood loss


b. decreased red cell production
c. increased red cell destruction 
d. acute blood loss

The correct answer is: increased red cell destruction

https://moodle.feu-nrmf.ph/mod/quiz/review.php?attempt=1295489&cmid=98440 2/6
1/4/22, 11:53 AM PLATING EXAM ON RBC AND BLEEDING DISORDERS: Attempt review

Question 8

Correct

Mark 1.00 out of 1.00

The PBS show small red cells with large central pallor and scattered elongated red cells:

a. Vit. B12 deficiency


b. IDA 
c. Pernicious anemia
d. Folic Acid deficiency

The correct answer is: IDA

Question 9
Incorrect

Mark 0.00 out of 1.00

A 20 year old female had mitral valve replacement secondary to rheumatic valvulitis. What are the laboratory
findings that would be seen in this condition?

a. numerous target cells on PBS 


b. erythrocytosis
c. basophilic stipplings
d. increased indirect bilirubin

The correct answer is: increased indirect bilirubin

Question 10
Incorrect

Mark 0.00 out of 1.00

The most important and most common cause of anemia of decreased erythropoiesis is:

a. Renal failure 
b. Chronic inflammation
c. Aplastic anemia
d. Nutritional deficiencies

The correct answer is: Nutritional deficiencies

Question 11

Correct

Mark 1.00 out of 1.00

A 20 year old male presenting with pallor , weakness and shortness of breath. RBC count = 3.5 x 10 12/L, MCV = 65 fl,
MCH = 20 pg, MCHC = 28. Based on the red cell indices, which of the following condition/s should NOT be included
among your differential diagnosis?

a. Pernicious anemia 
b. Iron deficiency anemia(IDA)
c. Thalassemia
d. Sideroblastic anemia

The correct answer is: Pernicious anemia

https://moodle.feu-nrmf.ph/mod/quiz/review.php?attempt=1295489&cmid=98440 3/6
1/4/22, 11:53 AM PLATING EXAM ON RBC AND BLEEDING DISORDERS: Attempt review

Question 12

Correct

Mark 1.00 out of 1.00

The following cells come from the same cell lineage, EXCEPT:

a. Granulocytes
b. Lymphocytes 
c. Megakaryocytes
d. Erythrocytes

The correct answer is: Lymphocytes

Question 13
Correct

Mark 1.00 out of 1.00

The only hemolytic anemia caused by an acquired genetic defect is:

a. Warm antibody Immune hemolytic anemia


b. Paroxysmal Cold Hemoglobinuria
c. Cold agglutinin Immune hemolytic anemia
d. Paroxysmal Nocturnal Hemoglobinuria 

The correct answer is: Paroxysmal Nocturnal Hemoglobinuria

Question 14
Correct

Mark 1.00 out of 1.00

A 25 year old male was diagnosed with AML . What is the best site for bone marrow aspiration?

a. humerus
b. tibia
c. pelvis 
d. femur

The correct answer is: pelvis

Question 15
Correct

Mark 1.00 out of 1.00

In a patient with bleeding disorder, the following must be elicited in the history:

a. history of eating disorders


b. drug history 
c. alcohol intake
d. smoking

The correct answer is: drug history

https://moodle.feu-nrmf.ph/mod/quiz/review.php?attempt=1295489&cmid=98440 4/6
1/4/22, 11:53 AM PLATING EXAM ON RBC AND BLEEDING DISORDERS: Attempt review

Question 16

Correct

Mark 1.00 out of 1.00

Patient suffered of sepsis and presented with hypotension and multiple petechial hemorrhages. Platelet count and
coagulation factors are expected to be:

a. high
b. low 
c. normal

The correct answer is: low

Question 17
Correct

Mark 1.00 out of 1.00

Patients suffering of hemolytic anemia due to extravascular hemolysis usually presents with the following EXCEPT:

a. splenomegaly
b. anemia
c. jaundice
d. hemoglobinemia 

The correct answer is: hemoglobinemia

Question 18
Correct

Mark 1.00 out of 1.00

A 7 year old male presents with easy bruising. Lab results showed normal platelet count, normal PT, aPTT. What is
the cause of the symptoms?

a. quantitative platelet abnormality


b. clotting factor deficiency
c. qualitative platelet abnormality
d. blood vessel abnormality 

The correct answer is: blood vessel abnormality

Question 19
Incorrect

Mark 0.00 out of 1.00

Elevation of this bleeding parameter favors a diagnosis of Chronic DIC:

a. PT
b. d-dimer
c. Thrombin time
d. aPTT 

The correct answer is: d-dimer

https://moodle.feu-nrmf.ph/mod/quiz/review.php?attempt=1295489&cmid=98440 5/6
1/4/22, 11:53 AM PLATING EXAM ON RBC AND BLEEDING DISORDERS: Attempt review

Dashboard My courses Patho 2A 1st Sem SY 2021 MD2-A,B,C,D,E,F,G,H,I,J PLATING EXAMS

PLATING EXAM ON RBC AND BLEEDING DISORDERS

Started on Friday, 19 November 2021, 7:05 PM


State Finished
Completed on Friday, 19 November 2021, 7:29 PM
Time taken 24 mins 3 secs
Marks 15.00/20.00
Grade 37.50 out of 50.00 (75%)

Question 1
Correct

Mark 1.00 out of 1.00

True of Polycythemia vera EXCEPT:

a. WBC count is increased


b. Erythropeoitin level is low
c. Erythropoeitin level is high 
d. RBC count is elevated

The correct answer is: Erythropoeitin level is high

Question 2
Correct

Mark 1.00 out of 1.00

Hemoglobin electrophoresis results are as follows: Alpha=absent Beta=absent, Delta=absent , Gamma = present,
identify this hemoglobin ?

a. fetal hemoglobin
b. Gowers hemohlobin
c. adult hemoglobin
d. Barts hemoglobin 

The correct answer is: Barts hemoglobin

Question 3
Incorrect

Mark 0.00 out of 1.00

Patient presented with hemarthroses with no note of petechial hemorrhages. The lab exams revealed: Platelet
count is normal, Bleeding time and Pro-time are also normal but with prolonged Clotting time & PTT consistent
with:

a. DIC
b. Hemophilia A
c. Von Willebrand disease
d. Bernard-Soulier disease 

The correct answer is: Hemophilia A

https://moodle.feu-nrmf.ph/mod/quiz/review.php?attempt=1295489&cmid=98440 1/6
1/4/22, 11:52 AM PLATING EXAM ON WBC: Attempt review

Dashboard My courses Patho 2A 1st Sem SY 2021 MD2-A,B,C,D,E,F,G,H,I,J PLATING EXAMS

PLATING EXAM ON WBC

Started on Monday, 15 November 2021, 12:04 PM


State Finished
Completed on Monday, 15 November 2021, 12:29 PM
Time taken 25 mins 56 secs
Marks 12.00/20.00
Grade 60.00 out of 100.00

Question 1
Incorrect

Mark 0.00 out of 1.00

Mandibular mass is observed in the following:

a. African Burkitt lymphoma 


b. Non-endemic lymhpoma
c. None of the other choices
d. Sporadic lymphoma

The correct answer is: None of the other choices

Question 2
Incorrect

Mark 0.00 out of 1.00

Lymphocyte rich Hodgkin diseases is made up of:

a. Malignant lymphocytes
b. None of the other choices
c. Reactive lymphocytes 
d. Mesothial

The correct answer is: None of the other choices

Question 3
Incorrect

Mark 0.00 out of 1.00

This provides the best assessment of the morphology of hematopoietic cells.

a. bone marrow biopsy 


b. bone marrow count
c. peripheral blood smear
d. bone marrow aspirate

The correct answer is: bone marrow aspirate

https://moodle.feu-nrmf.ph/mod/quiz/review.php?attempt=1275133&cmid=97683 1/6
1/4/22, 11:52 AM PLATING EXAM ON WBC: Attempt review

Question 4

Incorrect

Mark 0.00 out of 1.00

Spleen neoplasms are characterized by the following:

a. All of the other choice 


b. Rare
c. Fibroma is the most common type
d. Repeated trauma is a primary risk

The correct answer is: Rare

Question 5
Correct

Mark 1.00 out of 1.00

Which among the following is a peripheral B cell neoplasm?

a. All of the other choices


b. extranodal NK lymphoma
c. Acute lymphoblastic lymphoma
d. Follicular lymphoma 

The correct answer is: Follicular lymphoma

Question 6
Correct

Mark 1.00 out of 1.00

Accelerated destruction of neutrophils occurs with

a. viral infections
b. splenomegaly 
c. hepatomegaly
d. decreased peripheral utilization

The correct answer is: splenomegaly

Question 7
Correct

Mark 1.00 out of 1.00

Blood cell progenitors first appear during the ____ week of embryonic development in the yolk sac.

a. 3rd 
b. 4th
c. 1st
d. 2nd

The correct answer is: 3rd

https://moodle.feu-nrmf.ph/mod/quiz/review.php?attempt=1275133&cmid=97683 2/6
1/4/22, 11:52 AM PLATING EXAM ON WBC: Attempt review

Question 8

Correct

Mark 1.00 out of 1.00

The clinicopathologic diagnosis of multiple myeloma relies on identification of clonal plasma cells in the marrow
and the presence of the following EXCEPT

a. bone lesions
b. renal dysfunction
c. anemia
d. hypocalcemia 

The correct answer is: hypocalcemia

Question 9
Incorrect

Mark 0.00 out of 1.00

The following laboratory examinations finding/s is/are found in polycythemia vera:

a. Decreased white cell count


b. Morphologic abnormalities of platelets
c. Platelet count of 350,000/mm3 
d. All of the other choices

The correct answer is: Morphologic abnormalities of platelets

Question 10
Incorrect

Mark 0.00 out of 1.00

Adult reference range for white cells

a. 1.4 to 6.5 
b. 150 to 140
c. 40 to 70
d. 4.8 to 10.8

The correct answer is: 4.8 to 10.8

Question 11

Correct

Mark 1.00 out of 1.00

True of Multiple myeloma

a. plasma cell neoplasm 


b. lymphoid neoplasm
c. myeloid neoplasm
d. peripheral T- and NK-cell neoplasm

The correct answer is: plasma cell neoplasm

https://moodle.feu-nrmf.ph/mod/quiz/review.php?attempt=1275133&cmid=97683 3/6
1/4/22, 11:52 AM PLATING EXAM ON WBC: Attempt review

Question 12

Incorrect

Mark 0.00 out of 1.00

In normal adults, only about ______% of the marrow space is hematopoietically active,

a. 60%
b. 50%
c. 70% 
d. 80%

The correct answer is: 50%

Question 13
Correct

Mark 1.00 out of 1.00

In multiple myeloma, high serum levels of _____are seen in patients with active disease.

a. IL-8
b. IL-4
c. IL-6 
d. IL-2

The correct answer is: IL-6

Question 14
Correct

Mark 1.00 out of 1.00

Punched out defect is a typical finding in:

a. A neoplasm with CD138 (+) tumor cells


b. All of the other choices 
c. A patient with a condition that is a associated with renal dysfunction
d. Plasma cell neoplasm

The correct answer is: All of the other choices

Question 15
Correct

Mark 1.00 out of 1.00

In primary myelofibrosis, the following is/are observed:

a. All of the other choices 


b. Cytopenias
c. Extensive collagen deposition
d. Extramedullary hematopoiesis is observed

The correct answer is: All of the other choices

https://moodle.feu-nrmf.ph/mod/quiz/review.php?attempt=1275133&cmid=97683 4/6
1/4/22, 11:52 AM PLATING EXAM ON WBC: Attempt review

Question 16

Correct

Mark 1.00 out of 1.00

TRUE of Reed Sternberg cell:

a. Metastatic cells to the lymph nodes


b. Seen in Hodgkin disease 
c. None of the other choices
d. Benign lymphoid cells

The correct answer is: Seen in Hodgkin disease

Question 17
Incorrect

Mark 0.00 out of 1.00

The peripheral blood smear of a 27 year old male shows maturation stages of the granulocytic series with around
5% blasts. Which of the following is associated?

a. BCR-ABL fusion gene


b. extramedullary hematopoiesis is prominent
c. Prominent fibrosis is found in the bone marrow
d. all of the other choices 

The correct answer is: BCR-ABL fusion gene

Question 18
Correct

Mark 1.00 out of 1.00

Leukocytosis due to exercise is secondary to

a. decreased extravasation into tissues


b. increased release from marrow stores
c. decreased margination 
d. increased marrow production

The correct answer is: decreased margination

Question 19

Correct

Mark 1.00 out of 1.00

The bone most commonly affected in multiple myeloma is the

a. clavicle
b. scapula
c. femur
d. vertebral column 

The correct answer is: vertebral column

https://moodle.feu-nrmf.ph/mod/quiz/review.php?attempt=1275133&cmid=97683 5/6
1/4/22, 11:52 AM PLATING EXAM ON WBC: Attempt review

Question 20

Correct

Mark 1.00 out of 1.00

Which among the ff. is the most common form of Hodgkin lyphoma:

a. Mixed cellularity
b. Nodular sclerosis 
c. Lymphocyte depletion
d. Lymphocyte depletion

The correct answer is: Nodular sclerosis

Previous activity
◄ PLATING EXAM ON DISEASES OF INFANCY AND CHILDHOOD

Jump to...

Next activity

PLATING EXAM ON RBC AND BLEEDING DISORDERS ►

Stay in touch

http://www.feu-nrmf.ph

+632 8983 8338 / +639176272623

info@feu-nrmf.ph

 Data retention summary

 Get the mobile app

https://moodle.feu-nrmf.ph/mod/quiz/review.php?attempt=1275133&cmid=97683 6/6
1/4/22, 11:52 AM PLATING EXAM ON DISEASES OF INFANCY AND CHILDHOOD: Attempt review

Dashboard My courses Patho 2A 1st Sem SY 2021 MD2-A,B,C,D,E,F,G,H,I,J PLATING EXAMS

PLATING EXAM ON DISEASES OF INFANCY AND CHILDHOOD

Started on Monday, 8 November 2021, 12:05 PM


State Finished
Completed on Monday, 8 November 2021, 12:30 PM
Time taken 24 mins 57 secs
Marks 15.00/20.00
Grade 75.00 out of 100.00

Question 1
Incorrect

Mark 0.00 out of 1.00

PLATING ON DISEASES OF INFANCY AND CHILDHOOD

Assess whether the statement is True or False: Most of the tumors in infants and children are benign rather than
cancer.

a. False 
b. True

The correct answer is: True

Question 2
Correct

Mark 1.00 out of 1.00

PLATING ON DISEASES OF INFANCY AND CHILDHOOD

What is the reason for the peak of sensitivity of the embryo to teratogenic agents between fourth and fifth week of
life?

a. Organs are being crafted from germ cell layers during the period 
b. The feto-maternal circulation is very poor during the period hence poor nutrition
c. The fertilized ovum is being implanted to the endometrium during the period
d. The amniotic fluid is expected to be very low in volume during the period

The correct answer is: Organs are being crafted from germ cell layers during the period

https://moodle.feu-nrmf.ph/mod/quiz/review.php?attempt=1255201&cmid=96660 1/8
1/4/22, 11:52 AM PLATING EXAM ON DISEASES OF INFANCY AND CHILDHOOD: Attempt review

Question 3

Correct

Mark 1.00 out of 1.00

PLATING ON DISEASES OF INFANCY AND CHILDHOOD

A premature newborn had blood stools and abdominal distention. Abdominal radiograph showed gas within
intestinal walls. Surgical resection of the involved bowel was done. What is the expected appearance of the
involved bowel?

a. Normal looking
b. Distended, congested and friable 
c. Fibrotic with stricture
d. None of the given options

The correct answer is: Distended, congested and friable

Question 4
Correct

Mark 1.00 out of 1.00

PLATING ON DISEASES OF INFANCY AND CHILDHOOD

Which of the following inborn errors of metabolism or genetic disorders is inherited through autosomal recessive
fashion?

a. Mucoviscidosis
b. Galactosemia
c. PKU
d. All of the given options 

The correct answer is: All of the given options

Question 5
Incorrect

Mark 0.00 out of 1.00

PLATING ON DISEASES OF INFANCY AND CHILDHOOD

The following conditions are compatible with sudden infant death syndrome, except:

a. Unremarkable death scene


b. Lungs float in water during autopsy exam 
c. Heart defect in autopsy
d. Unremarkable medical history

The correct answer is: Heart defect in autopsy

https://moodle.feu-nrmf.ph/mod/quiz/review.php?attempt=1255201&cmid=96660 2/8
1/4/22, 11:52 AM PLATING EXAM ON DISEASES OF INFANCY AND CHILDHOOD: Attempt review

Question 6

Correct

Mark 1.00 out of 1.00

PLATING ON DISEASES OF INFANCY AND CHILDHOOD

Assess whether the statement is True or False: Newborns with fetal growth restriction are born at term but are
small for gestational age.

a. False
b. True 

The correct answer is: True

Question 7
Correct

Mark 1.00 out of 1.00

PLATING ON DISEASES OF INFANCY AND CHILDHOOD

The following are reasons why there are less cases of hemolytic disease in ABO incompatibility compared by Rh
incompatibility, except:

a. A and B antigens are also found on cells other than red cells
b. Rh incompatibility is protective against ABO hemolytic disease 
c. Neonatal red cells express A and B antigens poorly
d. Most anti A and anti B antibodies are of IgM isotype

The correct answer is: Rh incompatibility is protective against ABO hemolytic disease

Question 8

Correct

Mark 1.00 out of 1.00

PLATING ON DISEASES OF INFANCY AND CHILDHOOD

The following statements are true about prematurity, except:

a. There is increased risk of prematurity if the pregnant woman hash cervical incompetence
b. This can be caused by rupture of fetal membrane before the 37th week of gestation
c. It is the most common cause of neonatal mortality 
d. Prematurity can cause hyaline membrane disease

The correct answer is: It is the most common cause of neonatal mortality

https://moodle.feu-nrmf.ph/mod/quiz/review.php?attempt=1255201&cmid=96660 3/8
1/4/22, 11:52 AM PLATING EXAM ON DISEASES OF INFANCY AND CHILDHOOD: Attempt review

Question 9

Incorrect

Mark 0.00 out of 1.00

PLATING ON DISEASES OF INFANCY AND CHILDHOOD

Which of the following organisms reached the fetus through ascending route?

a. Plasmodium spp. 
b. Herpes simplex virus II
c. Listeria spp.
d. Treponema spp.

The correct answer is: Herpes simplex virus II

Question 10
Correct

Mark 1.00 out of 1.00

PLATING ON DISEASES OF INFANCY AND CHILDHOOD

The left kidney of a 4 year old male was removed due to a mass. It was examined by a pathologist who saw that
the mass has blastemal, epithelial and stromal component. What is the most common clinical presentation of this
tumor?

a. Hematuria
b. Hypertension
c. Abdominal mass 
d. Intestinal obstruction

The correct answer is: Abdominal mass

Question 11
Incorrect

Mark 0.00 out of 1.00

PLATING ON DISEASES OF INFANCY AND CHILDHOOD

A newborn is cyanotic with respiratory distress at birth. The condition improved after resuscitation but after 30
minutes the patient was noted to be cyanotic and with difficulty breathing. Chest x-ray showed ground glass
picture. If the patient will die and an autopsy will be done, the lungs are expected to have the following
characteristics, except:

a. The lungs are normal in size and shape 


b. The lungs float in water due to the presence of air within alveolar spaces
c. The lungs look like the liver with reddish purple external surface
d. The lungs are solid and airless

The correct answer is: The lungs float in water due to the presence of air within alveolar spaces

https://moodle.feu-nrmf.ph/mod/quiz/review.php?attempt=1255201&cmid=96660 4/8
1/4/22, 11:52 AM PLATING EXAM ON DISEASES OF INFANCY AND CHILDHOOD: Attempt review

Question 12

Correct

Mark 1.00 out of 1.00

PLATING ON DISEASES OF INFANCY AND CHILDHOOD

Which organ is involved in the most serious complications of having mucoviscidosis?

a. Lung 
b. Testis
c. Liver
d. Pancreas

The correct answer is: Lung

Question 13
Correct

Mark 1.00 out of 1.00

PLATING ON DISEASES OF INFANCY AND CHILDHOOD

The accumulation of fluid among patients with immune hydrops is due to:

a. Fibrosis in lymphatic channels


b. Decreased hydrostatic pressure and hyponatremia
c. Cardiac decompensation and liver damage 
d. Accumulation of free hemoglobin in the blood due to hemolysis

The correct answer is: Cardiac decompensation and liver damage

Question 14
Correct

Mark 1.00 out of 1.00

PLATING ON DISEASES OF INFANCY AND CHILDHOOD

A newborn has perinatal parvovirus B 19 infection, which can be expected to be pale?

a. Fetus
b. Both the fetus and placenta 
c. None will be pale
d. Placenta

The correct answer is: Both the fetus and placenta

https://moodle.feu-nrmf.ph/mod/quiz/review.php?attempt=1255201&cmid=96660 5/8
1/4/22, 11:52 AM PLATING EXAM ON DISEASES OF INFANCY AND CHILDHOOD: Attempt review

Question 15

Correct

Mark 1.00 out of 1.00

PLATING ON DISEASES OF INFANCY AND CHILDHOOD

Which of the following can cause non-immune hydrops?

a. All of the given options 


b. Chromosomal anomalies
c. Cardiovascular defects
d. Parvovirus B infection

The correct answer is: All of the given options

Question 16
Correct

Mark 1.00 out of 1.00

PLATING ON DISEASES OF INFANCY AND CHILDHOOD

Which of the following congenital anomalies exhibit an intrinsically abnormal developmental process?

a. Anencephaly as a malformation 
b. Deformed left shoulder associated with large uterine leiomyomas
c. Potter sequence due to rupture of fetal membrane
d. Absence of lower leg associated with amniotic band

The correct answer is: Anencephaly as a malformation

Question 17
Correct

Mark 1.00 out of 1.00

PLATING ON DISEASES OF INFANCY AND CHILDHOOD

In which of the following age groups is accident not the leading cause of death?

a. 1 to 4 years of age
b. 5 to 14 years of age
c. Infancy 
d. None of the given choices

The correct answer is: Infancy

Question 18
Correct

Mark 1.00 out of 1.00

PLATING ON DISEASES OF INFANCY AND CHILDHOOD

Assess whether the statement is True or False: Most cases of early onset neonatal sepsis are acquired
immediately after birth upon exposure to external environment.

a. True
b. False 

The correct answer is: False

https://moodle.feu-nrmf.ph/mod/quiz/review.php?attempt=1255201&cmid=96660 6/8
1/4/22, 11:52 AM PLATING EXAM ON DISEASES OF INFANCY AND CHILDHOOD: Attempt review

Question 19

Incorrect

Mark 0.00 out of 1.00

PLATING ON DISEASES OF INFANCY AND CHILDHOOD

In patients with phenylalanine hydroxylase deficiency, there will be decreased:

a. NADH
b. Tyrosine
c. DHPR 
d. Phenylalanine

The correct answer is: Tyrosine

Question 20
Correct

Mark 1.00 out of 1.00

PLATING ON DISEASES OF INFANCY AND CHILDHOOD

Which congenital anomaly cannot be explained on the basis of a single localized initiating defect?

a. Deformation
b. Sequence
c. Malformation
d. Malformation syndrome 

The correct answer is: Malformation syndrome

https://moodle.feu-nrmf.ph/mod/quiz/review.php?attempt=1255201&cmid=96660 7/8
1/4/22, 11:52 AM PLATING EXAM ON DISEASES OF INFANCY AND CHILDHOOD: Attempt review

Previous activity
◄ PLATING EXAM ON ENVIRONMENTAL AND NUTRITIONAL DISEASES

Jump to...

Next activity

PLATING EXAM ON WBC ►

Stay in touch

http://www.feu-nrmf.ph

+632 8983 8338 / +639176272623

info@feu-nrmf.ph

 Data retention summary

 Get the mobile app

https://moodle.feu-nrmf.ph/mod/quiz/review.php?attempt=1255201&cmid=96660 8/8
1/4/22, 11:51 AM PLATING EXAM ON ENVIRONMENTAL AND NUTRITIONAL DISEASES: Attempt review

Dashboard My courses Patho 2A 1st Sem SY 2021 MD2-A,B,C,D,E,F,G,H,I,J PLATING EXAMS

PLATING EXAM ON ENVIRONMENTAL AND NUTRITIONAL DISEASES

Started on Monday, 25 October 2021, 12:00 PM


State Finished
Completed on Monday, 25 October 2021, 12:27 PM
Time taken 26 mins 14 secs
Marks 11.00/20.00
Grade 55.00 out of 100.00

Question 1
Correct

Mark 1.00 out of 1.00

PLATING ON ENVIRONMENTAL AND NUTRITIONAL DISEASES

Nicotine in cigarette smoke is most likely to result in ________________.

a. mucosal irritation
b. depression 
c. definitely cancer development
d. toxicity to the cilia

The correct answer is: depression

Question 2
Incorrect

Mark 0.00 out of 1.00

PLATING ON ENVIRONMENTAL AND NUTRITIONAL DISEASES

Burn injury reaching the muscles underneath the subcutaneous tissue.

a. partial thickness burn


b. fourth degree burn
c. third degree burn 

The correct answer is: fourth degree burn

https://moodle.feu-nrmf.ph/mod/quiz/review.php?attempt=1236543&cmid=94660 1/8
1/4/22, 11:52 AM PLATING EXAM ON WBC: Attempt review

Question 16

Correct

Mark 1.00 out of 1.00

TRUE of Reed Sternberg cell:

a. Metastatic cells to the lymph nodes


b. Seen in Hodgkin disease 
c. None of the other choices
d. Benign lymphoid cells

The correct answer is: Seen in Hodgkin disease

Question 17
Incorrect

Mark 0.00 out of 1.00

The peripheral blood smear of a 27 year old male shows maturation stages of the granulocytic series with around
5% blasts. Which of the following is associated?

a. BCR-ABL fusion gene


b. extramedullary hematopoiesis is prominent
c. Prominent fibrosis is found in the bone marrow
d. all of the other choices 

The correct answer is: BCR-ABL fusion gene

Question 18
Correct

Mark 1.00 out of 1.00

Leukocytosis due to exercise is secondary to

a. decreased extravasation into tissues


b. increased release from marrow stores
c. decreased margination 
d. increased marrow production

The correct answer is: decreased margination

Question 19

Correct

Mark 1.00 out of 1.00

The bone most commonly affected in multiple myeloma is the

a. clavicle
b. scapula
c. femur
d. vertebral column 

The correct answer is: vertebral column

https://moodle.feu-nrmf.ph/mod/quiz/review.php?attempt=1275133&cmid=97683 5/6
1/4/22, 11:52 AM PLATING EXAM ON WBC: Attempt review

Question 20

Correct

Mark 1.00 out of 1.00

Which among the ff. is the most common form of Hodgkin lyphoma:

a. Mixed cellularity
b. Nodular sclerosis 
c. Lymphocyte depletion
d. Lymphocyte depletion

The correct answer is: Nodular sclerosis

Previous activity
◄ PLATING EXAM ON DISEASES OF INFANCY AND CHILDHOOD

Jump to...

Next activity

PLATING EXAM ON RBC AND BLEEDING DISORDERS ►

Stay in touch

http://www.feu-nrmf.ph

+632 8983 8338 / +639176272623

info@feu-nrmf.ph

 Data retention summary

 Get the mobile app

https://moodle.feu-nrmf.ph/mod/quiz/review.php?attempt=1275133&cmid=97683 6/6
1/4/22, 11:51 AM PLATING EXAM ON ENVIRONMENTAL AND NUTRITIONAL DISEASES: Attempt review

Question 9

Incorrect

Mark 0.00 out of 1.00

PLATING ON ENVIRONMENTAL AND NUTRITIONAL DISEASES

Which of the following conditions is worsened by heat waves?

a. waterborne infections
b. cardiovascular diseases
c. foodborne diseases 
d. malnutrition

The correct answer is: cardiovascular diseases

Question 10
Correct

Mark 1.00 out of 1.00

PLATING ON ENVIRONMENTAL AND NUTRITIONAL DISEASES

A patient who apparently attempted suicide had cherry-red color of the skin including the mucous membranes.
What is the likely diagnosis?

a. Nitro dioxide toxicity


b. Radon poisoning
c. Chronic carbon monoxide poisoning
d. Acute carbon monoxide poisoning 

The correct answer is: Acute carbon monoxide poisoning

Question 11
Correct

Mark 1.00 out of 1.00

PLATING ON ENVIRONMENTAL AND NUTRITIONAL DISEASES

A patient with liver angiosarcoma is most likely exposed to which occupational hazards?

a. Vinyl chloride 
b. Beryllium
c. Benzene
d. Naphthylamines

The correct answer is: Vinyl chloride

https://moodle.feu-nrmf.ph/mod/quiz/review.php?attempt=1236543&cmid=94660 4/8
1/4/22, 11:51 AM PLATING EXAM ON ENVIRONMENTAL AND NUTRITIONAL DISEASES: Attempt review

Question 12

Incorrect

Mark 0.00 out of 1.00

PLATING ON ENVIRONMENTAL AND NUTRITIONAL DISEASES

Mercury from contaminated fish.

a. methyl mercury
b. mercuric chloride
c. metallic mercury
d. inorganic mercury 

The correct answer is: methyl mercury

Question 13
Incorrect

Mark 0.00 out of 1.00

PLATING ON ENVIRONMENTAL AND NUTRITIONAL DISEASES

Most markedly noticed in Marasmus.

a. hypoalbuminemia 
b. flaky paint skin
c. peripheral edema
d. atrophy of muscles

The correct answer is: atrophy of muscles

Question 14
Correct

Mark 1.00 out of 1.00

PLATING ON ENVIRONMENTAL AND NUTRITIONAL DISEASES

Fat-burning molecule.

a. Amylin
b. Leptin
c. Adiponectin 
d. Ghrelin

The correct answer is: Adiponectin

https://moodle.feu-nrmf.ph/mod/quiz/review.php?attempt=1236543&cmid=94660 5/8
1/4/22, 11:51 AM PLATING EXAM ON ENVIRONMENTAL AND NUTRITIONAL DISEASES: Attempt review

Question 15

Incorrect

Mark 0.00 out of 1.00

PLATING ON ENVIRONMENTAL AND NUTRITIONAL DISEASES

Causes specific mutation in codon 249 of TP53 gene.

a. Nitrosamines 
b. Sodium nitrite
c. Gherlin
d. Aflatoxin

The correct answer is: Aflatoxin

Question 16
Incorrect

Mark 0.00 out of 1.00

PLATING ON ENVIRONMENTAL AND NUTRITIONAL DISEASES

Tachycardia, hypertension, and peripheral vasoconstriction.

a. Phencyclidine
b. Cocaine
c. Opiates
d. Ecstasy 

The correct answer is: Cocaine

Question 17
Incorrect

Mark 0.00 out of 1.00

PLATING ON ENVIRONMENTAL AND NUTRITIONAL DISEASES

Core body temperature is 41 degrees centigrade and there was absence of sweat. What is this condition?

a. Heat exhaustion
b. Adverse reaction to anesthetic 
c. Heat cramps
d. Heat stroke

The correct answer is: Heat stroke

https://moodle.feu-nrmf.ph/mod/quiz/review.php?attempt=1236543&cmid=94660 6/8
1/4/22, 11:51 AM PLATING EXAM ON ENVIRONMENTAL AND NUTRITIONAL DISEASES: Attempt review

Question 18

Correct

Mark 1.00 out of 1.00

PLATING ON ENVIRONMENTAL AND NUTRITIONAL DISEASES

Good OZONE.

a. formed by interaction of UV radiation and oxygen 


b. ground-level ozone
c. formed by nitrogen oxides and volatile organic compounds in the presence of sunlight
d. released by industrial emissions

The correct answer is: formed by interaction of UV radiation and oxygen

Question 19
Correct

Mark 1.00 out of 1.00

PLATING ON ENVIRONMENTAL AND NUTRITIONAL DISEASES

What is the likely diagnosis in a patient with dementia, dermatitis and diarrhea?

a. Folate deficiency
b. Niacin deficiency 
c. Vitamin A deficiency
d. Scurvy

The correct answer is: Niacin deficiency

Question 20
Incorrect

Mark 0.00 out of 1.00

PLATING ON ENVIRONMENTAL AND NUTRITIONAL DISEASES

Far more widespread hazard and has claimed many more lives.

a. Marijuana
b. Alcohol
c. Smoking 
d. Methamphetamines

The correct answer is: Alcohol

https://moodle.feu-nrmf.ph/mod/quiz/review.php?attempt=1236543&cmid=94660 7/8
1/4/22, 11:51 AM PLATING EXAM ON ENVIRONMENTAL AND NUTRITIONAL DISEASES: Attempt review

Previous activity
◄ PLATING EXAM ON NEOPLASIA 2

Jump to...

Next activity

PLATING EXAM ON DISEASES OF INFANCY AND CHILDHOOD ►

Stay in touch

http://www.feu-nrmf.ph

+632 8983 8338 / +639176272623

info@feu-nrmf.ph

 Data retention summary

 Get the mobile app

https://moodle.feu-nrmf.ph/mod/quiz/review.php?attempt=1236543&cmid=94660 8/8
1/4/22, 11:51 AM PLATING EXAM ON NEOPLASIA 2: Attempt review

Dashboard My courses Patho 2A 1st Sem SY 2021 MD2-A,B,C,D,E,F,G,H,I,J PLATING EXAMS

PLATING EXAM ON NEOPLASIA 2

Started on Monday, 11 October 2021, 12:01 PM


State Finished
Completed on Monday, 11 October 2021, 12:29 PM
Time taken 28 mins 59 secs
Marks 11.00/20.00
Grade 55.00 out of 100.00

Question 1
Incorrect

Mark 0.00 out of 1.00

PLATING ON NEOPLASIA 2

The following statements are true about the new blood vessels in a malignant tumor?

a. Upregulated expression of p53 molecules promote angiogenesis


b. The new blood vessels are normal in structure as well as in function 
c. Decreased thrombospondin-1 will allow expression of molecule that will stimulate migration and
proliferation of endothelial cells
d. The new tumor vasculature is not efficient in delivering oxygen as well as nutrients to malignant cells

The correct answer is: Decreased thrombospondin-1 will allow expression of molecule that will stimulate migration
and proliferation of endothelial cells

Question 2
Incorrect

Mark 0.00 out of 1.00

PLATING ON NEOPLASIA 2

Xeroderma pigmentosa develops due to what type of genetic aberration in the nucleotide excision repair system?

a. Gain of function mutation


b. Methylation 
c. Histone modification
d. Loss of function mutation

The correct answer is: Loss of function mutation

https://moodle.feu-nrmf.ph/mod/quiz/review.php?attempt=1175056&cmid=92821 1/9
1/4/22, 11:51 AM PLATING EXAM ON NEOPLASIA 2: Attempt review

Question 3

Incorrect

Mark 0.00 out of 1.00

PLATING ON NEOPLASIA 2

A 45 year old female has chronic myeloid leukemia and has been on treatment using BCR-ABL inhibitor. Despite
the presence of other genetic aberration, the patient is improving with the treatment targeting only the said
chromosomal translocation. This phenomenon is known as:

a. Philadelphia chromosome 
b. Warburg effect
c. Oncogenic stress
d. Oncogene addiction

The correct answer is: Oncogene addiction

Question 4
Correct

Mark 1.00 out of 1.00

PLATING ON NEOPLASIA 2

How does p53 prevent carcinogenesis?

a. By promoting permanent cell cycle arrest


b. By promoting mechanisms that evade senescence
c. By promoting apoptosis 
d. By mechanisms that inhibit catabolic metabolism in cells

The correct answers are: By promoting apoptosis, By promoting permanent cell cycle arrest

Question 5
Incorrect

Mark 0.00 out of 1.00

PLATING ON NEOPLASIA 2

A 40 year old female has a non-healing wound on her left breast. An excision biopsy was performed by her
surgeon and during the microscopic examination by the pathologist, cancer cells are present surrounded by
intense inflammatory reaction. How did the cancer cells survive and proliferate despite the inflammatory response
in the tissue?

a. All provided choices are correct 


b. Leukocytes release molecules that can promote release of growth factors from ECM
c. Inflammatory cells release VEGF which are directly mitogenic for cancer cells
d. Proteases released by leukocytes can promote cancer cell division and proliferation

The correct answer is: Leukocytes release molecules that can promote release of growth factors from ECM

https://moodle.feu-nrmf.ph/mod/quiz/review.php?attempt=1175056&cmid=92821 2/9
1/4/22, 11:51 AM PLATING EXAM ON NEOPLASIA 2: Attempt review

Question 6

Incorrect

Mark 0.00 out of 1.00

PLATING ON NEOPLASIA 2

A 4 year old female has tumor in the right eye which was histopathologically diagnosed as retinoblastoma.
Involving the governor of proliferation, which of the following is/are the possible mechanisms that lead to the
formation of the tumor?

a. Gain of function mutation in both alleles of the RB gene


b. All provided choices are correct 
c. Loss of function mutation in p16/INK4a gene
d. Loss of function mutation in genes encoding for kinases that phosphorylate RB

The correct answer is: Loss of function mutation in p16/INK4a gene

Question 7
Correct

Mark 1.00 out of 1.00

PLATING ON NEOPLASIA 2

Why is there a need for cellular metabolism to shift to Warburg metabolism?

a. Due to decreased supply of oxygen which is the final electron acceptor in oxidative phosphorylation
b. Because of the inefficient supply of nutrients in solid tumors
c. Warburg metabolism will provide more ATPs needed by cancer cells
d. Aerobic glycolysis will provide the needed metabolic intermediates for malignant cell components 

The correct answer is: Aerobic glycolysis will provide the needed metabolic intermediates for malignant cell
components

Question 8
Correct

Mark 1.00 out of 1.00

PLATING ON NEOPLASIA 2

A 35 year old female was diagnosed to have diffuse type of gastric adenocarcinoma based on the finding of the
pathologist that neoplastic cells are seen singly and loose clusters that are invading the stroma. Which of the
following must have been downregulated or poorly expressed which facilitated the first step in the invasion of the
tumor cells in this case?

a. E-cadherin 
b. MMPs
c. Cathepsin D
d. Fibronectin

The correct answer is: E-cadherin

https://moodle.feu-nrmf.ph/mod/quiz/review.php?attempt=1175056&cmid=92821 3/9
1/4/22, 11:51 AM PLATING EXAM ON NEOPLASIA 2: Attempt review

Question 9

Correct

Mark 1.00 out of 1.00

PLATING ON NEOPLASIA 2

A 60 year old female was diagnosed to have breast cancer 5 years ago. The tumor has recurred with metastatic
deposits to the liver, lung and a vertebral bone. Which of the following conditions is/are expected following
secretion of PTHRP by the tumor cells in the bone?

a. All provided choices are correct 


b. There will be upregulated expression of RANKL in osteoblast
c. Hypercalcemia following degradation of the bone matrix
d. There will be release of growth factors like IGF that has pro-growth effect on cancer cells

The correct answer is: All provided choices are correct

Question 10
Correct

Mark 1.00 out of 1.00

PLATING ON NEOPLASIA 2

The following molecules are tumor suppressors, except:

a. RB
b. JAK2 
c. P53
d. APC

The correct answer is: JAK2

Question 11
Correct

Mark 1.00 out of 1.00

PLATING ON NEOPLASIA 2

The following are roles of MHC class I molecule in anti-tumor response of the immune system, except:

a. to display processed endogenous tumor neoantigens to CTLs


b. the display the processed exogenous antigen through the use of proteasome by dendritic cells to CTLs 
c. in presentation of antigen by tumor cells to NK cells as a first line of defense
d. their downregulation in tumor cells allow killing of such cells by NK cells

The correct answer is: the display the processed exogenous antigen through the use of proteasome by dendritic
cells to CTLs

https://moodle.feu-nrmf.ph/mod/quiz/review.php?attempt=1175056&cmid=92821 4/9
1/4/22, 11:51 AM PLATING EXAM ON NEOPLASIA 2: Attempt review

Question 12

Incorrect

Mark 0.00 out of 1.00

PLATING ON NEOPLASIA 2

RB exerts its anti-proliferative effects when the molecule is:

a. Methylated
b. Bound to E2F 
c. Glycosylated
d. Hypophosphorylated

The correct answer is: Hypophosphorylated

Question 13
Correct

Mark 1.00 out of 1.00

PLATING ON NEOPLASIA 2

Which of the following can serve as tumor antigen?

a. Any of the given choices 


b. Overexpressed normal protein
c. Mutated self protein
d. Virus proteins produced following infection

The correct answer is: Any of the given choices

Question 14
Correct

Mark 1.00 out of 1.00

PLATING ON NEOPLASIA 2

Which of the following processes will allow survival of cancer cells?

a. Binding of APAF-1 to cytochrome c to activate caspase 9


b. BAX and BAK activation acting on mitochondrial membrane
c. Upregulated expression of BCL2 
d. Upregulated expression of p53

The correct answer is: Upregulated expression of BCL2

https://moodle.feu-nrmf.ph/mod/quiz/review.php?attempt=1175056&cmid=92821 5/9
1/4/22, 11:51 AM PLATING EXAM ON NEOPLASIA 2: Attempt review

Question 15

Correct

Mark 1.00 out of 1.00

PLATING ON NEOPLASIA 2

Which of the following will not support the growth-promoting metabolism in cancer cells?

a. Loss-of-function mutation of p53 gene


b. RAS gene gain-of-function mutation
c. Loss-of-function mutation of MYC gene 
d. Constitutively active PI3K/AKT signaling pathway

The correct answer is: Loss-of-function mutation of MYC gene

Question 16
Correct

Mark 1.00 out of 1.00

PLATING ON NEOPLASIA 2

These conditions promote cancer cell immortality, except:

a. increased replicative capacity


b. cell cycle arrest to avoid shortening of the telomeric ends of chromatin 
c. increased expression of telomerase
d. alternative lengthening of chromosomal telomeric ends

The correct answer is: cell cycle arrest to avoid shortening of the telomeric ends of chromatin

Question 17
Incorrect

Mark 0.00 out of 1.00

PLATING ON NEOPLASIA 2

A 65 year old female non-smoker was diagnosed to have lung adenocarcinoma. Molecular test to detect for
mutation in the gene encoding for EGFR was requested by her oncologist. The following statements are true about
the analyte in the test, except:

a. It is a receptor with intrinsic tyrosine kinase activity


b. it is the initiating mutation if it is the first mutation
c. ERBB1 point mutation can produce constitutively active EGFR 
d. The mutation is a passenger mutation but is of prognostic significance in the case

The correct answer is: The mutation is a passenger mutation but is of prognostic significance in the case

https://moodle.feu-nrmf.ph/mod/quiz/review.php?attempt=1175056&cmid=92821 6/9
1/4/22, 11:51 AM PLATING EXAM ON NEOPLASIA 2: Attempt review

Question 18

Correct

Mark 1.00 out of 1.00

PLATING ON NEOPLASIA 2 

A 38 year old female who was recently diagnosed to have Invasive Ductal Carcinoma of the left breast has an
immunohistochemistry result of: Estrogen Receptor (ER) Assay – Positive, Progesterone Receptor Assay – Positive,
HER2 Assay – Positive for HER2 Overexpression. The HER2 assay result is due to:

a. ERBB2 gene amplification 


b. ERBB1 gene rearrangement
c. ERBB2 gene methylation
d. ERBB1 gene point mutation

The correct answer is: ERBB2 gene amplification

Question 19
Incorrect

Mark 0.00 out of 1.00

PLATING ON NEOPLASIA 2

Which of the following constitutes the principal mechanism of the immune system to eradicate tumor cells?

a. Cytotoxicity mechanism by natural killer cells


b. Opsonization of tumor cells by antibodies with subsequent processing and display of antigen to Th cells
c. Processing of endogenous antigen and display to CD8+ T cells for killing of tumor cells
d. Cytokine secretion by T helper cells to stimulate the killing mechanisms of antigen presenting cells 

The correct answer is: Processing of endogenous antigen and display to CD8+ T cells for killing of tumor cells

Question 20
Incorrect

Mark 0.00 out of 1.00

PLATING ON NEOPLASIA 2

A 55 year old male who had several colonic polyps removed years ago now has positive fecal occult blood test.
Colonoscopy was done which showed a new colonic polyp. It was removed and was examined by a pathologist
with a finding of adenocarcinoma. The gene that is commonly implicated as the first gene to be mutated in the
development of this malignancy is a:

a. gene involved in apoptosis


b. tumor suppressor gene
c. protooncogene 
d. gene involved in DNA repair

The correct answer is: tumor suppressor gene

Previous activity
◄ PLATING ON NEOPLASIA 1

Jump to...

Next activity

PLATING EXAM ON ENVIRONMENTAL AND NUTRITIONAL DISEASES ►

https://moodle.feu-nrmf.ph/mod/quiz/review.php?attempt=1175056&cmid=92821 7/9
1/4/22, 11:52 AM PLATING EXAM ON DISEASES OF INFANCY AND CHILDHOOD: Attempt review

Question 15

Correct

Mark 1.00 out of 1.00

PLATING ON DISEASES OF INFANCY AND CHILDHOOD

Which of the following can cause non-immune hydrops?

a. All of the given options 


b. Chromosomal anomalies
c. Cardiovascular defects
d. Parvovirus B infection

The correct answer is: All of the given options

Question 16
Correct

Mark 1.00 out of 1.00

PLATING ON DISEASES OF INFANCY AND CHILDHOOD

Which of the following congenital anomalies exhibit an intrinsically abnormal developmental process?

a. Anencephaly as a malformation 
b. Deformed left shoulder associated with large uterine leiomyomas
c. Potter sequence due to rupture of fetal membrane
d. Absence of lower leg associated with amniotic band

The correct answer is: Anencephaly as a malformation

Question 17
Correct

Mark 1.00 out of 1.00

PLATING ON DISEASES OF INFANCY AND CHILDHOOD

In which of the following age groups is accident not the leading cause of death?

a. 1 to 4 years of age
b. 5 to 14 years of age
c. Infancy 
d. None of the given choices

The correct answer is: Infancy

Question 18
Correct

Mark 1.00 out of 1.00

PLATING ON DISEASES OF INFANCY AND CHILDHOOD

Assess whether the statement is True or False: Most cases of early onset neonatal sepsis are acquired
immediately after birth upon exposure to external environment.

a. True
b. False 

The correct answer is: False

https://moodle.feu-nrmf.ph/mod/quiz/review.php?attempt=1255201&cmid=96660 6/8
1/4/22, 11:52 AM PLATING EXAM ON DISEASES OF INFANCY AND CHILDHOOD: Attempt review

Question 19

Incorrect

Mark 0.00 out of 1.00

PLATING ON DISEASES OF INFANCY AND CHILDHOOD

In patients with phenylalanine hydroxylase deficiency, there will be decreased:

a. NADH
b. Tyrosine
c. DHPR 
d. Phenylalanine

The correct answer is: Tyrosine

Question 20
Correct

Mark 1.00 out of 1.00

PLATING ON DISEASES OF INFANCY AND CHILDHOOD

Which congenital anomaly cannot be explained on the basis of a single localized initiating defect?

a. Deformation
b. Sequence
c. Malformation
d. Malformation syndrome 

The correct answer is: Malformation syndrome

https://moodle.feu-nrmf.ph/mod/quiz/review.php?attempt=1255201&cmid=96660 7/8
1/4/22, 11:51 AM PLATING ON NEOPLASIA 1: Attempt review

Dashboard My courses Patho 2A 1st Sem SY 2021 MD2-A,B,C,D,E,F,G,H,I,J PLATING EXAMS

PLATING ON NEOPLASIA 1

Started on Monday, 4 October 2021, 12:01 PM


State Finished
Completed on Monday, 4 October 2021, 12:26 PM
Time taken 25 mins 2 secs
Marks 12.00/20.00
Grade 60.00 out of 100.00

Question 1
Correct

Mark 1.00 out of 1.00

PLATING ON NEOPLASIA 1

Patient is a diagnosed case of Squamous cell lung cancer presented with the most common paraneoplastic
syndrome of:

a. Cushing syndrome
b. MG
c. Acanthosis nigricans
d. hypercalcemia 

The correct answer is: hypercalcemia

Question 2
Correct

Mark 1.00 out of 1.00

PLATING ON NEOPLASIA 1

The most important prognostic indicator of cancer is the:

a. Detection of specific mutation or receptor by molecular diagnosis for personalize therapy


b. level of tumor marker
c. stage 
d. tumor grade

The correct answer is: stage

https://moodle.feu-nrmf.ph/mod/quiz/review.php?attempt=1151162&cmid=91552 1/8
1/4/22, 11:51 AM PLATING ON NEOPLASIA 1: Attempt review

Question 3

Incorrect

Mark 0.00 out of 1.00

PLATING ON NEOPLASIA 1

When the cancer cells infiltrate and destroy the basement membrane, the feature being displayed is:

a. differentiation
b. metastasis 
c. anaplasia
d. invasion

The correct answer is: invasion

Question 4
Correct

Mark 1.00 out of 1.00

PLATING ON NEOPLASIA 1

Dermoid cyst of the ovary is an example of:

a. choristoma
b. adenoma
c. hamartoma
d. benign mixed tumor
e. teratoma 

The correct answer is: teratoma

Question 5
Incorrect

Mark 0.00 out of 1.00

PLATING ON NEOPLASIA 1

Which of the following is a benign neoplasm?

a. ovarian mature cystic teratoma


b. GIT extranodal lymphoma 
c. testicular seminoma
d. liver hepatoma

The correct answer is: ovarian mature cystic teratoma

https://moodle.feu-nrmf.ph/mod/quiz/review.php?attempt=1151162&cmid=91552 2/8
1/4/22, 11:51 AM PLATING ON NEOPLASIA 1: Attempt review

Question 6

Correct

Mark 1.00 out of 1.00

PLATING ON NEOPLASIA 1

Based on the 2019 estimated cancer data, which of the following is incorrect?

a. prostate cancer is the most common cancer in adult males


b. Breast cancer is the most common cancer in adult females
c. lung cancer is the common cause of death in adults for both sexes
d. lung cancer is the second most common cancer in adult male only 

The correct answer is: lung cancer is the second most common cancer in adult male only

Question 7
Correct

Mark 1.00 out of 1.00

PLATING ON NEOPLASIA 1

A 56 y/o Caucasian who is fond of sun bathing was diagnosed of melanoma. The implicated agent is:

a. UVB 
b. UVA
c. Ionizing electromagnetic forces
d. Alpha & beta particles

The correct answer is: UVB

Question 8
Incorrect

Mark 0.00 out of 1.00

PLATING ON NEOPLASIA 1

Of the following features of benign neoplasm, which is the most important in differentiating with malignant
counterpart?

a. amenable to surgical removal


b. cannot spread to other sites
c. relatively innocent
d. remains localized 

The correct answer is: cannot spread to other sites

https://moodle.feu-nrmf.ph/mod/quiz/review.php?attempt=1151162&cmid=91552 3/8
1/4/22, 11:51 AM PLATING ON NEOPLASIA 1: Attempt review

Question 9

Correct

Mark 1.00 out of 1.00

PLATING ON NEOPLASIA 1

Nearly 80% of the total decrease death rate among men is due to the following EXCEPT:

a. colorectal CA
b. pancreas 
c. lung
d. prostate

The correct answer is: pancreas

Question 10
Correct

Mark 1.00 out of 1.00

PLATING ON NEOPLASIA 1

The single most important environmental factor contributing to premature death in the US is:

a. diet
b. infectious agents
c. alcohol
d. cigarette smoking 

The correct answer is: cigarette smoking

Question 11
Correct

Mark 1.00 out of 1.00

PLATING ON NEOPLASIA 1

The growth and spread of a tumor is dependent or determined by its:

a. capsule
b. parenchyma
c. none of the choices
d. stroma 

The correct answer is: stroma

https://moodle.feu-nrmf.ph/mod/quiz/review.php?attempt=1151162&cmid=91552 4/8
1/4/22, 11:51 AM PLATING ON NEOPLASIA 1: Attempt review

Question 12

Correct

Mark 1.00 out of 1.00

PLATING ON NEOPLASIA 1

A 31 y/o female diagnosed of invasive ductal carcinoma presented with headache associated with change in
sensorium. Cranial CT-scan with contrast revealed multiple enhancing lesion both in the meninges and brain
parenchyma. The pathway by which the cancer cells spread and reached the brain is by:

a. seeding
b. hematogenous 
c. lymphatics
d. none of the choices

The correct answer is: hematogenous

Question 13
Incorrect

Mark 0.00 out of 1.00

PLATING ON NEOPLASIA 1

This is a benign epithelial tumor:

a. liver adenoma
b. Osteoid osteoma
c. GIT lipoma
d. Ovarian fibroma 

The correct answer is: liver adenoma

Question 14
Correct

Mark 1.00 out of 1.00

PLATING ON NEOPLASIA 1

Included in the small round blue cell tumors in infants and childhood EXCEPT:

a. Carcinoid tumor 
b. Wilms tumor
c. Acute leukemias
d. rhabdomyosarcomas

The correct answer is: Carcinoid tumor

https://moodle.feu-nrmf.ph/mod/quiz/review.php?attempt=1151162&cmid=91552 5/8
1/4/22, 11:51 AM PLATING ON NEOPLASIA 1: Attempt review

Question 15

Incorrect

Mark 0.00 out of 1.00

PLATING ON NEOPLASIA 1

Patients suffering of cancer cachexia presents with the following EXCEPT:

a. progressive loss of muscle mass of the extremities only


b. weakness
c. anorexia
d. Anemia 

The correct answer is: progressive loss of muscle mass of the extremities only

Question 16
Correct

Mark 1.00 out of 1.00

PLATING ON NEOPLASIA 1

The hallmark of malignant transformation is:

a. dysplasia
b. anaplasia 
c. hyperplasia
d. metaplasia

The correct answer is: anaplasia

Question 17
Incorrect

Mark 0.00 out of 1.00

PLATING ON NEOPLASIA 1

The following tumor and risk are correctly paired EXCEPT:

a. Gastric cancer & Chronic atrophic gastritis associated with H. pylori


b. Colonic adenocarcinoma & villous adenoma 
c. Endometrial carcinoma and endocervical metaplasia
d. Lymphoma & T cell immunodeficiency associated viral infection

The correct answer is: Endometrial carcinoma and endocervical metaplasia

https://moodle.feu-nrmf.ph/mod/quiz/review.php?attempt=1151162&cmid=91552 6/8
1/4/22, 11:51 AM PLATING EXAM ON ENVIRONMENTAL AND NUTRITIONAL DISEASES: Attempt review

Dashboard My courses Patho 2A 1st Sem SY 2021 MD2-A,B,C,D,E,F,G,H,I,J PLATING EXAMS

PLATING EXAM ON ENVIRONMENTAL AND NUTRITIONAL DISEASES

Started on Monday, 25 October 2021, 12:00 PM


State Finished
Completed on Monday, 25 October 2021, 12:27 PM
Time taken 26 mins 14 secs
Marks 11.00/20.00
Grade 55.00 out of 100.00

Question 1
Correct

Mark 1.00 out of 1.00

PLATING ON ENVIRONMENTAL AND NUTRITIONAL DISEASES

Nicotine in cigarette smoke is most likely to result in ________________.

a. mucosal irritation
b. depression 
c. definitely cancer development
d. toxicity to the cilia

The correct answer is: depression

Question 2
Incorrect

Mark 0.00 out of 1.00

PLATING ON ENVIRONMENTAL AND NUTRITIONAL DISEASES

Burn injury reaching the muscles underneath the subcutaneous tissue.

a. partial thickness burn


b. fourth degree burn
c. third degree burn 

The correct answer is: fourth degree burn

https://moodle.feu-nrmf.ph/mod/quiz/review.php?attempt=1236543&cmid=94660 1/8
1/4/22, 11:51 AM PLATING ON NEOPLASIA 1: Attempt review

Previous activity
◄ PLATING ON THE DISEASES OF THE IMMUNE SYSTEM PART 2

Jump to...

Next activity

PLATING EXAM ON NEOPLASIA 2 ►

Stay in touch

http://www.feu-nrmf.ph

+632 8983 8338 / +639176272623

info@feu-nrmf.ph

 Data retention summary

 Get the mobile app

https://moodle.feu-nrmf.ph/mod/quiz/review.php?attempt=1151162&cmid=91552 8/8
1/4/22, 11:51 AM PLATING EXAM ON ENVIRONMENTAL AND NUTRITIONAL DISEASES: Attempt review

Question 3

Correct

Mark 1.00 out of 1.00

PLATING ON ENVIRONMENTAL AND NUTRITIONAL DISEASES

Peripheral blood smear findings of hypochromic microcytic anemia with ringed sideroblasts. What is the likely
diagnosis?

a. Sulfur dioxide poisoning


b. Formaldehyde toxicity
c. Cadmium toxicity
d. Lead toxicity 

The correct answer is: Lead toxicity

Question 4
Correct

Mark 1.00 out of 1.00

PLATING ON ENVIRONMENTAL AND NUTRITIONAL DISEASES

The poison of kings and the king of poisons.

a. Cadmium
b. Arsenic 
c. Lead
d. Mercury

The correct answer is: Arsenic

Question 5
Correct

Mark 1.00 out of 1.00

PLATING ON ENVIRONMENTAL AND NUTRITIONAL DISEASES

Granulocytopenia, aplastic anemia and pancytopenia maybe associated with which of the following drugs?

a. Salicylates
b. Dabigatran
c. Chloramphenicol 
d. Doxorubicin

The correct answer is: Chloramphenicol

https://moodle.feu-nrmf.ph/mod/quiz/review.php?attempt=1236543&cmid=94660 2/8
1/4/22, 11:50 AM PLATING ON THE DISEASES OF THE IMMUNE SYSTEM PART 2: Attempt review

Question 3

Correct

Mark 1.00 out of 1.00

PLATING ON DISEASES OF THE IMMUNE SYSTEM PART 2

Which of the following disease-etiology pairing is CORRECT?

a. acute cellular  rejection – lymphocytes reacting to alloantigens in the vessel wall

b. chronic cellular rejection- CD4 T cells


c. hyperacute rejection- preformed antibodies in the serum of recipient 
d. acute antibody mediated rejection- circulating antibodies

The correct answer is: hyperacute rejection- preformed antibodies in the serum of recipient

Question 4
Correct

Mark 1.00 out of 1.00

PLATING ON DISEASES OF THE IMMUNE SYSTEM PART 2

The earliest lesion seen in the salivary and lacrimal glands among patients with Sjogren syndrome:

a. Periductal and perivascular fibrosis


b. Formation of lymphoid follicles
c. Periductal and perivascular lymphocytic infiltration 
d. Epithelial hyperplasia

The correct answer is: Periductal and perivascular lymphocytic infiltration

Question 5
Incorrect

Mark 0.00 out of 1.00

PLATING ON DISEASES OF THE IMMUNE SYSTEM PART 2

Which is the proper sequence of HIV infection of a CD4 T cell:

a. gp41 binds to CD4 - gp41 membrane penetration- release of viral genomic RNA 
b. release of viral genomic RNA- gp120 binds to CD4 – gp41 membrane penetration

c. p24 binds to CD4- release of viral genomic RNA – gp41 membrane penetration

d. gp120 binds to CD4 – gp41 membrane penetration – release of viral genomic RNA

The correct answer is:


gp120 binds to CD4 – gp41 membrane penetration – release of viral genomic RNA

https://moodle.feu-nrmf.ph/mod/quiz/review.php?attempt=1133229&cmid=90454 2/8
1/4/22, 11:50 AM PLATING ON THE DISEASES OF THE IMMUNE SYSTEM PART 2: Attempt review

Question 6

Correct

Mark 1.00 out of 1.00

PLATING ON DISEASES OF THE IMMUNE SYSTEM PART 2

A 4 year old boy with recurrent viral infections was seen at the emergency department. A generalized eczematous
rash was noted. Laboratory examinations showed decrease in platelet count. What is your diagnosis?

a. Ataxia telangiectasia
b. Di George syndrome
c. Severe combined immunodeficiency
d. Wiskott- Aldrich syndrome 

The correct answer is: Wiskott- Aldrich syndrome

Question 7
Correct

Mark 1.00 out of 1.00

PLATING ON DISEASES OF THE IMMUNE SYSTEM PART 2

Amyloid deposits are often seen in this type of thyroid malignancy:

a. Anaplastic carcinoma
b. Follicular carcinoma
c. Papillary carcinoma
d. Medullary carcinoma 

The correct answer is: Medullary carcinoma

Question 8
Incorrect

Mark 0.00 out of 1.00

PLATING ON DISEASES OF THE IMMUNE SYSTEM PART 2

These events are seen in the clinical latency stages of HIV infection:

a. infection of dendritic cells


b. CD4+ T cell infection 
c. spleen and lymph nodes are the sites of viral replication
d. destruction of lymphoid tissues

The correct answer is: spleen and lymph nodes are the sites of viral replication

https://moodle.feu-nrmf.ph/mod/quiz/review.php?attempt=1133229&cmid=90454 3/8
1/4/22, 11:50 AM PLATING ON THE DISEASES OF THE IMMUNE SYSTEM PART 2: Attempt review

Question 9

Incorrect

Mark 0.00 out of 1.00

PLATING ON DISEASES OF THE IMMUNE SYSTEM PART 2

Possible explanation/s as to the reason why auto-immune disorders are more frequent in females:

a. the role of the X chromosome


b. increased susceptibility to infections
c. more prone to anemia
d. all of the choices are correct 

The correct answer is: the role of the X chromosome

Question 10
Incorrect

Mark 0.00 out of 1.00

PLATING ON DISEASES OF THE IMMUNE SYSTEM PART 2

Clinical characteristics of systemic sclerosis:

a. all of the choices are correct


b. a disease of black women in their 50s
c. usually preceded by an upper respiratory tract infection 
d. increased risk of developing B cell lymphomas

The correct answer is: a disease of black women in their 50s

Question 11
Correct

Mark 1.00 out of 1.00

PLATING ON DISEASES OF THE IMMUNE SYSTEM PART 2

A 5 year old female with facial deformities and congenital heart disease was brought to the attention of
pediatrician due to repeated viral infections of the upper respiratory tract. Cervical lymph nodes were enlarged.
Lymph node biopsy revealed absent T cells . What is the diagnosis?

a. Wiskott- Aldrich syndrome


b. X linked severe combined immunodeficiency
c. Bruton aggamaglobulinemia
d. Di George’s syndrome

The correct answer is:


Di George’s syndrome

https://moodle.feu-nrmf.ph/mod/quiz/review.php?attempt=1133229&cmid=90454 4/8
1/4/22, 11:50 AM PLATING ON THE DISEASES OF THE IMMUNE SYSTEM PART 2: Attempt review

Question 12

Correct

Mark 1.00 out of 1.00

PLATING ON DISEASES OF THE IMMUNE SYSTEM PART 2

CD8 T cells usually attacks this part of the renal allograft:

a. podocytes
b. Bowmans capsule
c. renal vasculature 
d. mesangium

The correct answer is: renal vasculature

Question 13
Correct

Mark 1.00 out of 1.00

PLATING ON DISEASES OF THE IMMUNE SYSTEM PART 2

AIDS is characterized by:

a. life threatening opportunistic infections, secondary malignant neoplasms and CNS infections
b. life threatening opportunistic infections, secondary benign neoplasms and CNS infections
c. life threatening opportunistic infections, secondary malignant neoplasms and CNS manifestations 
d. life threatening opportunistic infections, secondary malignant neoplasms and CNS tumors

The correct answer is: life threatening opportunistic infections, secondary malignant neoplasms and CNS
manifestations

Question 14
Incorrect

Mark 0.00 out of 1.00

PLATING ON DISEASES OF THE IMMUNE SYSTEM PART 2

Which of the following disease-etiology pairing is correct?

a. Ataxia telangiectasia- isotype switched antibodies


b. X linked lymphoproliferative disease- BAFF mutation

c. X linked lymphoproliferative disease- BAFF mutation


d. Bruton’s agammaglobulinemia- Adenosine deaminase deficiency

The correct answer is: Ataxia telangiectasia- isotype switched antibodies

https://moodle.feu-nrmf.ph/mod/quiz/review.php?attempt=1133229&cmid=90454 5/8
1/4/22, 11:50 AM PLATING ON THE DISEASES OF THE IMMUNE SYSTEM PART 2: Attempt review

Question 15

Correct

Mark 1.00 out of 1.00

PLATING ON DISEASES OF THE IMMUNE SYSTEM PART 2

Biopsy of the lymphoid organs of patients with Common variable immunodeficiency shows:

a. hyperplastic germinal centers 


b. germinal centers are absent
c. T cells are depleted
d. B cells are absent to reduced

The correct answer is: hyperplastic germinal centers

Question 16
Correct

Mark 1.00 out of 1.00

PLATING ON DISEASES OF THE IMMUNE SYSTEM PART 2

Which of the following primary immunodeficiency syndromes has a similar presentation with Bruton
aggammaglobulinemia:

a. Common variable immunodeficiency 


b. X linked lymphoproliferative disease
c. Severe combined immunodeficiency
d. Bare lymphocyte syndrome

The correct answer is: Common variable immunodeficiency

Question 17
Incorrect

Mark 0.00 out of 1.00

PLATING ON DISEASES OF THE IMMUNE SYSTEM PART 2

Which of the following chronic rejection reaction is caused by T cells and anti-bodies?

a. transplant glomerulopathy
b. reaction vasculitis 
c. tubulitis
d. graft arteriosclerosis

The correct answer is: graft arteriosclerosis

https://moodle.feu-nrmf.ph/mod/quiz/review.php?attempt=1133229&cmid=90454 6/8
1/4/22, 11:50 AM PLATING ON THE DISEASES OF THE IMMUNE SYSTEM PART 2: Attempt review

Question 18

Correct

Mark 1.00 out of 1.00

PLATING ON DISEASES OF THE IMMUNE SYSTEM PART 2

The concept of molecular mimicry is the presumptive pathogenic mechanisms in the following diseases:

a. Type 1 DM
b. Rheumatic heart disease 
c. Graves disease
d. SLE

The correct answer is: Rheumatic heart disease

Question 19
Correct

Mark 1.00 out of 1.00

PLATING ON DISEASES OF THE IMMUNE SYSTEM PART 2

Systemic sclerosis patients with anti-centromere antibody usually presents as:

a. pulmonary artery hypertension


b. malignant hypertension
c. limited type of scleroderma 
d. diffuse scleroderma

The correct answer is: limited type of scleroderma

Question 20
Correct

Mark 1.00 out of 1.00

PLATING ON DISEASES OF THE IMMUNE SYSTEM PART 2

Characteristics of patients with Sjogren syndrome:

a. Females in their 50s 


b. Elderly females
c. Females in the reproductive age group
d. Black females in their teens

The correct answer is: Females in their 50s

https://moodle.feu-nrmf.ph/mod/quiz/review.php?attempt=1133229&cmid=90454 7/8
1/4/22, 11:50 AM PLATING ON THE DISEASES OF THE IMMUNE SYSTEM PART 2: Attempt review

Previous activity
◄ PLATING ON DISEASES OF THE IMMUNE SYSTEM PART 1

Jump to...

Next activity

PLATING ON NEOPLASIA 1 ►

Stay in touch

http://www.feu-nrmf.ph

+632 8983 8338 / +639176272623

info@feu-nrmf.ph

 Data retention summary

 Get the mobile app

https://moodle.feu-nrmf.ph/mod/quiz/review.php?attempt=1133229&cmid=90454 8/8
1/4/22, 11:50 AM PLATING ON DISEASES OF THE IMMUNE SYSTEM PART 1: Attempt review

Dashboard My courses Patho 2A 1st Sem SY 2021 MD2-A,B,C,D,E,F,G,H,I,J PLATING EXAMS

PLATING ON DISEASES OF THE IMMUNE SYSTEM PART 1

Started on Monday, 20 September 2021, 12:00 PM


State Finished
Completed on Monday, 20 September 2021, 12:23 PM
Time taken 23 mins 7 secs
Marks 8.00/20.00
Grade 40.00 out of 100.00

Question 1
Incorrect

Mark 0.00 out of 1.00

PLATING ON IMMUNE SYSTEM PART I

The secretion profile of Th2 cells include the following, except:

a. IL-17
b. IL-5
c. IL-13 
d. IL-4

The correct answer is: IL-17

Question 2
Incorrect

Mark 0.00 out of 1.00

PLATING ON IMMUNE SYSTEM PART I

An anemic patient who had blood transfusion is presenting with flank pain and fever. The urine was also noted to
be brown in color. Transfusion reaction investigation was conducted and the major blood grouping (ABO)
between the blood unit that was transfused and the recipient did not match as a wrong unit was released by the
laboratory staff. What is the most possible mechanism of the cell destruction in this case of hypersensitivity?

a. Neutrophil degranulation
b. Phagocytosis
c. Binding of antibody stimulates apoptosis 
d. CD8+ T cell activation causes cell cytotoxicity

The correct answer is: Phagocytosis

https://moodle.feu-nrmf.ph/mod/quiz/review.php?attempt=1117443&cmid=89374 1/9
1/4/22, 11:50 AM PLATING ON DISEASES OF THE IMMUNE SYSTEM PART 1: Attempt review

Question 3

Incorrect

Mark 0.00 out of 1.00

PLATING ON IMMUNE SYSTEM PART I (ADL) 

A
43 year old female suddenly complained of difficulty of breathing after
consuming crab claw sandwich. She
was brought to hospital but was pronounced
“dead on arrival” with diagnosis is hypersensitivity (hps) reaction. 
Which of the following cells and substances
are major contributory to the condition?

a. mast cells, IgE, Th2


b. neutrophils, IgE, Th1
c. basophils, IgE, Th1 
d. eosinophils, IgA, Th2

The correct answer is: mast cells, IgE, Th2

Question 4
Correct

Mark 1.00 out of 1.00

PLATING ON IMMUNE SYSTEM PART I

Antibodies produced by exposure to a T dependent antigen months ago can participate in which of the following
immune responses?

a. classical activation of the complement system


b. opsonization with subsequent phagocytosis
c. binding and neutralizing microbes
d. All of the given options 

The correct answer is: All of the given options

Question 5
Correct

Mark 1.00 out of 1.00

PLATING ON IMMUNE SYSTEM PART I

The following are general features of hypersensitivity, except:

a. Hypersensitivity may result from imbalance between the effector immune mechanisms and the control
mechanisms
b. Hypersensitivity reactions are excessive immune reactions to non-self antigens only 
c. The mechanisms of tissue injury in hypersensitivity are the same as the mechanisms against infectious
pathogens
d. Hypersensitivity is often associated with the inheritance of particular susceptibility gene

The correct answer is: Hypersensitivity reactions are excessive immune reactions to non-self antigens only

https://moodle.feu-nrmf.ph/mod/quiz/review.php?attempt=1117443&cmid=89374 2/9
1/4/22, 11:50 AM PLATING ON DISEASES OF THE IMMUNE SYSTEM PART 1: Attempt review

Question 6

Correct

Mark 1.00 out of 1.00

PLATING ON IMMUNE SYSTEM PART I (ADL)

A patient was diagnosed with polyarteritis nodosa, the reaction is due to

a. delayed type hps


b. immune complex deposition 
c. cell mediated mechanism
d. antibody mediated hps

The correct answer is: immune complex deposition

Question 7
Correct

Mark 1.00 out of 1.00

PLATING ON IMMUNE SYSTEM PART I (ADL)

Which is considered NOT an immune privileged site?

a. testis
b. kidney 
c. brain
d. eye

The correct answer is: kidney

Question 8
Incorrect

Mark 0.00 out of 1.00

PLATING ON IMMUNE SYSTEM PART I

What is the immune mechanism of hypersensitivity among patients with Myasthenia Gravis?

a. Antibody dependent cellular cytotoxicity 


b. Continuous activation of CD4 T cell with cytokine secretion
c. Immune complex deposition and activation of inflammation
d. Cellular dysfunction

The correct answer is: Cellular dysfunction

https://moodle.feu-nrmf.ph/mod/quiz/review.php?attempt=1117443&cmid=89374 3/9
1/4/22, 11:50 AM PLATING ON DISEASES OF THE IMMUNE SYSTEM PART 1: Attempt review

Question 9

Incorrect

Mark 0.00 out of 1.00

PLATING ON IMMUNE SYSTEM PART I

Which of the following conditions will trigger an NK cell to kill a virus-infected lymphocyte with which it is
interacting?

a. When the lymphocyte presents the antigen to the NK cell using MHC class I 
b. When the lymphocyte synthesizes Type I interferon
c. Absence of MHC class I on the surface of lymphocytes
d. When the inhibitory receptor of NK cell binds with its ligand, NK cell will phagocytose the lymphocyte

The correct answer is: Absence of MHC class I on the surface of lymphocytes

Question 10
Incorrect

Mark 0.00 out of 1.00

PLATING ON IMMUNE SYSTEM PART I (ADL)

The typical heart finding in SLE is characterized by

a. bread and butter pericarditis 


b. mitral valve prolapse
c. fibrinous pericarditis
d. vegetations on the mitral valve leaflet

The correct answer is: vegetations on the mitral valve leaflet

Question 11
Correct

Mark 1.00 out of 1.00

PLATING ON IMMUNE SYSTEM PART I (ADL)

A 13 y/o male had biopsy of cervical lymph node which revealed a granuloma with central area of caseation
surrounded by lymphocytes, macrophages, plasma cells and multinucleated giant cells. This type of
hypersensitivity reaction is mediated by

a. mast cells
b. T cells 
c. basophils
d. B cells

The correct answer is: T cells

https://moodle.feu-nrmf.ph/mod/quiz/review.php?attempt=1117443&cmid=89374 4/9
1/4/22, 11:50 AM PLATING ON DISEASES OF THE IMMUNE SYSTEM PART 1: Attempt review

Question 12

Correct

Mark 1.00 out of 1.00

PLATING ON IMMUNE SYSTEM PART I

A 40 year old has a non-healing skin wound in the leg. The organisms in the wound have started to spread though
the blood. Which of the following organs is the most important in forming immediate adaptive response to the
organisms in the blood?

a. Spleen 
b. Axillary lymph node
c. Peyers patches
d. MALT

The correct answer is: Spleen

Question 13
Incorrect

Mark 0.00 out of 1.00

PLATING ON IMMUNE SYSTEM PART I

Which of the following statements is not true about IgE in Type I hypersensitivity?

a. Th2 cells secrete IL-4 that stimulates B cell differentiation into IgE producing plasma cells 
b. The IgE on mast cells undergo class switching to IgG after the allergic reaction
c. The Fc portion of IgE binds to the Fc portion on mast cell surfaces
d. The IgE on mast cell surface can directly recognize allergen to which it exhibits specificity

The correct answer is: The IgE on mast cells undergo class switching to IgG after the allergic reaction

Question 14
Correct

Mark 1.00 out of 1.00

PLATING ON IMMUNE SYSTEM PART I (ADL)

Prolonged and irreversible functional inactivation of lymphocytes is known a

a. negative selection
b. receptor editing
c. anergy 
d. AIRE

The correct answer is: anergy

https://moodle.feu-nrmf.ph/mod/quiz/review.php?attempt=1117443&cmid=89374 5/9
1/4/22, 11:50 AM PLATING ON DISEASES OF THE IMMUNE SYSTEM PART 1: Attempt review

Question 15

Correct

Mark 1.00 out of 1.00

PLATING ON IMMUNE SYSTEM PART I (ADL)

The following can lead to failure of self tolerance and eventually to autoimmunity, EXCEPT

a. inheritance of susceptibility genes


b. tissue injury
c. infections
d. no exception 

The correct answer is: no exception

Question 16
Incorrect

Mark 0.00 out of 1.00

PLATING ON IMMUNE SYSTEM PART I

Peptides from phagocytosed bacterial organisms will be displayed to a T cell. What will be the co-receptor of the
TCR in the reaction?

a. CD16 
b. CD1
c. CD4
d. CD8

The correct answer is: CD4

Question 17
Incorrect

Mark 0.00 out of 1.00

PLATING ON IMMUNE SYSTEM PART I

Which of the following statements is true about dendritic cell being considered as the most important antigen
presenting cell to T cells?

a. They express many receptors such as TLRs for capturing antigens


b. All of the given choices 
c. Because they have low CD80 but high MHC molecules
d. They do not need to travel to present antigen to T cells are they are all located in the T cell zone of lymph
nodes

The correct answer is: They express many receptors such as TLRs for capturing antigens

https://moodle.feu-nrmf.ph/mod/quiz/review.php?attempt=1117443&cmid=89374 6/9
1/4/22, 11:50 AM PLATING ON DISEASES OF THE IMMUNE SYSTEM PART 1: Attempt review

Question 18

Incorrect

Mark 0.00 out of 1.00

PLATING ON IMMUNE SYSTEM PART I (ADL)

A patient with autoimmune hemolytic anemia exhibits which mechanism of hypersensitivity reaction?

a. Fc receptor mediated inflammation 


b. complement mediated inflammation
c. antibody mediated cellular dysfunction
d. opsonization and phagocytosis

The correct answer is: opsonization and phagocytosis

Question 19
Incorrect

Mark 0.00 out of 1.00

PLATING ON IMMUNE SYSTEM PART I (ADL)

Which is the key cell player in hps seen in tuberculin testing

a. CD4 T cells
b. Macrophage effector cells
c. CD8 T cells
d. memory B cells 

The correct answer is: CD4 T cells

Question 20
Incorrect

Mark 0.00 out of 1.00

PLATING ON IMMUNE SYSTEM PART I (ADL)

An 18 year old female was recently diagnosed with SLE. Which of the following manifestations is more commonly
encountered in this condition

a. renal 
b. weight loss
c. skin
d. hematologic

The correct answer is: hematologic

Previous activity
◄ PLATING ON GENETIC DISORDERS

Jump to...

Next activity
PLATING ON THE DISEASES OF THE IMMUNE SYSTEM PART 2 ►

https://moodle.feu-nrmf.ph/mod/quiz/review.php?attempt=1117443&cmid=89374 7/9
1/4/22, 11:50 AM PLATING ON DISEASES OF THE IMMUNE SYSTEM PART 1: Attempt review

Stay in touch

http://www.feu-nrmf.ph

https://moodle.feu-nrmf.ph/mod/quiz/review.php?attempt=1117443&cmid=89374 8/9
1/4/22, 11:51 AM PLATING EXAM ON NEOPLASIA 2: Attempt review

Question 9

Correct

Mark 1.00 out of 1.00

PLATING ON NEOPLASIA 2

A 60 year old female was diagnosed to have breast cancer 5 years ago. The tumor has recurred with metastatic
deposits to the liver, lung and a vertebral bone. Which of the following conditions is/are expected following
secretion of PTHRP by the tumor cells in the bone?

a. All provided choices are correct 


b. There will be upregulated expression of RANKL in osteoblast
c. Hypercalcemia following degradation of the bone matrix
d. There will be release of growth factors like IGF that has pro-growth effect on cancer cells

The correct answer is: All provided choices are correct

Question 10
Correct

Mark 1.00 out of 1.00

PLATING ON NEOPLASIA 2

The following molecules are tumor suppressors, except:

a. RB
b. JAK2 
c. P53
d. APC

The correct answer is: JAK2

Question 11
Correct

Mark 1.00 out of 1.00

PLATING ON NEOPLASIA 2

The following are roles of MHC class I molecule in anti-tumor response of the immune system, except:

a. to display processed endogenous tumor neoantigens to CTLs


b. the display the processed exogenous antigen through the use of proteasome by dendritic cells to CTLs 
c. in presentation of antigen by tumor cells to NK cells as a first line of defense
d. their downregulation in tumor cells allow killing of such cells by NK cells

The correct answer is: the display the processed exogenous antigen through the use of proteasome by dendritic
cells to CTLs

https://moodle.feu-nrmf.ph/mod/quiz/review.php?attempt=1175056&cmid=92821 4/9
1/4/22, 11:50 AM PLATING ON GENETIC DISORDERS: Attempt review

Dashboard My courses Patho 2A 1st Sem SY 2021 MD2-A,B,C,D,E,F,G,H,I,J PLATING EXAMS

PLATING ON GENETIC DISORDERS

Started on Monday, 13 September 2021, 12:00 PM


State Finished
Completed on Monday, 13 September 2021, 12:27 PM
Time taken 27 mins 19 secs
Marks 15.00/20.00
Grade 75.00 out of 100.00

Question 1
Correct

Mark 1.00 out of 1.00

PLATING ON GENETIC DISORDERS (RRA)

A pediatrician called a geneticist because she wants confirmation that his patient has Fragile X syndrome. Based
on what you know about the disease, which of the following methods would you recommend?

a. Cytogenomic Array Technology


b. FISH
c. PCR 
d. Karyotype

The correct answer is: PCR

Question 2
Correct

Mark 1.00 out of 1.00

PLATING ON GENETIC DISORDERS (RRA)

A 20-year-old primigravid was seen in the OB admitting section due to decreased fetal sounds. Contractions were
induced and she delivered a stillborn fetus at 19 weeks age of gestation and subsequently sent for autopsy.
Autopsy revealed marked hydrops fetalis, cystic hygroma of the neck, aortic coarctation and horseshoe kidneys.
Which of the following karyotypes does the fetus likely have?

a. 45, X 
b. 47, XX, +21
c. 47, XXY
d. 46, XX/47,XX,+13

The correct answer is: 45, X

https://moodle.feu-nrmf.ph/mod/quiz/review.php?attempt=1107739&cmid=88124 1/9
1/4/22, 11:51 AM PLATING EXAM ON NEOPLASIA 2: Attempt review

Question 18

Correct

Mark 1.00 out of 1.00

PLATING ON NEOPLASIA 2 

A 38 year old female who was recently diagnosed to have Invasive Ductal Carcinoma of the left breast has an
immunohistochemistry result of: Estrogen Receptor (ER) Assay – Positive, Progesterone Receptor Assay – Positive,
HER2 Assay – Positive for HER2 Overexpression. The HER2 assay result is due to:

a. ERBB2 gene amplification 


b. ERBB1 gene rearrangement
c. ERBB2 gene methylation
d. ERBB1 gene point mutation

The correct answer is: ERBB2 gene amplification

Question 19
Incorrect

Mark 0.00 out of 1.00

PLATING ON NEOPLASIA 2

Which of the following constitutes the principal mechanism of the immune system to eradicate tumor cells?

a. Cytotoxicity mechanism by natural killer cells


b. Opsonization of tumor cells by antibodies with subsequent processing and display of antigen to Th cells
c. Processing of endogenous antigen and display to CD8+ T cells for killing of tumor cells
d. Cytokine secretion by T helper cells to stimulate the killing mechanisms of antigen presenting cells 

The correct answer is: Processing of endogenous antigen and display to CD8+ T cells for killing of tumor cells

Question 20
Incorrect

Mark 0.00 out of 1.00

PLATING ON NEOPLASIA 2

A 55 year old male who had several colonic polyps removed years ago now has positive fecal occult blood test.
Colonoscopy was done which showed a new colonic polyp. It was removed and was examined by a pathologist
with a finding of adenocarcinoma. The gene that is commonly implicated as the first gene to be mutated in the
development of this malignancy is a:

a. gene involved in apoptosis


b. tumor suppressor gene
c. protooncogene 
d. gene involved in DNA repair

The correct answer is: tumor suppressor gene

Previous activity
◄ PLATING ON NEOPLASIA 1

Jump to...

Next activity

PLATING EXAM ON ENVIRONMENTAL AND NUTRITIONAL DISEASES ►

https://moodle.feu-nrmf.ph/mod/quiz/review.php?attempt=1175056&cmid=92821 7/9
1/4/22, 11:50 AM PLATING ON GENETIC DISORDERS: Attempt review

Question 6

Incorrect

Mark 0.00 out of 1.00

PLATING ON GENETIC DISORDERS (RRA)

Given the karyotype 45,X/47,XXX which of the following statement is true?

a. Isochromosome formation has occurred in this patient. 


b. This represents a special form of deletion with fusion of damaged ends.
c. This represents a mosaic karyotype of Down Syndrome.
d. This is likely due to a mitotic error during cleavage of the fertilized ovum or in somatic cells.

The correct answer is: This is likely due to a mitotic error during cleavage of the fertilized ovum or in somatic cells.

Question 7
Correct

Mark 1.00 out of 1.00

PLATING ON GENETIC DISORDERS (RRA)

A 10-year-old boy was seen in the pediatrician’s clinic (again) due to coughs and colds. The pediatrician was
alarmed because the infections seem to be recurrent, with the patient having otitis media, pneumonia, a diarrhea
in past 6 months. Physical examination shows a murmur suggesting of a congenital heart disease. CT scan of the
chest reveals a small thymus. Which of the following genetic defects is likely present in this patient?

a. Karyotype of 46,XX/47,XX, +18


b. Small deletion of band q11.2 in the long arm of chromosome 22 
c. Karyotype of 47, XYY
d. More than 200 repeats of CGG

The correct answer is: Small deletion of band q11.2 in the long arm of chromosome 22

Question 8
Correct

Mark 1.00 out of 1.00

PLATING ON GENETIC DISORDERS (RGB)

A ten month old baby girl was noted to show muscular flaccidity and failure to develop normal milestone was
brought to a Pediatrician. Cherry red spot was noted in the cornea. Enzyme assay revealed deficiency of
hexoaminidase A. Which of the following morphologic findings would you expect to see in the baby?

a. Neurons with large lipid cytoplasmic vacuoles 


b. Distended phagocytic cells with crumpled tissue paper appearance
c. Presence of balloon cells and zebra bodies in phagocytic cells
d. Marked visceral accumulation of sphingomyelin

The correct answer is: Neurons with large lipid cytoplasmic vacuoles

https://moodle.feu-nrmf.ph/mod/quiz/review.php?attempt=1107739&cmid=88124 3/9
1/4/22, 11:50 AM PLATING ON GENETIC DISORDERS: Attempt review

Question 9

Correct

Mark 1.00 out of 1.00

PLATING ON GENETIC DISORDERS (RGB)

Which of the following genetic disorders of non-classic inheritance is transmitted by ovum?

a. Gonadal mosaicism
b. Genomic imprinting
c. Mitochondrial DNA mutation 
d. Triplet repeat mutation

The correct answer is: Mitochondrial DNA mutation

Question 10
Correct

Mark 1.00 out of 1.00

PLATING ON GENETIC DISORDERS (RRA)

Which of the following statements is true regarding the Lyon Hypothesis?

a. Inactivation of either the maternal or paternal X occurs at random at day 5.5 of embryonic life. 
b. Different X chromosomes can be inactivated in a subset of population of cells.
c. If there is X inactivation of either the maternal or paternal X, it can undergo heteropyknosis forming a Lyon
Body
d. Normally, both X chromosomes are genetically active.

The correct answer is: Inactivation of either the maternal or paternal X occurs at random at day 5.5 of embryonic
life.

Question 11
Correct

Mark 1.00 out of 1.00

PLATING ON GENETIC DISORDERS (RGB)

Deficiency of functional lysosomal enzymes leading to incomplete catabolism of substrate will cause which of the
following abnormalities?

a. Secondary accumulations
b. Lack of transport proteins
c. Primary accumulation
d. Accumulation of partially degraded metabolites 

The correct answers are: Primary accumulation, Accumulation of partially degraded metabolites

https://moodle.feu-nrmf.ph/mod/quiz/review.php?attempt=1107739&cmid=88124 4/9
1/4/22, 11:50 AM PLATING ON GENETIC DISORDERS: Attempt review

Question 12

Correct

Mark 1.00 out of 1.00

PLATING ON GENETIC DISORDERS (RRA)

Which of the following findings in Klinefelter syndrome is considered as the most consistent?

a. Eunuchoid body habitus


b. Lower IQ points
c. Osteoporosis
d. Hypogonadism 

The correct answer is: Hypogonadism

Question 13
Incorrect

Mark 0.00 out of 1.00

PLATING ON GENETIC DISORDERS (RGB)

A 25 year old male sought consult because of painful muscle cramping after playing basketball. He also note
darkening of his urine. Which of the following is the most probable diagnosis?

a. Von Gierke disease


b. McArdle disease
c. Pompe disease 
d. Gaucher disease

The correct answer is: McArdle disease

Question 14
Correct

Mark 1.00 out of 1.00

PLATING ON GENETIC DISORDERS (RGB)

Patients who are homozygous for Familial hypercholesterolemia usually develop which of the following significant
disease?

a. Premature atherosclerosis and Myocardial infarction 


b. Premature atherosclerosis and ruptured aortic aneurysm
c. Cerebrovascular hemorrhage
d. Congestive heart failure

The correct answer is: Premature atherosclerosis and Myocardial infarction

https://moodle.feu-nrmf.ph/mod/quiz/review.php?attempt=1107739&cmid=88124 5/9
1/4/22, 11:50 AM PLATING ON GENETIC DISORDERS: Attempt review

Question 15

Incorrect

Mark 0.00 out of 1.00

PLATING ON GENETIC DISORDERS (RRA)

Which of the following characteristics would make one classified as a “male pseudohermaphrodite”?

a. Gonads contain both ovarian and testicular tissue histologically


b. Patient has female external genitalia but have histologically testicular tissue
c. Patient has male external genitalia but have histologically ovarian tissue 
d. None of the given choices

The correct answer is: Patient has female external genitalia but have histologically testicular tissue

Question 16
Correct

Mark 1.00 out of 1.00

PLATING ON GENETIC DISORDERS (RRA)

As a rotator in pediatrics, you were assigned to the catcher area in the labor and delivery area to assess newborn
babies. Your resident received a critical baby and on your initial examination revealed microcephaly,
microphthalmia, polydactyly, cleft lip and palate, an umbilical hernia and rocker-bottom feet. Which of the
following disease does the baby likely have?

a. Patau Syndrome 
b. Edward Syndrome
c. Turner Syndrome
d. Down Syndrome

The correct answer is: Patau Syndrome

Question 17
Correct

Mark 1.00 out of 1.00

PLATING ON GENETIC DISORDERS (RGB)

Which of the following features is not true of Complex multigenic disorders?

a. Environmental factors plays a role


b. Associated with mutation in multiple genes of small effect
c. Different gene polymorphisms vary in significance
d. One is destined to develop the disease 

The correct answer is: One is destined to develop the disease

https://moodle.feu-nrmf.ph/mod/quiz/review.php?attempt=1107739&cmid=88124 6/9
1/4/22, 11:50 AM PLATING ON GENETIC DISORDERS: Attempt review

Question 18

Correct

Mark 1.00 out of 1.00

PLATING ON GENETIC DISORDERS (RRA)

An 8-year-old boy that was born at home was being evaluated in the developmental pediatrician’s clinic due to
possible developmental delays. The boy was able to perform activities of daily living including feeding and
dressing himself but cannot cope with schoolwork. Physical examination shows that he is brachycephalic with
oblique palpebral fissures and prominent epicanthal folds. There is also a transverse crease on the palm of his
hand. Which of the following diseases may the patient also have?

a. Amyotrophic lateral sclerosis


b. Congenital heart disease 
c. Diseases related to atopy
d. Chronic leukemia

The correct answer is: Congenital heart disease

Question 19
Correct

Mark 1.00 out of 1.00

PLATING ON GENETIC DISORDERS (RRA)

CASE WITH IMAGE. A 15-year-old male was seen in the ophthalmologist’s clinic due to failing eyesight and
progressive muscle weakness for the past 3 years. Family history revealed that many in the family have the same
condition. The medical student decided to make a pedigree of the patient’s family and marking those with the
same condition (see image below). Based on this pedigree, what type of genetic disease does the patient (and
his family members) most likely have?

a. Trinucleotide repeat expansion


b. Mitochondrial mutation 
c. X-linked inheritance pattern
d. Genomic imprinting

The correct answer is: Mitochondrial mutation

Question 20
Correct

Mark 1.00 out of 1.00

PLATING ON GENETIC DISORDERS (RGB)

Mutations which lead to formation of a stop codon is known as which of the following types of mutation?

a. Nonsense 
b. Conservative missense
c. Frameshift
d. Nonconservative missense

The correct answer is: Nonsense

https://moodle.feu-nrmf.ph/mod/quiz/review.php?attempt=1107739&cmid=88124 7/9
1/4/22, 11:50 AM PLATING ON GENETIC DISORDERS: Attempt review

Previous activity
◄ PLATING ON HEMODYNAMIC DISORDERS, THROMBOEMBOLIC DISEASES AND SHOCK

Jump to...

Next activity
PLATING ON DISEASES OF THE IMMUNE SYSTEM PART 1 ►

Stay in touch

http://www.feu-nrmf.ph

+632 8983 8338 / +639176272623

https://moodle.feu-nrmf.ph/mod/quiz/review.php?attempt=1107739&cmid=88124 8/9
1/4/22, 11:50 AM PLATING ON GENETIC DISORDERS: Attempt review


info@feu-nrmf.ph

 Data retention summary

 Get the mobile app

https://moodle.feu-nrmf.ph/mod/quiz/review.php?attempt=1107739&cmid=88124 9/9
1/4/22, 11:49 AM PLATING ON HEMODYNAMIC DISORDERS, THROMBOEMBOLIC DISEASES AND SHOCK: Attempt review

Dashboard My courses Patho 2A 1st Sem SY 2021 MD2-A,B,C,D,E,F,G,H,I,J PLATING EXAMS

PLATING ON HEMODYNAMIC DISORDERS, THROMBOEMBOLIC DISEASES AND SHOCK

Started on Friday, 3 September 2021, 7:01 PM


State Finished
Completed on Friday, 3 September 2021, 7:25 PM
Time taken 24 mins 2 secs
Marks 17.00/20.00
Grade 85.00 out of 100.00

Question 1
Correct

Mark 1.00 out of 1.00

PLATING ON HEMODYNAMIC DISORDERS, THROMBOEMBOLIC DISEASES AND SHOCK

White (anemic) infarcts are seen in:

a. organs with dual circulation


b. organs with end-arterial circulation 
c. previously congested tissues
d. loose tissues

The correct answer is: organs with end-arterial circulation

Question 2
Correct

Mark 1.00 out of 1.00

PLATING ON HEMODYNAMIC DISORDERS, THROMBOEMBOLIC DISEASES AND SHOCK

Hypercoagulability due to substitution of glutamine for the normal arginine residue rendering the protein resistant
to cleavage by protein C

a. Inherited deficiency of protein S


b. Heparin induced thrombocytopenia
c. Inherited deficiency of cystathione-beta-synthetase
d. Leiden mutation 

The correct answer is: Leiden mutation

https://moodle.feu-nrmf.ph/mod/quiz/review.php?attempt=1063809&cmid=86090 1/8
1/4/22, 11:49 AM PLATING ON HEMODYNAMIC DISORDERS, THROMBOEMBOLIC DISEASES AND SHOCK: Attempt review

Question 3

Correct

Mark 1.00 out of 1.00

PLATING ON HEMODYNAMIC DISORDERS, THROMBOEMBOLIC DISEASES AND SHOCK

Single most important factor for thrombus formation

a. Thrombophilia
b. Stasis of blood flow
c. Turbulet blood flow
d. Endothelial injury 

The correct answer is: Endothelial injury

Question 4
Incorrect

Mark 0.00 out of 1.00

PLATING ON HEMODYNAMIC DISORDERS, THROMBOEMBOLIC DISEASES AND SHOCK

A test which assesses the function of platelets

a. Bleeding Time
b. D-dimer
c. Prothombin Time
d. Clotting Time 

The correct answer is: Bleeding Time

Question 5
Correct

Mark 1.00 out of 1.00

PLATING ON HEMODYNAMIC DISORDERS, THROMBOEMBOLIC DISEASES AND SHOCK

Deficiency of this factor is incompatible with life

a. Prothrombin 
b. Proaccelerin
c. Hegman factor
d. Christmas factor

The correct answer is: Prothrombin

https://moodle.feu-nrmf.ph/mod/quiz/review.php?attempt=1063809&cmid=86090 2/8
1/4/22, 11:49 AM PLATING ON HEMODYNAMIC DISORDERS, THROMBOEMBOLIC DISEASES AND SHOCK: Attempt review

Question 6

Correct

Mark 1.00 out of 1.00

PLATING ON HEMODYNAMIC DISORDERS, THROMBOEMBOLIC DISEASES AND SHOCK

A process that results from reduced venous outflow of blood from a tissue

a. Hemorrhage
b. Thrombosis
c. Hyperemia
d. Congestion 

The correct answer is: Congestion

Question 7
Correct

Mark 1.00 out of 1.00

PLATING ON HEMODYNAMIC DISORDERS, THROMBOEMBOLIC DISEASES AND SHOCK

Type of necrosis seen in infarcted myocardium

a. Liquefaction
b. Caseation
c. Coagulation 
d. Gangrenous

The correct answer is: Coagulation

Question 8
Incorrect

Mark 0.00 out of 1.00

PLATING ON HEMODYNAMIC DISORDERS, THROMBOEMBOLIC DISEASES AND SHOCK

Edema in malnutrition is caused by:

a. reduced plasma oncotic pressure


b. increased hydrostatic pressure
c. failure of the lymphatic system
d. sodium retention 

The correct answer is: reduced plasma oncotic pressure

https://moodle.feu-nrmf.ph/mod/quiz/review.php?attempt=1063809&cmid=86090 3/8
1/4/22, 11:49 AM PLATING ON HEMODYNAMIC DISORDERS, THROMBOEMBOLIC DISEASES AND SHOCK: Attempt review

Question 9

Correct

Mark 1.00 out of 1.00

PLATING ON HEMODYNAMIC DISORDERS, THROMBOEMBOLIC DISEASES AND SHOCK

Nutmeg appearance is the classic gross description:

a. Chronic pulmonary congestion


b. Acute pulmonary congestion
c. Chronic passive hepatic congestion 
d. Acute hepatic congestion

The correct answer is: Chronic passive hepatic congestion

Question 10
Incorrect

Mark 0.00 out of 1.00

PLATING ON HEMODYNAMIC DISORDERS, THROMBOEMBOLIC DISEASES AND SHOCK

This is characterized by pulmonary insufficiency, neurologic symptoms, anemia and thrombocytopenia with or
without rash

a. Air embolism
b. Fat embolism
c. Amniotic fluid embolism
d. Systemic thromboembolism 

The correct answer is: Fat embolism

Question 11
Correct

Mark 1.00 out of 1.00

PLATING ON HEMODYNAMIC DISORDERS, THROMBOEMBOLIC DISEASES AND SHOCK A defect of primary hemostasis

a. von Willebrand disease 


b. Anti-phospholipid antibody syndrome
c. Disseminated intravascular coagulation
d. Factor V Leiden disease

The correct answer is: von Willebrand disease

https://moodle.feu-nrmf.ph/mod/quiz/review.php?attempt=1063809&cmid=86090 4/8
1/4/22, 11:51 AM PLATING ON NEOPLASIA 1: Attempt review

Question 3

Incorrect

Mark 0.00 out of 1.00

PLATING ON NEOPLASIA 1

When the cancer cells infiltrate and destroy the basement membrane, the feature being displayed is:

a. differentiation
b. metastasis 
c. anaplasia
d. invasion

The correct answer is: invasion

Question 4
Correct

Mark 1.00 out of 1.00

PLATING ON NEOPLASIA 1

Dermoid cyst of the ovary is an example of:

a. choristoma
b. adenoma
c. hamartoma
d. benign mixed tumor
e. teratoma 

The correct answer is: teratoma

Question 5
Incorrect

Mark 0.00 out of 1.00

PLATING ON NEOPLASIA 1

Which of the following is a benign neoplasm?

a. ovarian mature cystic teratoma


b. GIT extranodal lymphoma 
c. testicular seminoma
d. liver hepatoma

The correct answer is: ovarian mature cystic teratoma

https://moodle.feu-nrmf.ph/mod/quiz/review.php?attempt=1151162&cmid=91552 2/8
1/4/22, 11:51 AM PLATING ON NEOPLASIA 1: Attempt review

Question 6

Correct

Mark 1.00 out of 1.00

PLATING ON NEOPLASIA 1

Based on the 2019 estimated cancer data, which of the following is incorrect?

a. prostate cancer is the most common cancer in adult males


b. Breast cancer is the most common cancer in adult females
c. lung cancer is the common cause of death in adults for both sexes
d. lung cancer is the second most common cancer in adult male only 

The correct answer is: lung cancer is the second most common cancer in adult male only

Question 7
Correct

Mark 1.00 out of 1.00

PLATING ON NEOPLASIA 1

A 56 y/o Caucasian who is fond of sun bathing was diagnosed of melanoma. The implicated agent is:

a. UVB 
b. UVA
c. Ionizing electromagnetic forces
d. Alpha & beta particles

The correct answer is: UVB

Question 8
Incorrect

Mark 0.00 out of 1.00

PLATING ON NEOPLASIA 1

Of the following features of benign neoplasm, which is the most important in differentiating with malignant
counterpart?

a. amenable to surgical removal


b. cannot spread to other sites
c. relatively innocent
d. remains localized 

The correct answer is: cannot spread to other sites

https://moodle.feu-nrmf.ph/mod/quiz/review.php?attempt=1151162&cmid=91552 3/8
1/4/22, 11:51 AM PLATING ON NEOPLASIA 1: Attempt review

Question 9

Correct

Mark 1.00 out of 1.00

PLATING ON NEOPLASIA 1

Nearly 80% of the total decrease death rate among men is due to the following EXCEPT:

a. colorectal CA
b. pancreas 
c. lung
d. prostate

The correct answer is: pancreas

Question 10
Correct

Mark 1.00 out of 1.00

PLATING ON NEOPLASIA 1

The single most important environmental factor contributing to premature death in the US is:

a. diet
b. infectious agents
c. alcohol
d. cigarette smoking 

The correct answer is: cigarette smoking

Question 11
Correct

Mark 1.00 out of 1.00

PLATING ON NEOPLASIA 1

The growth and spread of a tumor is dependent or determined by its:

a. capsule
b. parenchyma
c. none of the choices
d. stroma 

The correct answer is: stroma

https://moodle.feu-nrmf.ph/mod/quiz/review.php?attempt=1151162&cmid=91552 4/8
1/4/22, 11:51 AM PLATING ON NEOPLASIA 1: Attempt review

Question 12

Correct

Mark 1.00 out of 1.00

PLATING ON NEOPLASIA 1

A 31 y/o female diagnosed of invasive ductal carcinoma presented with headache associated with change in
sensorium. Cranial CT-scan with contrast revealed multiple enhancing lesion both in the meninges and brain
parenchyma. The pathway by which the cancer cells spread and reached the brain is by:

a. seeding
b. hematogenous 
c. lymphatics
d. none of the choices

The correct answer is: hematogenous

Question 13
Incorrect

Mark 0.00 out of 1.00

PLATING ON NEOPLASIA 1

This is a benign epithelial tumor:

a. liver adenoma
b. Osteoid osteoma
c. GIT lipoma
d. Ovarian fibroma 

The correct answer is: liver adenoma

Question 14
Correct

Mark 1.00 out of 1.00

PLATING ON NEOPLASIA 1

Included in the small round blue cell tumors in infants and childhood EXCEPT:

a. Carcinoid tumor 
b. Wilms tumor
c. Acute leukemias
d. rhabdomyosarcomas

The correct answer is: Carcinoid tumor

https://moodle.feu-nrmf.ph/mod/quiz/review.php?attempt=1151162&cmid=91552 5/8
1/4/22, 11:49 AM PLATING ON INFLAMMATION AND REPAIR: Attempt review

Dashboard My courses Patho 2A 1st Sem SY 2021 MD2-A,B,C,D,E,F,G,H,I,J PLATING EXAMS

PLATING ON INFLAMMATION AND REPAIR

Started on Monday, 23 August 2021, 12:01 PM


State Finished
Completed on Monday, 23 August 2021, 12:26 PM
Time taken 24 mins 39 secs
Marks 14.00/20.00
Grade 70.00 out of 100.00

Question 1
Incorrect

Mark 0.00 out of 1.00

PLATING ON INFLAMMATION AND REPAIR

A non-absorbable suture was left in the skin for more than a year. Regular wound cleaning was done when the
wound was still fresh and open hence it was not infected with any microbial organism. With the pattern of
inflammation in the tissue with the suture, what morphologic change is compatible?

a. Excavation of the skin with inflammatory surface and underlying granulation tissue as wells as scar
b. Presence of epithelioid cells, giant cells and lymphocytes
c. Edema fluid in the interstitium 
d. Abundant neutrophils and cell debris wit serous exudate

The correct answer is: Presence of epithelioid cells, giant cells and lymphocytes

Question 2
Incorrect

Mark 0.00 out of 1.00

PLATING ON INFLAMMATION AND REPAIR

In patients diagnosed to have Chronic Granulomatous Disease where there is absence of an enzyme needed to
produce superoxide anion in order to kill organisms engulfed by neutrophils, what antimicrobial chemical will not
be produced that renders the bactericidal system of neutrophils less potent?

a. Hypochlorite
b. Myeloperoxidase
c. Inducible NOS
d. Peroxynitrite 

The correct answer is: Hypochlorite

https://moodle.feu-nrmf.ph/mod/quiz/review.php?attempt=1015739&cmid=84283 1/9
1/4/22, 11:49 AM PLATING ON INFLAMMATION AND REPAIR: Attempt review

Question 3

Correct

Mark 1.00 out of 1.00

PLATING ON INFLAMMATION AND REPAIR

A 60 year old male was admitted due to pneumonia complicated by abscess formation. He also developed
pleural effusion which resulted to fibrin deposition on the pleural surface and inflammation. Which pattern of
inflammation is not present in the case?

a. Fibrinous
b. Granulomatous 
c. Purulent
d. Serous

The correct answer is: Granulomatous

Question 4
Correct

Mark 1.00 out of 1.00

PLATING ON INFLAMMATION AND REPAIR

How do leukocytes cause injury to normal tissue?

a. When inflammatory response is directed against a normal tissue


b. When the inflammatory response is against a harmless exogenous substance in the body
c. All of the given options 
d. Prolonged inflammatory response causing collateral damage to tissue

The correct answer is: All of the given options

Question 5
Correct

Mark 1.00 out of 1.00

PLATING ON INFLAMMATION AND REPAIR

The following are the roles of T helper lymphocytes in inflammatory processes, except:

a. Production of immunoglobulins in chronic inflammation 


b. Secretion of cytokines to promote inflammation
c. IFN gamma secretion by Th1 to activate M1 macrophages
d. Cytokine secretion by Th2 for recruitment and activation of eosinophils

The correct answer is: Production of immunoglobulins in chronic inflammation

https://moodle.feu-nrmf.ph/mod/quiz/review.php?attempt=1015739&cmid=84283 2/9
1/4/22, 11:49 AM PLATING ON INFLAMMATION AND REPAIR: Attempt review

Question 6

Incorrect

Mark 0.00 out of 1.00

PLATING ON INFLAMMATION AND REPAIR

A child with recurrent bacterial infections was diagnosed to have Leukocyte Adhesion Deficiency Type 2 due to
absence of sialyl-Lewis X, what process in migration of leukocytes is defective?

a. Margination
b. Rolling
c. Firm adhesion 
d. Transmigration

The correct answer is: Rolling

Question 7
Correct

Mark 1.00 out of 1.00

PLATING ON INFLAMMATION AND REPAIR 

The skin of the right middle finger of a cook was accidentally cut an hour ago with the knife that she is using to
slice vegetables. The knife has bacterial organisms that were introduced into the skin wound. The skin wound is no
longer bleeding and appears to be red and slightly swollen. Which of the following statements is not true about
the cook’s present wound?  

a. Its local signs, such as redness, are prominent


b. Fibrosis associated with the damage is not yet present
c. The inflammation is expected to be confined to the site of damage and infection
d. The cellular infiltrates are predominantly lymphocytes 

The correct answer is: The cellular infiltrates are predominantly lymphocytes

Question 8
Correct

Mark 1.00 out of 1.00

PLATING ON INFLAMMATION AND REPAIR

Repair by regeneration is observed in the following:

a. neural tissue
b. skin 
c. myocardium
d. Any of the other choices

The correct answer is: skin

https://moodle.feu-nrmf.ph/mod/quiz/review.php?attempt=1015739&cmid=84283 3/9
1/4/22, 11:49 AM PLATING ON INFLAMMATION AND REPAIR: Attempt review

Question 9

Correct

Mark 1.00 out of 1.00

PLATING ON INFLAMMATION AND REPAIR

In an inflammed tissue with increased vascular permeability secondary to the most common mechanism of
vascular leakage, the following statements apply, except:

a. Contraction of endothelial cells


b. It can be elicited by histamine and leukotrienes
c. Leukocytes migrate through the endothelium and wall of the venules
d. Vascular leakage lasts for days 

The correct answer is: Vascular leakage lasts for days

Question 10
Correct

Mark 1.00 out of 1.00

PLATING ON INFLAMMATION AND REPAIR

What is the principal mediator of immediate transient response of increase in vascular permeability?

a. Serotonin
b. Leukotriene
c. Histamine 
d. Thromboxane

The correct answer is: Histamine

Question 11
Correct

Mark 1.00 out of 1.00

PLATING ON INFLAMMATION AND REPAIR

Which of the following complement fragments can directly promote vasodilation and increase in vascular
permeability?

a. C5a
b. C3a
c. All of the given options 
d. C4a

The correct answer is: All of the given options

https://moodle.feu-nrmf.ph/mod/quiz/review.php?attempt=1015739&cmid=84283 4/9
1/4/22, 11:49 AM PLATING ON INFLAMMATION AND REPAIR: Attempt review

Question 12

Correct

Mark 1.00 out of 1.00

PLATING ON INFLAMMATION AND REPAIR

Bacterial organisms were able to enter the subcutaneous tissue and IgG antibodies immediately coated all of the
available antigens. How will the surrounding leukocytes phagocytose the bacterial organisms?

a. Using Toll-like receptors


b. Using receptors for the Fc portion of IgG 
c. By activating inflammasome of the leukocytes
d. Using their NOD-like receptors

The correct answer is: Using receptors for the Fc portion of IgG

Question 13
Correct

Mark 1.00 out of 1.00

PLATING ON INFLAMMATION AND REPAIR

The is/are role/s of macrophages in wound healing:

a. All of the other choices 


b. Providing growth factors
c. Cytokines secretion
d. Degrading offending agents

The correct answer is: All of the other choices

Question 14
Correct

Mark 1.00 out of 1.00

PLATING ON INFLAMMATION AND REPAIR

Which of the following is expected to induce an inflammatory reaction?

a. Necrotic cells
b. All of the given choices 
c. Immune complexes in vascular walls
d. Uric acid crystals in the joints

The correct answer is: All of the given choices

https://moodle.feu-nrmf.ph/mod/quiz/review.php?attempt=1015739&cmid=84283 5/9
1/4/22, 11:49 AM PLATING ON INFLAMMATION AND REPAIR: Attempt review

Question 15

Correct

Mark 1.00 out of 1.00

PLATING ON INFLAMMATION AND REPAIR

A 45 year old has cough and colds and has palpable cervical lymph nodes associated with the infection. What do
you call this condition?

a. Edema
b. Lymphadenitis 
c. Lymphangitis
d. Lymphoma

The correct answer is: Lymphadenitis

Question 16
Correct

Mark 1.00 out of 1.00

PLATING ON INFLAMMATION AND REPAIR

The following statements are true about fluid accumulation in the thoracic cavity of a patient with pulmonary
tuberculosis:

a. The fluid has high protein and has cell debris


b. The fluid is an ultrafiltrate of plasma due to high hydrostatic pressure
c. It is an exudative fluid 
d. The fluid accumulation is associated with increase in vascular permeability

The correct answers are: It is an exudative fluid, The fluid has high protein and has cell debris, The fluid
accumulation is associated with increase in vascular permeability

Question 17
Incorrect

Mark 0.00 out of 1.00

PLATING ON INFLAMMATION AND REPAIR

A 30 year old male patient underwent excision of a poypoid new growth on his chest. Several weeks after the
procedure, he noticed an enlarging darker brown elevated lesion on the incision site which eventually grew
beyond the incision borders. Which of the following statements is TRUE? The microscopic findings show

a. predominant epithelioid cells with lymphocytic infiltrates


b. numerous vascular channels admixed with fibroblasts 
c. dermal thick connective tissue deposition
d. exuberant neutrophilic infiltration

The correct answer is: dermal thick connective tissue deposition

https://moodle.feu-nrmf.ph/mod/quiz/review.php?attempt=1015739&cmid=84283 6/9
1/4/22, 11:49 AM PLATING ON INFLAMMATION AND REPAIR: Attempt review

Question 18

Correct

Mark 1.00 out of 1.00

PLATING ON INFLAMMATION AND REPAIR

A medical technologist received a specimen labeled as skin wound discharge. It is tan-white to yellow, thick fluid.
On microscopy, it is composed of abundant neutrophils with some gram positive cocci in clusters surrounded by
cell debris. What is the pattern of inflammation?

a. Suppurative 
b. Granulomatous
c. Fibrinous
d. Serous

The correct answer is: Suppurative

Question 19
Incorrect

Mark 0.00 out of 1.00

PLATING ON INFLAMMATION AND REPAIR

When a cell is damaged and it releases ATP molecules from the mitochondria into the cytosol, what receptor will
then be activated to promote recruitment of leukocytes and thus induce inflammation?

a. Lectin that can bind to fucose


b. NLRs
c. TLRs 
d. Inflammasome

The correct answer is: NLRs

Question 20
Incorrect

Mark 0.00 out of 1.00

PLATING ON INFLAMMATION AND REPAIR

The following are prominent in second intention healing compared to primary intention healing EXCEPT:

a. Wound contraction
b. Focal disruption of epithelial basement membrane
c. Granulation tissue formation
d. None of the other choices 

The correct answer is: Focal disruption of epithelial basement membrane

Previous activity

◄ PLATING ON CELL INJURY, CELL DEATH AND ADAPTATIONS

Jump to...

Next activity
PLATING ON HEMODYNAMIC DISORDERS, THROMBOEMBOLIC DISEASES AND SHOCK ►

https://moodle.feu-nrmf.ph/mod/quiz/review.php?attempt=1015739&cmid=84283 7/9
1/4/22, 11:49 AM PLATING ON INFLAMMATION AND REPAIR: Attempt review

Stay in touch

http://www.feu-nrmf.ph

+632 8983 8338 / +639176272623

https://moodle.feu-nrmf.ph/mod/quiz/review.php?attempt=1015739&cmid=84283 8/9
1/4/22, 11:49 AM PLATING ON INFLAMMATION AND REPAIR: Attempt review


info@feu-nrmf.ph

 Data retention summary

 Get the mobile app

https://moodle.feu-nrmf.ph/mod/quiz/review.php?attempt=1015739&cmid=84283 9/9
1/4/22, 11:48 AM PLATING ON CELL INJURY, CELL DEATH AND ADAPTATIONS: Attempt review

Dashboard My courses Patho 2A 1st Sem SY 2021 MD2-A,B,C,D,E,F,G,H,I,J PLATING EXAMS

PLATING ON CELL INJURY, CELL DEATH AND ADAPTATIONS

Started on Monday, 16 August 2021, 12:00 PM


State Finished
Completed on Monday, 16 August 2021, 12:26 PM
Time taken 26 mins 14 secs
Marks 14.00/20.00
Grade 70.00 out of 100.00

Question 1
Correct

Mark 1.00 out of 1.00

PLATING ON CELL INJURY, CELL DEATH AND ADAPTATIONS 

Accumulation of free radicals in cell injury is countered by which of the following mechanisms?

a. inactivation of free radicals 


b. reduction-oxidation reaction
c. leukocytes degradation during inflammation
d. absorption of radiant energies

The correct answer is: inactivation of free radicals

Question 2
Correct

Mark 1.00 out of 1.00

PLATING ON CELL INJURY, CELL DEATH AND ADAPTATIONS 

Which is TRUE of Russel bodies?

a. a protein cellular accumulation 


b. may be stained with PAS
c. seen in Neimann Pick
d. microscopically appear as clear vacuoles

The correct answer is: a protein cellular accumulation

https://moodle.feu-nrmf.ph/mod/quiz/review.php?attempt=997025&cmid=82898 1/8
1/4/22, 11:48 AM PLATING ON CELL INJURY, CELL DEATH AND ADAPTATIONS: Attempt review

Question 3

Correct

Mark 1.00 out of 1.00

PLATING ON CELL INJURY, CELL DEATH AND ADAPTATIONS 

SARS-CoV2 infection will result into COVID-19 where patients would have fever and couch progressing into
difficulty of breathing. There will be Infiltration of lung tissue by mononuclear inflammatory cells, with
desquamation of alveolar epithelium and formation of hyaline membrane. These changes will result into a heavy
and firm lungs observed in patients upon autopsy. Which of the following features of COVID-19 pertains to
morphological change?

a. hyaline membrane formation 


b. positive RT-PCR test for SARS-CoV2
c. cough and dyspnea
d. difficulty of breathing

The correct answer is: hyaline membrane formation

Question 4

Correct

Mark 1.00 out of 1.00

PLATING ON CELL INJURY, CELL DEATH AND ADAPTATIONS 

Which of the following statements is consistent with hypertrophy?

a. mechanical stretch on the cell surface may initiate the process 


b. there is cell proliferation
c. may be a nidus of malignant transformation
d. it is always pathologic

The correct answer is: mechanical stretch on the cell surface may initiate the process

Question 5
Correct

Mark 1.00 out of 1.00

PLATING ON CELL INJURY, CELL DEATH AND ADAPTATIONS 

Ischemic injury to the brain results into _______________________

a. cardiomegaly
b. liquefactive necrosis 
c. infarct
d. coagulative necrosis

The correct answer is: liquefactive necrosis

https://moodle.feu-nrmf.ph/mod/quiz/review.php?attempt=997025&cmid=82898 2/8
1/4/22, 11:48 AM PLATING ON CELL INJURY, CELL DEATH AND ADAPTATIONS: Attempt review

Question 6

Correct

Mark 1.00 out of 1.00

PLATING ON CELL INJURY, CELL DEATH AND ADAPTATIONS 

In irreversible cell injury, which of the following is first to be observed?

a. electron microscopic changes


b. elevation of intracellular enzymes 
c. gross morphologic changes
d. changes seen in light microscope

The correct answer is: elevation of intracellular enzymes

Question 7
Correct

Mark 1.00 out of 1.00

PLATING ON CELL INJURY, CELL DEATH AND ADAPTATIONS 

Sections of renal infarct will show which of the following?

a. ghost cells
b. accumulation of pus
c. intact architecture of the area of necrosis 
d. secondary bacterial infection

The correct answer is: intact architecture of the area of necrosis

Question 8
Correct

Mark 1.00 out of 1.00

PLATING ON CELL INJURY, CELL DEATH AND ADAPTATIONS 

Which best explains why apoptosis occurs in a setting of necrosis?

a. activated leukocytes from inflammation release mediators that activate caspases


b. increase in intracellular calcium directly induce caspase 
c. intracellular calcium are sequestered in the endoplasmic reticulum inducing apoptosis
d. free radicals directly enhance apoptotic activity

The correct answer is: increase in intracellular calcium directly induce caspase

https://moodle.feu-nrmf.ph/mod/quiz/review.php?attempt=997025&cmid=82898 3/8
1/4/22, 11:48 AM PLATING ON CELL INJURY, CELL DEATH AND ADAPTATIONS: Attempt review

Question 9

Correct

Mark 1.00 out of 1.00

PLATING ON CELL INJURY, CELL DEATH AND ADAPTATIONS 

Which of these is consistent with pyroptosis?

a. there is excessive intracellular iron levels


b. iron chelation may reduce it
c. fever 
d. absolutely no inflammatory reaction

The correct answer is: fever

Question 10
Incorrect

Mark 0.00 out of 1.00

PLATING ON CELL INJURY, CELL DEATH AND ADAPTATIONS 

What is the adaptive change observed in Barett esophagus?

a. metaplasia
b. atrophy
c. hypertrophy
d. hyperplasia 

The correct answer is: metaplasia

Question 11
Incorrect

Mark 0.00 out of 1.00

PLATING ON CELL INJURY, CELL DEATH AND ADAPTATIONS 

Hypoxic injury eventually results into which of these?

a. increase in intracellular pH 
b. increase ATP synthesis
c. increase in lactic acid
d. enhanced aerobic glycolysis

The correct answer is: increase in lactic acid

https://moodle.feu-nrmf.ph/mod/quiz/review.php?attempt=997025&cmid=82898 4/8
1/4/22, 11:48 AM PLATING ON CELL INJURY, CELL DEATH AND ADAPTATIONS: Attempt review

Question 12

Correct

Mark 1.00 out of 1.00

PLATING ON CELL INJURY, CELL DEATH AND ADAPTATIONS 

How does the decrease ATP in injury lead to phospholipid loss and eventual membrane damage?

a. protease activation that leads to cytoskeletal damage


b. decrease ATP leads to mitochondrial dysfunction which in turn decrease the phospholipid synthesis 
c. p53 activation
d. enhanced phospholipase degradation

The correct answer is: decrease ATP leads to mitochondrial dysfunction which in turn decrease the phospholipid
synthesis

Question 13
Correct

Mark 1.00 out of 1.00

PLATING ON CELL INJURY, CELL DEATH AND ADAPTATIONS 

Exogenous carbon pigments that accumulate in the lungs associated with fibrosis.

a. pneumoconiosis 
b. anthracosis
c. pulmonary fibrosis
d. cystic fibrosis

The correct answer is: pneumoconiosis

Question 14
Correct

Mark 1.00 out of 1.00

PLATING ON CELL INJURY, CELL DEATH AND ADAPTATIONS 

Which type of necrosis is likely to be seen in a patient with an autoimmune disease systemic lupus
erythematosus?

a. coagulative necrosis
b. gangrenous necrosis
c. fat necrosis
d. fibrinoid necrosis 

The correct answer is: fibrinoid necrosis

https://moodle.feu-nrmf.ph/mod/quiz/review.php?attempt=997025&cmid=82898 5/8
1/4/22, 11:48 AM PLATING ON CELL INJURY, CELL DEATH AND ADAPTATIONS: Attempt review

Question 15

Correct

Mark 1.00 out of 1.00

PLATING ON CELL INJURY, CELL DEATH AND ADAPTATIONS 

Restoration of blood flow in a setting of ischemic tissue exacerbates and accelerate tissue injury in which of the
following mechanism?

a. suppression of sodium-potassium pump


b. calcium depletion
c. reperfusion counters the effects of inflammation
d. reperfused blood carries with it reactive oxygen species 

The correct answer is: reperfused blood carries with it reactive oxygen species

Question 16
Incorrect

Mark 0.00 out of 1.00

PLATING ON CELL INJURY, CELL DEATH AND ADAPTATIONS 

Reversible cell injury in the liver is most likely to manifest with which of the following?

a. cloudy swelling 
b. hydrophic change
c. fatty change
d. cellular swelling

The correct answer is: fatty change

Question 17
Correct

Mark 1.00 out of 1.00

PLATING ON CELL INJURY, CELL DEATH AND ADAPTATIONS 

Which pathway is associated with atrophy?

a. G-protein coupled receptors


b. PI3/AKT pathway
c. calcineurin-NFAT pathway
d. ubiquitin-proteosome pathway 

The correct answer is: ubiquitin-proteosome pathway

https://moodle.feu-nrmf.ph/mod/quiz/review.php?attempt=997025&cmid=82898 6/8
1/4/22, 11:48 AM PLATING ON CELL INJURY, CELL DEATH AND ADAPTATIONS: Attempt review

Question 18

Incorrect

Mark 0.00 out of 1.00

PLATING ON CELL INJURY, CELL DEATH AND ADAPTATIONS 

Replicative senescence that contributes to cellular aging is attributed to which of the following?

a. telomere shortening
b. activation of sirtuins 
c. DNA damage
d. free radicals

The correct answer is: telomere shortening

Question 19
Incorrect

Mark 0.00 out of 1.00

PLATING ON CELL INJURY, CELL DEATH AND ADAPTATIONS 

The release of cellular contents in necrosis leads to _____________________

a. apoptosis
b. cellular accumulation 
c. autophagy
d. inflammation

The correct answer is: inflammation

Question 20
Incorrect

Mark 0.00 out of 1.00

PLATING ON CELL INJURY, CELL DEATH AND ADAPTATIONS 

Dystrophic calcification suggests _________________________

a. bone malignancy
b. area of tissue necrosis
c. there is hypercalcemia 
d. evolving renal failure

The correct answer is: area of tissue necrosis

https://moodle.feu-nrmf.ph/mod/quiz/review.php?attempt=997025&cmid=82898 7/8
1/4/22, 11:48 AM PLATING ON CELL INJURY, CELL DEATH AND ADAPTATIONS: Attempt review

Previous activity
◄ PLATING ON CELL AS A UNIT OF HEALTH AND DISEASE

Jump to...

Next activity
PLATING ON INFLAMMATION AND REPAIR ►

Stay in touch

http://www.feu-nrmf.ph

+632 8983 8338 / +639176272623

info@feu-nrmf.ph

 Data retention summary

 Get the mobile app

https://moodle.feu-nrmf.ph/mod/quiz/review.php?attempt=997025&cmid=82898 8/8
1/4/22, 11:48 AM PLATING ON CELL AS A UNIT OF HEALTH AND DISEASE: Attempt review

Dashboard My courses Patho 2A 1st Sem SY 2021 MD2-A,B,C,D,E,F,G,H,I,J PLATING EXAMS

PLATING ON CELL AS A UNIT OF HEALTH AND DISEASE

Started on Monday, 9 August 2021, 12:03 PM


State Finished
Completed on Monday, 9 August 2021, 12:28 PM
Time taken 25 mins 18 secs

Question 1
Complete

Not graded

PLATING ON CELL AS A UNIT OF HEALTH AND DISEASE (MDV)

Approximately what percentage of the human genome codes for proteins?

a. 10%
b. 20%
c. 30%
d. 40%

The correct answer is: 30%

Question 2
Incorrect

Marked out of 1.00

PLATING ON CELL AS A UNIT OF HEALTH AND DISEASE (MDV)

Uptake of fluids or macromolecules is known as_____

a. Transcytosis
b. Endocytosis
c. Phagocytosis 
d. Exocytosis

The correct answer is: Endocytosis

Question 3
Incorrect

Marked out of 1.00

PLATING ON CELL AS A UNIT OF HEALTH AND DISEASE (MDV)

Transcriptionally active chromatin

a. Euchromatin
b. Heterochromatin 
c. Monochromatin
d. Homochromatin

The correct answer is: Euchromatin

https://moodle.feu-nrmf.ph/mod/quiz/review.php?attempt=981470&cmid=81370 1/8
1/4/22, 11:48 AM PLATING ON CELL AS A UNIT OF HEALTH AND DISEASE: Attempt review

Question 4

Incorrect

Marked out of 1.00

PLATING ON CELL AS A UNIT OF HEALTH AND DISEASE (ORB)

A 44-year-old male was diagnosed with aortic valve malformation. Which of the following substances might be
insufficient in this patient?

a. Elastin
b. Laminin 
c. Fibrillin
d. Fibronectin

The correct answers are: Fibrillin, Elastin

Question 5
Correct

Marked out of 1.00

PLATING ON CELL AS A UNIT OF HEALTH AND DISEASE (ORB)

Which of the following regulates collagen synthesis?

a. Vitamin A
b. Vitamin B
c. Vitamin D
d. Vitamin C 

The correct answer is: Vitamin C

Question 6
Incorrect

Marked out of 1.00

PLATING ON CELL AS A UNIT OF HEALTH AND DISEASE (MDV)

Characteristic of epigenetic alterations

a. Not amenable to treatment


b. Heritable up to a certain number of generations
c. Inherited up to the 2nd generation only
d. Irreversible 

The correct answer is: Heritable up to a certain number of generations

https://moodle.feu-nrmf.ph/mod/quiz/review.php?attempt=981470&cmid=81370 2/8
1/4/22, 11:48 AM PLATING ON CELL AS A UNIT OF HEALTH AND DISEASE: Attempt review

Question 7

Correct

Marked out of 1.00

PLATING ON CELL AS A UNIT OF HEALTH AND DISEASE (ORB)

Which collagen type is found in the basement membrane?

a. Type III
b. Type I
c. Type II
d. Type IV 

The correct answer is: Type IV

Question 8
Incorrect

Marked out of 1.00

PLATING ON CELL AS A UNIT OF HEALTH AND DISEASE (ORB)

Fibrillar collagens include the following, EXCEPT:

a. Type IV
b. Type I
c. Type II 
d. Type III

The correct answer is: Type IV

Question 9
Complete

Not graded

PLATING ON CELL AS A UNIT OF HEALTH AND DISEASE (MDV)

Characteristics of genetic variations EXCEPT

a. Includes polymorphisms
b. Located in non-protein coding regions
c. Located in protein coding regions
d. Inherited up to 4th generation only

The correct answer is: Located in protein coding regions

https://moodle.feu-nrmf.ph/mod/quiz/review.php?attempt=981470&cmid=81370 3/8
1/4/22, 11:48 AM PLATING ON CELL AS A UNIT OF HEALTH AND DISEASE: Attempt review

Question 10

Correct

Marked out of 1.00

PLATING ON CELL AS A UNIT OF HEALTH AND DISEASE (ORB)

Which of the following CDK inhibitors has selective effects on cyclin CDK4 and cyclin CDK6?

a. p16 
b. p57
c. p27
d. p21

The correct answer is: p16

Question 11
Correct

Marked out of 1.00

PLATING ON CELL AS A UNIT OF HEALTH AND DISEASE (MDV)

Which of the following are methods of epigenetic gene regulation?

a. DNA methylation
b. RNA acetylation
c. Histone phosphorylation
d. Histone acetylation 

The correct answers are: DNA methylation, Histone acetylation

Question 12
Incorrect

Marked out of 1.00

PLATING ON CELL AS A UNIT OF HEALTH AND DISEASE (ORB)

A 30-year-old male presented with a gash on his left forearm from a barb wire accident. After appropriate
treatment, the wound will show granulation tissue formation, fibroblastic proliferation, and angiogenesis. Which of
the following growth factors may be responsible for these processes?

a. FGF
b. EGF 
c. TGF-beta
d. TGF-alpha

The correct answer is: FGF

https://moodle.feu-nrmf.ph/mod/quiz/review.php?attempt=981470&cmid=81370 4/8
1/4/22, 11:48 AM PLATING ON CELL AS A UNIT OF HEALTH AND DISEASE: Attempt review

Question 13

Correct

Marked out of 1.00

PLATING ON CELL AS A UNIT OF HEALTH AND DISEASE (ORB)

This describes the differences between pluripotential, multipotential, or totipotential stem cells.

a. Pluripotential cells differentiate into components of the three germ cell layers.
b. Multipotential cells differentiate into components of the three germ cell layers.
c. Totipotential cells differentiate into components of the three germ cell layers. 
d. All options given are correct.

The correct answers are: Totipotential cells differentiate into components of the three germ cell layers.,
Pluripotential cells differentiate into components of the three germ cell layers.

Question 14
Incorrect

Marked out of 1.00

PLATING ON CELL AS A UNIT OF HEALTH AND DISEASE (ORB)

The p53 anti-oncogene nuclear protein product, which acts as a DNA replication regulator and prevents cells with
damaged DNA to proliferate, stops the cell cycle:

a. Between G1 and S
b. Between M and S
c. Between S and G2
d. Between G2 and M 

The correct answer is: Between G1 and S

Question 15
Incorrect

Marked out of 1.00

PLATING ON CELL AS A UNIT OF HEALTH AND DISEASE (ORB)

Microscopic findings of a section from a pale area taken from the anterior wall of the heart of an 85-year-old
female diagnosed with myocardial infarction shows myocytes replaced with diffuse red material, which stains
blue using trichrome stain. Which of the following describes its components?

a. It is secreted by monocytes and contains a core protein that is linked to mucopolysaccharides


b. It is secreted by endothelial cells and links macromolecules to integrins 
c. It is secreted by hepatocytes and is mainly responsible for intravascular oncotic pressure
d. It is secreted by fibroblasts and has a high content of glycine and hydroxyproline
e. It is secreted by plasma cells and is important in mediating humoral immunity

The correct answer is: It is secreted by fibroblasts and has a high content of glycine and hydroxyproline

https://moodle.feu-nrmf.ph/mod/quiz/review.php?attempt=981470&cmid=81370 5/8
1/4/22, 11:48 AM PLATING ON CELL AS A UNIT OF HEALTH AND DISEASE: Attempt review

Question 16

Incorrect

Marked out of 1.00

PLATING ON CELL AS A UNIT OF HEALTH AND DISEASE (ORB)

These are considered as totipotential stem cells.

a. Hematopoietic
b. Mesenchymal
c. Morula 
d. Blastocyst

The correct answer is: Blastocyst

Question 17
Incorrect

Marked out of 1.00

PLATING ON CELL AS A UNIT OF HEALTH AND DISEASE (MDV)

Correct pairing of function and cell organelle EXCEPT

a. Biosynthetic machineries: Endoplasmic reticulum


b. Waste disposal: Lysosomes 
c. Protection and nutrient acquisition: Golgi apparatus
d. Maintains polarity: Cytoskeleton

The correct answer is: Protection and nutrient acquisition: Golgi apparatus

Question 18
Incorrect

Marked out of 1.00

PLATING ON CELL AS A UNIT OF HEALTH AND DISEASE (MDV)

Type of cell-cell interaction also known as communicating junction

a. Desmosome 
b. Macula adherens
c. Gap junction
d. Tight junction

The correct answer is: Gap junction

https://moodle.feu-nrmf.ph/mod/quiz/review.php?attempt=981470&cmid=81370 6/8
1/4/22, 11:48 AM PLATING ON CELL AS A UNIT OF HEALTH AND DISEASE: Attempt review

Question 19

Correct

Marked out of 1.00

PLATING ON CELL AS A UNIT OF HEALTH AND DISEASE (MDV)

Which of the following is included in the five major classes of genes that are involved in regulating gene
expression?

a. mRNAs
b. Introns
c. Exons
d. Transposons 

The correct answer is: Transposons

Question 20
Correct

Marked out of 1.00

PLATING ON CELL AS A UNIT OF HEALTH AND DISEASE (MDV)

Which of the following contains all the inheritable traits of an organism?

a. Chromosome
b. Chromatin
c. Genome 
d. Gene

The correct answer is: Genome

https://moodle.feu-nrmf.ph/mod/quiz/review.php?attempt=981470&cmid=81370 7/8
1/4/22, 11:48 AM PLATING ON CELL AS A UNIT OF HEALTH AND DISEASE: Attempt review

Previous activity
◄ 2D SGD QUIZ 9

Jump to...

Next activity
PLATING ON CELL INJURY, CELL DEATH AND ADAPTATIONS ►

Stay in touch

http://www.feu-nrmf.ph

+632 8983 8338 / +639176272623

info@feu-nrmf.ph

 Data retention summary

 Get the mobile app

https://moodle.feu-nrmf.ph/mod/quiz/review.php?attempt=981470&cmid=81370 8/8
1/4/22, 11:48 AM FINAL EXAMINATION: Attempt review (page 1 of 12)

Dashboard My courses Patho 2A 1st Sem SY 2021 MD2-A,B,C,D,E,F,G,H,I,J MAJOR EXAMS

FINAL EXAMINATION

Started on Monday, 6 December 2021, 9:00 AM


State Finished
Completed on Monday, 6 December 2021, 11:29 AM
Time taken 2 hours 28 mins
Grade 73.00 out of 120.00 (61%)

Question 1
Incorrect

Mark 0.00 out of 1.00

INFANCY AND CHILDHOOD DISEASES

Mental retardation, liver cirrhosis, and cataract are clinical features of which of the following inborn errors of
metabolism?

a. Cystic fibrosis
b. G6PD deficiency
c. Galactosemia
d. Phenylketonuria 

The correct answer is: Galactosemia

Question 2
Correct

Mark 1.00 out of 1.00

BLOOD VESSELS (MAE)

Chronic ulceration of toes, feet, fingers followed by gangrene is the natural course of the disease. It is often seen
among smokers

a. Thromboangiitis obliterans 
b. Wegeners granulomatosis
c. Microscopic polyangiitis
d. Churg strauss syndrome

The correct answer is: Thromboangiitis obliterans

https://moodle.feu-nrmf.ph/mod/quiz/review.php?attempt=1344415&cmid=99863 1/5
1/4/22, 11:50 AM PLATING ON DISEASES OF THE IMMUNE SYSTEM PART 1: Attempt review

Question 12

Correct

Mark 1.00 out of 1.00

PLATING ON IMMUNE SYSTEM PART I

A 40 year old has a non-healing skin wound in the leg. The organisms in the wound have started to spread though
the blood. Which of the following organs is the most important in forming immediate adaptive response to the
organisms in the blood?

a. Spleen 
b. Axillary lymph node
c. Peyers patches
d. MALT

The correct answer is: Spleen

Question 13
Incorrect

Mark 0.00 out of 1.00

PLATING ON IMMUNE SYSTEM PART I

Which of the following statements is not true about IgE in Type I hypersensitivity?

a. Th2 cells secrete IL-4 that stimulates B cell differentiation into IgE producing plasma cells 
b. The IgE on mast cells undergo class switching to IgG after the allergic reaction
c. The Fc portion of IgE binds to the Fc portion on mast cell surfaces
d. The IgE on mast cell surface can directly recognize allergen to which it exhibits specificity

The correct answer is: The IgE on mast cells undergo class switching to IgG after the allergic reaction

Question 14
Correct

Mark 1.00 out of 1.00

PLATING ON IMMUNE SYSTEM PART I (ADL)

Prolonged and irreversible functional inactivation of lymphocytes is known a

a. negative selection
b. receptor editing
c. anergy 
d. AIRE

The correct answer is: anergy

https://moodle.feu-nrmf.ph/mod/quiz/review.php?attempt=1117443&cmid=89374 5/9
1/4/22, 11:48 AM FINAL EXAMINATION: Attempt review (page 1 of 12)

Question 5

Correct

Mark 1.00 out of 1.00

BLOOD VESSELS (MAE)

The patient sought consult due to a mass in the lower extremity. Section shows that the resected tumor was a
sheet of tightly packed spindle cells surrounded by thin-walled, endothelium-lined vascular channels with
staghorn configuration. The finding is consistent with

a. Angiosarcoma
b. Hemangiopericytoma 
c. Glioma
d. Hemangioendothelioma

The correct answer is: Hemangiopericytoma

Question 6
Correct

Mark 1.00 out of 1.00

RBC AND BLEEDING DISORDERS (FAA)

MCV = 76 fl, MCH = 19 pg, MCHC = 28 g/dl, what is the etiology of the anemia?

a. Bone marrow suppression


b. Prolonged inflammation
c. Vitamin B12 deficiency
d. Iron deficiency 

The correct answer is: Iron deficiency

Question 7
Incorrect

Mark 0.00 out of 1.00

BLOOD VESSELS (MAE)

An African sought consult due to multiple lymphadenopthies with skin lesions described as reddish brown
patches. Based on the clinical data, the more likely tumor is

a. Hemangiopericytoma
b. Kaposi sarcoma
c. Hemangioendothelioma
d. Glomus tumor 

The correct answer is: Kaposi sarcoma

https://moodle.feu-nrmf.ph/mod/quiz/review.php?attempt=1344415&cmid=99863 3/5
1/4/22, 11:48 AM FINAL EXAMINATION: Attempt review (page 1 of 12)

Question 8

Correct

Mark 1.00 out of 1.00

WBC, LYMPH NODES, SPLEEN AND THYMUS (JQG)

In Hodgkin disease, there is accumulation of variety of reactive macrophages, neutrophils, and lymphocytes.
This is brought about by:

a. Sclerosing stroma
b. Reed Sternberg cells 
c. None othe other choices
d. Deposition of misfolded proteins

The correct answer is: Reed Sternberg cells

Question 9
Correct

Mark 1.00 out of 1.00

LABORATORY QUESTION: There is increased predisposition to this lesion in the following conditions, except in:

a. Premenopausal women 

b. Age of more than 50 years 

c. Familial hypercholesterolemia

d. Diabetes mellitus

The correct answer is:


Premenopausal women

https://moodle.feu-nrmf.ph/mod/quiz/review.php?attempt=1344415&cmid=99863 4/5
1/4/22, 11:48 AM FINAL EXAMINATION: Attempt review (page 1 of 12)

Question 10

Incorrect

Mark 0.00 out of 1.00

HEMODYNAMIC DISOODERS

Partial
thromboplastin time (PTT) assay screens the function of the following factors,
EXCEPT

a. factor VII

b. factor
IX

c. factor
V

d. factor
X

The correct answer is:


factor VII

Previous activity
◄ MIDTERM EXAMINATION

Jump to...

Next activity
LABORATORY CONSULTATION LINK ►

Stay in touch

http://www.feu-nrmf.ph

+632 8983 8338 / +639176272623

info@feu-nrmf.ph

 Data retention summary

 Get the mobile app

https://moodle.feu-nrmf.ph/mod/quiz/review.php?attempt=1344415&cmid=99863 5/5
1/4/22, 11:50 AM PLATING ON DISEASES OF THE IMMUNE SYSTEM PART 1: Attempt review


+632 8983 8338 / +639176272623

info@feu-nrmf.ph

 Data retention summary

 Get the mobile app

https://moodle.feu-nrmf.ph/mod/quiz/review.php?attempt=1117443&cmid=89374 9/9
1/4/22, 11:47 AM MIDTERM EXAMINATION: Attempt review (page 1 of 11)

Question 3

Incorrect

Mark 0.00 out of 1.00

MIDTERM EXAMINATION DISEASES OF IMMUNE SYSTEM I (ADL)

Which is true of chronic discoid lupus erythematosus?

a. systemic manifestations overlap with SLE


b. all cases are positive for ANA 
c. immunofluorescence test show immunoglobulin deposition
d. rash tends to be widespread

The correct answer is: immunofluorescence test show immunoglobulin deposition

Question 4
Correct

Mark 1.00 out of 1.00

MIDTERM NEOPLASIA 2

The most common type of abnormality involving proto-oncogenes has this type of genetic aberration:

a. Chromosomal translocation
b. Gene amplification
c. Epigenetic alteration
d. Point mutation 

The correct answer is: Point mutation

Question 5
Correct

Mark 1.00 out of 1.00

MIDTERM EXAMINATION NEOPLASIA I (RAM)

A mass in the right Fallopian tube was removed and sections of the mass showed histologic features of normal
pancreatic tissue. This is diagnosed as:

a. tubal metaplasia.
b. hamartoma.
c. pleomorphic pancreatoma.
d. choristoma. 

The correct answer is: choristoma.

https://moodle.feu-nrmf.ph/mod/quiz/review.php?attempt=1202445&cmid=93909 2/5
1/4/22, 11:47 AM MIDTERM EXAMINATION: Attempt review (page 1 of 11)

Question 6

Incorrect

Mark 0.00 out of 1.00

PRACTICAL EXAMINATION QUESTION

The presence of the tumor cells in this organ increases the:

a. M in TNM
b. N in TNM
c. T in TNM 

The correct answer is: M in TNM

Question 7
Incorrect

Mark 0.00 out of 1.00

MIDTERM EXAMINATION NEOPLASIA I (RAM)

A mass from the right forearm of a 55 year old male was biopsied revealing a well differentiated
rhabdomyosarcoma. Which feature correlates with the mass?

a. Small, rounded, hyperchromatic nuclei with scanty cytoplasm 


b. Many spindle cells with large, hyperchromatic nuclei and cytoplasmic striations
c. Pleomorphic, hyperchromatic cells with a whorled growth pattern
d. Increased fibrous stroma with tumor giant cells

The correct answer is: Many spindle cells with large, hyperchromatic nuclei and cytoplasmic striations

Question 8
Correct

Mark 1.00 out of 1.00

MIDTERM EXAMINATION DISEASES OF IMMUNE SYSTEM I (ADL)

The following can lead to failure of self tolerance, EXCEPT

a. infections
b. inheritance of susceptibility genes
c. no exception in the following choices 
d. Tissue injury

The correct answer is: no exception in the following choices

https://moodle.feu-nrmf.ph/mod/quiz/review.php?attempt=1202445&cmid=93909 3/5
1/4/22, 11:50 AM PLATING ON GENETIC DISORDERS: Attempt review

Dashboard My courses Patho 2A 1st Sem SY 2021 MD2-A,B,C,D,E,F,G,H,I,J PLATING EXAMS

PLATING ON GENETIC DISORDERS

Started on Monday, 13 September 2021, 12:00 PM


State Finished
Completed on Monday, 13 September 2021, 12:27 PM
Time taken 27 mins 19 secs
Marks 15.00/20.00
Grade 75.00 out of 100.00

Question 1
Correct

Mark 1.00 out of 1.00

PLATING ON GENETIC DISORDERS (RRA)

A pediatrician called a geneticist because she wants confirmation that his patient has Fragile X syndrome. Based
on what you know about the disease, which of the following methods would you recommend?

a. Cytogenomic Array Technology


b. FISH
c. PCR 
d. Karyotype

The correct answer is: PCR

Question 2
Correct

Mark 1.00 out of 1.00

PLATING ON GENETIC DISORDERS (RRA)

A 20-year-old primigravid was seen in the OB admitting section due to decreased fetal sounds. Contractions were
induced and she delivered a stillborn fetus at 19 weeks age of gestation and subsequently sent for autopsy.
Autopsy revealed marked hydrops fetalis, cystic hygroma of the neck, aortic coarctation and horseshoe kidneys.
Which of the following karyotypes does the fetus likely have?

a. 45, X 
b. 47, XX, +21
c. 47, XXY
d. 46, XX/47,XX,+13

The correct answer is: 45, X

https://moodle.feu-nrmf.ph/mod/quiz/review.php?attempt=1107739&cmid=88124 1/9
1/4/22, 11:47 AM MIDTERM EXAMINATION: Attempt review (page 1 of 11)

 Get the mobile app

https://moodle.feu-nrmf.ph/mod/quiz/review.php?attempt=1202445&cmid=93909 5/5
1/4/22, 11:29 AM PRELIMINARY EXAMINATION: Attempt review

Dashboard My courses Patho 2A 1st Sem SY 2021 MD2-A,B,C,D,E,F,G,H,I,J MAJOR EXAMS

PRELIMINARY EXAMINATION

Started on Monday, 6 September 2021, 9:20 AM


State Finished
Completed on Monday, 6 September 2021, 11:04 AM
Time taken 1 hour 44 mins
Grade 63.00 out of 100.00

Question 1
Correct

Mark 1.00 out of 1.00

PRELIMINARY EXAMINATION - HEMODYNAMIC DISORDERS, THROMBOEMBOLIC DISEASES AND SHOCK

The resident pathologist noted that the kidney is infarcted. If you are the intern assisting the pathologist, what is
the expected color of the infarct

a. hemorrhagic
b. anemic 

The correct answer is: anemic

Question 2
Correct

Mark 1.00 out of 1.00

PRELIMINARY EXAMINATION - INFLAMMATION AND REPAIR

Pericardial effusion in patients with myocarditis is an example of what pattern of inflammation?

a. Purulent
b. Fibrinoid
c. Granulomatous
d. Serous 

The correct answer is: Serous

https://moodle.feu-nrmf.ph/mod/quiz/review.php?attempt=1072772&cmid=86983&showall=1 1/42
1/4/22, 11:50 AM PLATING ON GENETIC DISORDERS: Attempt review

Question 6

Incorrect

Mark 0.00 out of 1.00

PLATING ON GENETIC DISORDERS (RRA)

Given the karyotype 45,X/47,XXX which of the following statement is true?

a. Isochromosome formation has occurred in this patient. 


b. This represents a special form of deletion with fusion of damaged ends.
c. This represents a mosaic karyotype of Down Syndrome.
d. This is likely due to a mitotic error during cleavage of the fertilized ovum or in somatic cells.

The correct answer is: This is likely due to a mitotic error during cleavage of the fertilized ovum or in somatic cells.

Question 7
Correct

Mark 1.00 out of 1.00

PLATING ON GENETIC DISORDERS (RRA)

A 10-year-old boy was seen in the pediatrician’s clinic (again) due to coughs and colds. The pediatrician was
alarmed because the infections seem to be recurrent, with the patient having otitis media, pneumonia, a diarrhea
in past 6 months. Physical examination shows a murmur suggesting of a congenital heart disease. CT scan of the
chest reveals a small thymus. Which of the following genetic defects is likely present in this patient?

a. Karyotype of 46,XX/47,XX, +18


b. Small deletion of band q11.2 in the long arm of chromosome 22 
c. Karyotype of 47, XYY
d. More than 200 repeats of CGG

The correct answer is: Small deletion of band q11.2 in the long arm of chromosome 22

Question 8
Correct

Mark 1.00 out of 1.00

PLATING ON GENETIC DISORDERS (RGB)

A ten month old baby girl was noted to show muscular flaccidity and failure to develop normal milestone was
brought to a Pediatrician. Cherry red spot was noted in the cornea. Enzyme assay revealed deficiency of
hexoaminidase A. Which of the following morphologic findings would you expect to see in the baby?

a. Neurons with large lipid cytoplasmic vacuoles 


b. Distended phagocytic cells with crumpled tissue paper appearance
c. Presence of balloon cells and zebra bodies in phagocytic cells
d. Marked visceral accumulation of sphingomyelin

The correct answer is: Neurons with large lipid cytoplasmic vacuoles

https://moodle.feu-nrmf.ph/mod/quiz/review.php?attempt=1107739&cmid=88124 3/9
1/4/22, 11:29 AM PRELIMINARY EXAMINATION: Attempt review

Question 6

Correct

Mark 1.00 out of 1.00

PRELIMINARY EXAMINATION - CELL INJURY, CELL DEATH AND ADAPTATION

Which of the following is consistent with necroptosis?

a. caspase-independent 
b. prevented by iron-chelation
c. there is widespread peroxidation of lipids
d. interleukin I release is characteristic

The correct answer is: caspase-independent

Question 7
Correct

Mark 1.00 out of 1.00

PRELIMINARY EXAMINATION – CELL AS AUNIT OF HEALTH AND DISEASE (MDV)

The main epigenetic mechanisms include the following EXCEPT

a. Histone acetylation
b. Histone methylation
c. DNA phosphorylation 
d. DNA methylation

The correct answer is: DNA phosphorylation

Question 8
Incorrect

Mark 0.00 out of 1.00

PRELIMINARY EXAMINATION - GENETIC DISORDERS (RGB)

Which of the following Polymerase Chain Reactions is used to detect mutations characterized by trinucleotide
expansion like Fragile X syndrome?

a. Real time
b. Single base primer extension
c. New generation sequencing 
d. Amplicon length analysis

The correct answer is: Amplicon length analysis

https://moodle.feu-nrmf.ph/mod/quiz/review.php?attempt=1072772&cmid=86983&showall=1 3/42
1/4/22, 11:29 AM PRELIMINARY EXAMINATION: Attempt review

Question 9

Incorrect

Mark 0.00 out of 1.00

PRELIMINARY EXAMINATION - GENETIC DISORDERS (RGB)

A 27-year- old male sought consult because of a long face, large mandible, hyperextensible joints and macro-
orchidism which were not apparent during his childhood. Which of the following genetic abnormalities can explain
these clinical findings?

a. Gonadal mosaicism 
b. Genomic imprinting
c. Triplet repeat mutation
d. Mitochondrial gene mutation

The correct answer is: Triplet repeat mutation

Question 10
Incorrect

Mark 0.00 out of 1.00

PRELIMINARY EXAMINATION - CELL INJURY, CELL DEATH AND ADAPTATION

Proteolysis in coagulative necrosis is inhibited by which of the following?

a. lipase
b. hypoxia 
c. water influx
d. intracellular acidosis

The correct answer is: intracellular acidosis

Question 11
Correct

Mark 1.00 out of 1.00

PRELIMINARY EXAMINATION - GENETIC DISORDERS (RGB)

A 17 -year- old male was noted to have long legs with eunuchoid body habitus. He also has gynecomastia, small
penis and atrophic testes. Which of the following karyotypes is probably present in this patient?

a. 47, XXY 
b. 46, XY
c. 47, XYY
d. 45, X

The correct answer is: 47, XXY

https://moodle.feu-nrmf.ph/mod/quiz/review.php?attempt=1072772&cmid=86983&showall=1 4/42
1/4/22, 11:29 AM PRELIMINARY EXAMINATION: Attempt review

Question 12

Correct

Mark 1.00 out of 1.00

PRELIMINARY EXAMINATION – CELL AS AUNIT OF HEALTH AND DISEASE (MDV)

This has a main role in posttranscriptional silencing of gene expression

a. mRNA
b. miRNA 
c. cRNA
d. lncRNA

The correct answer is: miRNA

Question 13
Incorrect

Mark 0.00 out of 1.00

PRELIMINARY EXAMINATION – CELL AS AUNIT OF HEALTH AND DISEASE (MDV)

Within cells, these serve as connecting cables for molecular motor

a. F-actin
b. Intermediate filaments 
c. Microtubules
d. G-actin

The correct answer is: Microtubules

Question 14
Correct

Mark 1.00 out of 1.00

PRELIMINARY EXAMINATION - HEMODYNAMIC DISORDERS, THROMBOEMBOLIC DISEASES AND SHOCK

An elderly man slipped and strikes the back of his head. He becomes increasingly somnolent. A head CT scan
shows an accumulation of fluid beneath the dura, compressing the left cerebral hemisphere. Which of the
following terms best describes this collection of fluid?

a. Congestion
b. Hyperemia
c. Petechiae
d. Hematoma 

The correct answer is: Hematoma

https://moodle.feu-nrmf.ph/mod/quiz/review.php?attempt=1072772&cmid=86983&showall=1 5/42
1/4/22, 11:29 AM PRELIMINARY EXAMINATION: Attempt review

Question 15

Correct

Mark 1.00 out of 1.00

PRELIMINARY EXAMINATION – CELL AS AUNIT OF HEALTH AND DISEASE (ORB)

Stem cell niches are normally protected within specialized microenvironments. Which of the following is NOT TRUE
of stem cell niches?

a. Epithelial stem cells are confined to the crypts


b. Observed in subcortical zone of the brain 
c. Hematopoietic stem cells characteristically congregate in perivascular niches
d. Located in the bulge region of hair follicles

The correct answer is: Observed in subcortical zone of the brain

Question 16
Incorrect

Mark 0.00 out of 1.00

PRELIMINARY EXAMINATION - HEMODYNAMIC DISORDERS, THROMBOEMBOLIC DISEASES AND SHOCK

The most common source of emboli lodging in the lungs

a. brain
b. intestine
c. lungs 
d. lower extremities

The correct answer is: lower extremities

https://moodle.feu-nrmf.ph/mod/quiz/review.php?attempt=1072772&cmid=86983&showall=1 6/42
1/4/22, 11:50 AM PLATING ON GENETIC DISORDERS: Attempt review

Previous activity
◄ PLATING ON HEMODYNAMIC DISORDERS, THROMBOEMBOLIC DISEASES AND SHOCK

Jump to...

Next activity
PLATING ON DISEASES OF THE IMMUNE SYSTEM PART 1 ►

Stay in touch

http://www.feu-nrmf.ph

+632 8983 8338 / +639176272623

https://moodle.feu-nrmf.ph/mod/quiz/review.php?attempt=1107739&cmid=88124 8/9
1/4/22, 11:29 AM PRELIMINARY EXAMINATION: Attempt review

Question 20

Incorrect

Mark 0.00 out of 1.00

PRELIMINARY EXAMINATION - GENETIC DISORDERS (RRA)

CASE WITH IMAGE. A 5-month-old male infant was seen in the pediatrics clinic due to failure to thrive and
abdominal enlargement. The parents also observed that the baby was showing only minimal movement since
birth compared to his peers. Physical examination showed severe muscle weakness and hepatosplenomegaly
while a chest radiograph shows cardiomegaly. After one year, the patient died and an autopsy was done.
Microscopy of the myocardium is seen below. A mutation in the genes encoding for which protein type is likely
causing the child’s disease?

a. Receptor
b. Structural protein 
c. Enzyme
d. Growth regulation

The correct answer is: Enzyme

Question 21
Incorrect

Mark 0.00 out of 1.00

PRELIMINARY EXAMINATION - HEMODYNAMIC DISORDERS, THROMBOEMBOLIC DISEASES AND SHOCK

A college nursing student has had multiple episodes of deep venous thrombosis during the past 10 years and one
episode of pulmonary thromboembolism during the past year. Her prothrombin time (PT), partial thromboplastin
time (PTT), platelet count, and platelet function studies all are normal. Which of the following risk factors is the
more likely cause?

a. Factor V mutation
b. Smoking cigarette
c. Hyperhomocysteinemia
d. Antithrombin III deficiency 

The correct answer is: Factor V mutation

https://moodle.feu-nrmf.ph/mod/quiz/review.php?attempt=1072772&cmid=86983&showall=1 8/42
1/4/22, 11:29 AM PRELIMINARY EXAMINATION: Attempt review

Question 22

Incorrect

Mark 0.00 out of 1.00

PRELIMINARY EXAMINATION - GENETIC DISORDERS (RGB)

Which of the following is true of a female pseudohermaphrodite?

a. positive testicular tissue with female external genitalia 


b. positive ovaries with male external genitalia
c. XY, with male external genitalia
d. XX, with female external genitalia

The correct answer is: positive ovaries with male external genitalia

Question 23
Correct

Mark 1.00 out of 1.00

PRELIMINARY EXAMINATION – CELL AS AUNIT OF HEALTH AND DISEASE (ORB)

Growth factors when bound to their specific receptors influences expression of genes necessary to:

a. Allow apoptosis
b. Enhance component synthesis 
c. Stop cell cycle entry
d. Inhibit replication

The correct answer is: Enhance component synthesis

Question 24
Correct

Mark 1.00 out of 1.00

PRELIMINARY EXAMINATION - INFLAMMATION AND REPAIR

A 5 year old has a wound on his knee. On the third day of the wound, it is seen covered by a thick yellowish white
fluid. Which of the following will be seen if the yellowish white thick fluid will be placed on a glass slide, stained, and
examined under the microscope?

a. Cell debris
b. Necrotic neutrophils
c. All of the given choices 
d. Neutrophils

The correct answer is: All of the given choices

https://moodle.feu-nrmf.ph/mod/quiz/review.php?attempt=1072772&cmid=86983&showall=1 9/42
1/4/22, 11:29 AM PRELIMINARY EXAMINATION: Attempt review

Question 25

Correct

Mark 1.00 out of 1.00

PRELIMINARY EXAMINATION - CELL INJURY, CELL DEATH AND ADAPTATION

Cells from children have the capacity to undergo more rounds of replication than do cells from older people.

a. replicative senescence 
b. dysregulated nutrient sensing
c. impaired chaperone and proteasome functions

The correct answer is: replicative senescence

Question 26
Correct

Mark 1.00 out of 1.00

PRELIMINARY EXAMINATION - INFLAMMATION AND REPAIR

Component/s of tissues undergoing repair:

a. Connective tissue proliferation


b. All of the other choices 
c. Collagen deposition
d. Newly formed cells

The correct answer is: All of the other choices

Question 27
Incorrect

Mark 0.00 out of 1.00

PRACTICAL EXAMINATION QUESTION

Which of the following statements is true about this cellular adaptive change?

a. This is an irreversible change 


b. All of the given choices
c. The cellular adaptive change allows cells to better withstand the chronic irritation or the stress
d. The differentiated columnar cells transformed to become squamous cells

The correct answer is: The cellular adaptive change allows cells to better withstand the chronic irritation or the
stress

https://moodle.feu-nrmf.ph/mod/quiz/review.php?attempt=1072772&cmid=86983&showall=1 10/42
1/4/22, 11:29 AM PRELIMINARY EXAMINATION: Attempt review

Question 28

Correct

Mark 1.00 out of 1.00

PRELIMINARY EXAMINATION - CELL INJURY, CELL DEATH AND ADAPTATION

Activation of various enzymes like phospholipase in cell injury is attributed mainly to which of these?

a. extracellular calcium depletion


b. lipid peroxidation
c. increased cytosolic Ca2+ 
d. protein misfolding

The correct answer is: increased cytosolic Ca2+

Question 29
Correct

Mark 1.00 out of 1.00

PRELIMINARY EXAMINATION - INFLAMMATION AND REPAIR

A non-healing wound on the left leg for one month of a 60 year old diabetic female was surgically removed and
was submitted to the laboratory for histopathologic examination. The diagnosis of the pathologist was "chronic
inflammation." The following are features of this type of inflammation, except:

a. The local and systemic signs are prominent 


b. This type of inflammation followed an acute inflammatory process
c. The tissue injury is severe and progressive
d. Monocytes and lymphocytes are the main inflammatory cell infiltrates

The correct answer is: The local and systemic signs are prominent

Question 30
Correct

Mark 1.00 out of 1.00

PRELIMINARY EXAMINATION - INFLAMMATION AND REPAIR

A 49 year old man had hepatitis B infection two years ago. Recently, he is apparently well and his hepatitis profile
showed absence of HBsAg and positive for anti-HBsAg. Which of the following is TRUE?

a. Any of the other choices


b. The liver cells has capability to regenerate when needed, but are generally quiescent without injury 
c. His liver cells underwent regeneration process because the liver is composed of labile cells
d. His liver became smaller due to fibrosis because the cells are permanent cells

The correct answer is: The liver cells has capability to regenerate when needed, but are generally quiescent
without injury

https://moodle.feu-nrmf.ph/mod/quiz/review.php?attempt=1072772&cmid=86983&showall=1 11/42
1/4/22, 11:29 AM PRELIMINARY EXAMINATION: Attempt review

Question 31

Correct

Mark 1.00 out of 1.00

PRELIMINARY EXAMINATION - HEMODYNAMIC DISORDERS, THROMBOEMBOLIC DISEASES AND SHOCK

A 65-year-old woman has a brief fainting episode. She was diagnosed with pancreatic adenocarcinoma.
Auscultation revealed heart murmur is heard. Echocardiography shows a 1-cm nodular lesion on the superior
aspect of an intact anterior mitral valve leaflet. A blood culture is negative. Which of the following terms best
describes this mitral valve lesion?

a. Chronic passive congestion


b. Mural thrombus
c. Atheroma
d. Vegetation 

The correct answer is: Vegetation

Question 32
Incorrect

Mark 0.00 out of 1.00

PRELIMINARY EXAMINATION - INFLAMMATION AND REPAIR

The presence of gram negative bacterial organisms with lipopolysaccharide in the tissue can activate which
pathway of the complement system?

a. Alternative
b. Lectin 
c. Classical

The correct answer is: Alternative

Question 33

Correct

Mark 1.00 out of 1.00

PRELIMINARY EXAMINATION - HEMODYNAMIC DISORDERS, THROMBOEMBOLIC DISEASES AND SHOCK

An underwater construction worker came in at the ER with complaints of difficulty of breathing and joint pains. The
more likely cause of embolism is

a. thrombi
b. air 
c. fat
d. amniotic fluid

The correct answer is: air

https://moodle.feu-nrmf.ph/mod/quiz/review.php?attempt=1072772&cmid=86983&showall=1 12/42
1/4/22, 11:29 AM PRELIMINARY EXAMINATION: Attempt review

Question 34

Correct

Mark 1.00 out of 1.00

PRACTICAL EXAMINATION QUESTION

Which of the following causes of necrosis is most applicable to this?

a. Tissue digestion by enzymes from leukocytes


b. Ischemia due to blood vessel obstruction 
c. Inflammation and immune response with continuous activation immune cells

The correct answer is: Ischemia due to blood vessel obstruction

Question 35
Incorrect

Mark 0.00 out of 1.00

PRELIMINARY EXAMINATION - HEMODYNAMIC DISORDERS, THROMBOEMBOLIC DISEASES AND SHOCK

A pediatrician noticed that his son is lethargic for the past three days. Periorbital edema and pitting edema at the
ankles are observed. He is normotensive, afebrile with marked albuminuria. He is given a thiazide diuretic and his
urine output increases and his edema resolves. Which of the following changes most likely potentiated his
edema?

a. osmotic pressure
b. inflammation
c. neurogenic
d. hydrostatic pressure 

The correct answer is: osmotic pressure

Question 36
Incorrect

Mark 0.00 out of 1.00

PRELIMINARY EXAMINATION - GENETIC DISORDERS (RGB)

Which of the following is an indication for newborn genetic testing?

a. Presence of neurodegenerative disorders 


b. Fetal anomalies observed on ultrasound
c. Suspicion of a metabolic syndrome
d. Advanced maternal age

The correct answer is: Suspicion of a metabolic syndrome

https://moodle.feu-nrmf.ph/mod/quiz/review.php?attempt=1072772&cmid=86983&showall=1 13/42
1/4/22, 11:29 AM PRELIMINARY EXAMINATION: Attempt review

Question 37

Incorrect

Mark 0.00 out of 1.00

PRELIMINARY EXAMINATION - GENETIC DISORDERS (RRA)

A 10-year-old boy was seen in the pediatrics clinic due to poor wound healing even with minor trauma ever since
infancy. Physical examination shows hyperextensible joints and a fragile but extremely stretchable skin. One
parent and 2 other siblings are affected by the same condition. What is the likely defect causing this condition?

a. Defect in the synthesis or structure of fibrillar collage


b. Growth regulation
c. Reduced synthesis of dystrophin
d. Inherited defect in fibrillin-1 

The correct answer is: Defect in the synthesis or structure of fibrillar collage

Question 38
Correct

Mark 1.00 out of 1.00

PRELIMINARY EXAMINATION - CELL INJURY, CELL DEATH AND ADAPTATION

Hypoxic brain injury typically results into which pattern of necrosis?

a. fat necrosis
b. coagulative necrosis
c. fibrinoid necrosis
d. liquefactive necrosis 

The correct answer is: liquefactive necrosis

Question 39
Correct

Mark 1.00 out of 1.00

PRELIMINARY EXAMINATION - CELL INJURY, CELL DEATH AND ADAPTATION

Which pathway is involved in exercise-induced hypertrophy?

a. ubiquitin-proteasome pathway
b. p53-dependent pathway
c. phosphoinositide 3-kinase pathway 

The correct answer is: phosphoinositide 3-kinase pathway

https://moodle.feu-nrmf.ph/mod/quiz/review.php?attempt=1072772&cmid=86983&showall=1 14/42
1/4/22, 11:29 AM PRELIMINARY EXAMINATION: Attempt review

Question 40

Incorrect

Mark 0.00 out of 1.00

PRELIMINARY EXAMINATION - GENETIC DISORDERS (RGB)

Which of the following structural genetic abnormalities is characterized by loss of genetic information?

a. Deletion
b. Ring chromosome 
c. Robertsonian translocation
d. All of these

The correct answer is: All of these

Question 41
Correct

Mark 1.00 out of 1.00

PRELIMINARY EXAMINATION - INFLAMMATION AND REPAIR

A second year medicine student has areas of abrasion on her facial skin following an aesthetic procedure. While
reviewing for Pathology A major exam, she took a nap on her table and bacterial organisms entered the small
abraded areas of the facial skin. Which of the following events will follow recognition of the bacterial organisms by
the sentinel cells?

a. Blood vessels will respond to inflammatory stimuli and leukocytes will be recruited 
b. Bacterial organisms will be recognized by leukocytes that have undergone chemotaxis
c. Inflammatory reaction will be terminated
d. Transforming growth factor will be released to promote tissue wound healing

The correct answer is: Blood vessels will respond to inflammatory stimuli and leukocytes will be recruited

Question 42
Correct

Mark 1.00 out of 1.00

PRELIMINARY EXAMINATION - HEMODYNAMIC DISORDERS, THROMBOEMBOLIC DISEASES AND SHOCK

This stage of shock is characterized by intracellular aerobic respiration replaced by anaerobic glycolysis with
production of lactic acid

a. non-progressive
b. irreversible
c. progressive 

The correct answer is: progressive

https://moodle.feu-nrmf.ph/mod/quiz/review.php?attempt=1072772&cmid=86983&showall=1 15/42
1/4/22, 11:29 AM PRELIMINARY EXAMINATION: Attempt review

Question 43

Incorrect

Mark 0.00 out of 1.00

PRELIMINARY EXAMINATION – CELL AS AUNIT OF HEALTH AND DISEASE (ORB)

These are ectodermally-derived structures EXCEPT:

a. Epidermis 
b. Neurons
c. Alveolar Cells
d. Tooth Enamel

The correct answer is: Alveolar Cells

Question 44
Correct

Mark 1.00 out of 1.00

PRELIMINARY EXAMINATION - CELL INJURY, CELL DEATH AND ADAPTATION

Predisposes to malignant transformation.

a. hypertrophy
b. ferroptosis
c. metaplasia 
d. atrophy

The correct answer is: metaplasia

Question 45
Correct

Mark 1.00 out of 1.00

PRELIMINARY EXAMINATION - INFLAMMATION AND REPAIR

A 65 year old is admitted in the ICU, intubated, and is being treated for Covid-19 infection. The patient has
pneumonia and pleural effusion. The patient is diabetic; no other known diseases. The fluid in the thoracic cavity of
the patient is expected to have:

a. low protein content 


b. high specific gravity
c. cellular debris
d. All of the given choices

The correct answer is: low protein content

https://moodle.feu-nrmf.ph/mod/quiz/review.php?attempt=1072772&cmid=86983&showall=1 16/42
1/4/22, 11:29 AM PRELIMINARY EXAMINATION: Attempt review

Question 46

Correct

Mark 1.00 out of 1.00

PRELIMINARY EXAMINATION - GENETIC DISORDERS (RGB)

Aneuploidy can be caused by nondisjunction which produces which of the following types of zygote upon
fertilization?

a. Monosomic 
b. Normal
c. Both normal and monosomic
d. Neither normal nor monosomic

The correct answer is: Monosomic

Question 47
Incorrect

Mark 0.00 out of 1.00

PRELIMINARY EXAMINATION - GENETIC DISORDERS (RRA)

A 17-year-old unusually tall male was seen in the clinic because sudden loss of vision in the right eye. Physical
examination shows subluxation of the left crystalline lens and a mid-systolic click on the chest during
auscultation. Echocardiogram shows a floppy mitral valve and a dilated aortic arch. His brother and cousin also
have the similar condition and all of them are taking beta blockers. If left unattended, which of the following events
can cause the patient’s death?

a. Congestive heart failure 


b. Acute respiratory distress syndrome
c. None of the given choices
d. Aortic dissection

The correct answer is: Aortic dissection

Question 48
Incorrect

Mark 0.00 out of 1.00

PRELIMINARY EXAMINATION - GENETIC DISORDERS (RRA)

A 20-year-old male underwent autopsy after suddenly dying alone in his apartment. Upon investigation, the
patient died of myocardial infarction and there was extensive atherosclerosis on many coronary vessels. Other
noteworthy findings include skin xanthomas. Which of the following statements holds true regarding the patient’s
condition?

a. Heterozygotes of this condition produces 5-6x the levels of cholesterol.


b. The patient is likely not producing any LDL receptor.
c. There are only 2 known groups of mutations causing the patient’s disorder.
d. HDL receptors are deranged in this condition. 

The correct answer is: The patient is likely not producing any LDL receptor.

https://moodle.feu-nrmf.ph/mod/quiz/review.php?attempt=1072772&cmid=86983&showall=1 17/42
1/4/22, 11:29 AM PRELIMINARY EXAMINATION: Attempt review

Question 49

Correct

Mark 1.00 out of 1.00

PRELIMINARY EXAMINATION - HEMODYNAMIC DISORDERS, THROMBOEMBOLIC DISEASES AND SHOCK

A security guard who works at the gate of SM Fairview noticed that at the end of her 8-hour shift that her lower
legs and feet are swollen. There was no swelling at the beginning of the day. Laboratory testing reveals normal
liver and renal function. Which of the following mechanisms best explain this phenomenon?

a. Secondary aldosteronism
b. Hypoalbuminemia
c. Lymphatic obstruction
d. Increased hydrostatic pressure 

The correct answer is: Increased hydrostatic pressure

Question 50
Correct

Mark 1.00 out of 1.00

PRACTICAL EXAMINATION QUESTION

Which of the following statements is not true about the abnormality in this chromosome? 

a. If regions 11, 12 and 13 will be lost in the process, it will also result to terminal deletion
b. This is a structural chromosomal abnormality
c. This is known as pericentric inversion in which a segment of the chromosome is reversed 
d. There is chromosomal rearrangement that does not involve the chromosome centromere

The correct answer is: This is known as pericentric inversion in which a segment of the chromosome is reversed

https://moodle.feu-nrmf.ph/mod/quiz/review.php?attempt=1072772&cmid=86983&showall=1 18/42
1/4/22, 11:49 AM PLATING ON HEMODYNAMIC DISORDERS, THROMBOEMBOLIC DISEASES AND SHOCK: Attempt review

Question 15

Correct

Mark 1.00 out of 1.00

PLATING ON HEMODYNAMIC DISORDERS, THROMBOEMBOLIC DISEASES AND SHOCK

An important activity of thrombin

a. Platelet inhibitor
b. Anti-inflammatory effects
c. Anticoagulant effects 
d. Conversion of plasminogen into plasmin

The correct answer is: Anticoagulant effects

Question 16
Correct

Mark 1.00 out of 1.00

PLATING ON HEMODYNAMIC DISORDERS, THROMBOEMBOLIC DISEASES AND SHOCK

Presence of squamous cells from fetal skin, lanugo hairs, fat from vernix caseosa and mucin in the maternal
pulmonary vasculature were evident on autopsy:

a. Fat embolism
b. Pulmonary thromboembolism
c. Air embolism
d. Amniotic fluid embolism 

The correct answer is: Amniotic fluid embolism

Question 17
Correct

Mark 1.00 out of 1.00

PLATING ON HEMODYNAMIC DISORDERS, THROMBOEMBOLIC DISEASES AND SHOCK

Hemorrhages that measure more than 1 cm

a. Ecchymoses 
b. Petechia
c. Hematoma
d. Purpura

The correct answer is: Ecchymoses

https://moodle.feu-nrmf.ph/mod/quiz/review.php?attempt=1063809&cmid=86090 6/8
1/4/22, 11:29 AM PRELIMINARY EXAMINATION: Attempt review

Question 53

Correct

Mark 1.00 out of 1.00

PRELIMINARY EXAMINATION - HEMODYNAMIC DISORDERS, THROMBOEMBOLIC DISEASES AND SHOCK

A 56-year-old came in due to sudden onset of chest pain that radiates to her neck and left arm. She is diaphoretic
and hypotensive. Her serum troponin I level is elevated. Which of the following best explains the findings

a. hyperemia
b. thrombosis 
c. edema
d. congestion

The correct answer is: thrombosis

Question 54
Incorrect

Mark 0.00 out of 1.00

PRELIMINARY EXAMINATION – CELL AS AUNIT OF HEALTH AND DISEASE (ORB)

HGF (scatter factor) functions to:

a. Increase cell motility


b. Suppress acute inflammation
c. Increase vascular permeability
d. Stimulates ECM protein synthesis 

The correct answer is: Increase cell motility

Question 55
Correct

Mark 1.00 out of 1.00

PRELIMINARY EXAMINATION - HEMODYNAMIC DISORDERS, THROMBOEMBOLIC DISEASES AND SHOCK

A medical technologist has a history of frequent nosebleeds and increased menstrual blood flow. Petechiae and
purpura are present on the skin of her extremities. Laboratory studies show normal partial thromboplastin time
(PTT), prothrombin time (PT), and platelet count, but decreased von Willebrand factor activity. This patient most
likely has a derangement in which of the following steps in hemostasis?

a. Fibrin polymerization
b. Platelet aggregation
c. Vasoconstriction
d. Platelet adhesion 

The correct answer is: Platelet adhesion

https://moodle.feu-nrmf.ph/mod/quiz/review.php?attempt=1072772&cmid=86983&showall=1 20/42
1/4/22, 11:29 AM PRELIMINARY EXAMINATION: Attempt review

Question 56

Incorrect

Mark 0.00 out of 1.00

PRELIMINARY EXAMINATION – CELL AS AUNIT OF HEALTH AND DISEASE (MDV)

Mobile genetic elements that can regulate gene expression

a. Promoter regions
b. Enhancer regions
c. Centromeres 
d. Transposons

The correct answer is: Transposons

Question 57
Correct

Mark 1.00 out of 1.00

PRELIMINARY EXAMINATION - INFLAMMATION AND REPAIR

The surgical wound of a 25 year old who recently delivered to a baby via cesarean section appears to be infected.
Her body temperature is 37 C and her CBC revealed elevated white blood cells with increased neutrophils. What
systemic effect of inflammation is present in this case?

a. Leukocytosis 
b. Fever
c. All of the given choices
d. Lymphocytosis

The correct answer is: Leukocytosis

Question 58
Correct

Mark 1.00 out of 1.00

PRELIMINARY EXAMINATION – CELL AS AUNIT OF HEALTH AND DISEASE (ORB)

Which of the following is a non-fibrillar collagen?

a. Type I
b. Type III
c. Type II
d. Type IV 

The correct answer is: Type IV

https://moodle.feu-nrmf.ph/mod/quiz/review.php?attempt=1072772&cmid=86983&showall=1 21/42
1/4/22, 11:29 AM PRELIMINARY EXAMINATION: Attempt review

Question 59

Correct

Mark 1.00 out of 1.00

PRELIMINARY EXAMINATION - CELL INJURY, CELL DEATH AND ADAPTATION

Consistent with dystrophic calcification.

a. hypercalcemia is certain
b. seen in hyperparathyroidism
c. seen in necrotic area of injury 
d. vitamin D deficiency is likely

The correct answer is: seen in necrotic area of injury

Question 60
Correct

Mark 1.00 out of 1.00

PRELIMINARY EXAMINATION - HEMODYNAMIC DISORDERS, THROMBOEMBOLIC DISEASES AND SHOCK

A salesman is involved in a vehicular accident and sustains fractures of the right femur and tibia and the left
humerus. He suddenly becomes severely dyspneic. Which of the following complications from his injuries is the
most likely cause of his sudden respiratory difficulty?

a. Fat embolism 
b. Pulmonary edema
c. Pulmonary infarction
d. Cardiac tamponade

The correct answer is: Fat embolism

Question 61
Incorrect

Mark 0.00 out of 1.00

PRELIMINARY EXAMINATION - GENETIC DISORDERS (RRA)

Li-Fraumeni syndrome is a disease were in an individual has a familial predisposition to a wide range of cancers
due to a mutation in the tumor suppressor gene known as p53. Which characteristic of Mendelian disorders does
this disease demonstrate?

a. Pleiotropism
b. None of the given choices
c. Co-dominance
d. Genetic heterogeneity 

The correct answer is: Pleiotropism

https://moodle.feu-nrmf.ph/mod/quiz/review.php?attempt=1072772&cmid=86983&showall=1 22/42
1/4/22, 11:29 AM PRELIMINARY EXAMINATION: Attempt review

Question 62

Correct

Mark 1.00 out of 1.00

PRELIMINARY EXAMINATION – CELL AS AUNIT OF HEALTH AND DISEASE (MDV)

Which of the following regulatory RNAs is responsible for human phenotypic diversity?

a. lncRNA 
b. mRNA
c. miRNA
d. cRNA

The correct answer is: lncRNA

Question 63
Incorrect

Mark 0.00 out of 1.00

PRELIMINARY EXAMINATION - HEMODYNAMIC DISORDERS, THROMBOEMBOLIC DISEASES AND SHOCK

A 20-year old with uncomplicated pregnancy develops sudden dyspnea with cyanosis and hypotension
intrapartum during routine vaginal delivery of a term infant. She has a generalized seizure and becomes
comatose. Which of the following findings is most likely to be present in her peripheral pulmonary arteries?

a. Fat globules
b. Gas bubbles
c. Amniotic fluid
d. Aggregates of platelets 

The correct answer is: Amniotic fluid

Question 64
Incorrect

Mark 0.00 out of 1.00

PRELIMINARY EXAMINATION - GENETIC DISORDERS (RRA)

Which of the following statements best describes Common Disease/Common Variant Hypothesis?

a. It suggests that the more polymorphisms an individual inherits, the more symptoms the individual
manifests.
b. It suggests that complex genetic disorders occur when many polymorphisms are co-inherited.
c. None of the given choices.
d. It suggests that there is only one molecular defect the drives the formation/progression of the disease. 

The correct answer is: It suggests that complex genetic disorders occur when many polymorphisms are co-
inherited.

https://moodle.feu-nrmf.ph/mod/quiz/review.php?attempt=1072772&cmid=86983&showall=1 23/42
1/4/22, 11:29 AM PRELIMINARY EXAMINATION: Attempt review

Question 65

Correct

Mark 1.00 out of 1.00

PRELIMINARY EXAMINATION - CELL INJURY, CELL DEATH AND ADAPTATION

Implicated in neurodegenerative disease.

a. caspases
b. autophagy 
c. ferroptosis
d. metaplasia

The correct answer is: autophagy

Question 66
Correct

Mark 1.00 out of 1.00

PRELIMINARY EXAMINATION – CELL AS AUNIT OF HEALTH AND DISEASE (ORB)

Which of the following is NOT a function of the extracellular matrix?

a. Mechanical support
b. Regulator of cell proliferation
c. Scaffolding for tissue renewal
d. Prohibit compressive resistance 

The correct answer is: Prohibit compressive resistance

Question 67
Incorrect

Mark 0.00 out of 1.00

PRACTICAL EXAMINATION QUESTION

The necrosis that accompanies this pattern of inflammation is due to:

a. free radical-mediated injury


b. release of leukocyte digestive enzymes into the tissue interstitium 
c. liquefaction of cells and tissues
d. fibrin formation and deposition 

The correct answer is: free radical-mediated injury

https://moodle.feu-nrmf.ph/mod/quiz/review.php?attempt=1072772&cmid=86983&showall=1 24/42
1/4/22, 11:29 AM PRELIMINARY EXAMINATION: Attempt review

Question 68

Correct

Mark 1.00 out of 1.00

PRELIMINARY EXAMINATION - INFLAMMATION AND REPAIR

A preschooler was diagnosed to have Leukocyte Adhesion Deficiency Type 2 with defective beta chain of LFA-1 and
Mac-1 integrins. Which of the following statements is not true about this condition?

a. The process of leukocyte rolling is defective due to non-functional leukocyte surface molecule 
b. Leukocytes will not be available in the site of infection to clear the tissue of bacterial organisms
c. Firm adhesion of leukocyte to endothelium is not possible using the defective surface molecules
d. The leukocytes will not be able to proceed with transmigration

The correct answer is: The process of leukocyte rolling is defective due to non-functional leukocyte surface
molecule

Question 69
Incorrect

Mark 0.00 out of 1.00

PRELIMINARY EXAMINATION – CELL AS AUNIT OF HEALTH AND DISEASE (MDV)

Which of these proteins bind their specific solute and undergo a series of conformational changes to transfer the
ligand across the membrane?

a. Transmembrane protein 
b. Channel protein
c. Carrier protein
d. Peripheral membrane protein

The correct answer is: Carrier protein

https://moodle.feu-nrmf.ph/mod/quiz/review.php?attempt=1072772&cmid=86983&showall=1 25/42
1/4/22, 11:29 AM PRELIMINARY EXAMINATION: Attempt review

Question 70

Correct

Mark 1.00 out of 1.00

PRACTICAL EXAMINATION QUESTION

What is the fate of the structure at the tip of the pointer if they get dislodged and travel to another part of the
body?

a. FIbrinolysis
b. Propagation
c. Embolization 
d. Recanalization

The correct answer is: Embolization

Question 71
Correct

Mark 1.00 out of 1.00

PRELIMINARY EXAMINATION - HEMODYNAMIC DISORDERS, THROMBOEMBOLIC DISEASES AND SHOCK

A bank manager with a history of diabetes mellitus had a myocardial infarction 7 months ago. She was given low
dose of aspirin. On which of the following steps in hemostasis does aspirin have its greatest effect?

a. vasoconstriction
b. Production of thrombin
c. Synthesis of von Willebrand factor
d. Aggregation of platelets 

The correct answer is: Aggregation of platelets

https://moodle.feu-nrmf.ph/mod/quiz/review.php?attempt=1072772&cmid=86983&showall=1 26/42
1/4/22, 11:29 AM PRELIMINARY EXAMINATION: Attempt review

Question 72

Correct

Mark 1.00 out of 1.00

PRELIMINARY EXAMINATION – CELL AS AUNIT OF HEALTH AND DISEASE (MDV)

Which of the following binds histones that bear particular marks and regulate gene expression?

a. Chromatin erasers
b. Chromatin markers
c. Chromatin readers 
d. Chromatin writers

The correct answer is: Chromatin readers

Question 73
Incorrect

Mark 0.00 out of 1.00

PRELIMINARY EXAMINATION – CELL AS AUNIT OF HEALTH AND DISEASE (ORB)

This checkpoint monitors DNA integrity before irreversibly committing cellular resources to DNA replication:

a. G1-S
b. S-G2 
c. G2-M
d. M-G1

The correct answer is: G1-S

Question 74
Correct

Mark 1.00 out of 1.00

PRELIMINARY EXAMINATION - CELL INJURY, CELL DEATH AND ADAPTATION SARS-CoV-2

Infection leads to corona virus disease of 2019 (COVID-19) and manifests with severe acute respiratory disease
syndrome that result into lung consolidation. Which of the following refers to the aspect of disease pertaining to
morphologic change?

a. lung consolidation 
b. COVID-19
c. severe respiratory disease syndrome
d. SARS-CoV-2

The correct answer is: lung consolidation

https://moodle.feu-nrmf.ph/mod/quiz/review.php?attempt=1072772&cmid=86983&showall=1 27/42
1/4/22, 11:29 AM PRELIMINARY EXAMINATION: Attempt review

Question 75

Correct

Mark 1.00 out of 1.00

PRELIMINARY EXAMINATION - HEMODYNAMIC DISORDERS, THROMBOEMBOLIC DISEASES AND SHOCK

A cardiologist underwent a mastectomy with axillary lymph node dissection. Postoperatively, marked swelling of
the left arm was noted. She is afebrile. Her left arm is not tender or erythematous, and it is not painful with
movement or to touch, but it is enlarged with a doughy consistency. Which of the following is the most likely
mechanism for these findings?

a. systemic increased in hydrostatic pressure


b. lymphatic obstruction 
c. sodium and water retention
d. decreased plasma oncotic pressure

The correct answer is: lymphatic obstruction

Question 76
Incorrect

Mark 0.00 out of 1.00

PRELIMINARY EXAMINATION - INFLAMMATION AND REPAIR

Which of the following is lesser in second intention healing compared to first intention healing?

a. Amount of granulation tissue


b. None of the other choices 
c. Extent of scar
d. Epidermal thickness

The correct answer is: Epidermal thickness

Question 77
Correct

Mark 1.00 out of 1.00

PRELIMINARY EXAMINATION – CELL AS AUNIT OF HEALTH AND DISEASE (MDV)

This serves as a cofactor in the clotting of blood in platelets

a. Phosphatidyllysine
b. Phosphatidylcholine
c. Phosphatidylinositol
d. Phosphatidylserine 

The correct answer is: Phosphatidylserine

https://moodle.feu-nrmf.ph/mod/quiz/review.php?attempt=1072772&cmid=86983&showall=1 28/42
1/4/22, 11:29 AM PRELIMINARY EXAMINATION: Attempt review

Question 78

Incorrect

Mark 0.00 out of 1.00

PRELIMINARY EXAMINATION - INFLAMMATION AND REPAIR

A classically activated macrophages will participate in the following processes, except:

a. Tissue repair
b. Cytokine secretion
c. Granuloma formation 
d. Kill engulfed organisms

The correct answer is: Tissue repair

Question 79
Incorrect

Mark 0.00 out of 1.00

PRELIMINARY EXAMINATION - HEMODYNAMIC DISORDERS, THROMBOEMBOLIC DISEASES AND SHOCK

A 20-year-old woman has had altered consciousness and slurred speech for the past 24 hours. CT scan shows a
right temporal hemorrhagic infarction with distal right middle cerebral arterial occlusion. She has had an episode
of pulmonary embolism and pregnancy miscarriage. Laboratory studies show a false-positive serologic test for
syphilis, normal prothrombin time (PT), elevated partial thromboplastin time (PTT), and normal platelet count.
Which of the following is the most likely cause of these findings?

a. Disseminated intravascular coagulation 


b. Hypercholesterolemia
c. Factor V mutation
d. Antiphospholipid antibody

The correct answer is: Antiphospholipid antibody

Question 80
Incorrect

Mark 0.00 out of 1.00

PRELIMINARY EXAMINATION - CELL INJURY, CELL DEATH AND ADAPTATION

A metaplastic change but not brought about by cellular adaptation.

a. squamous to columnar metaplasia 


b. columnar to squamous metaplasia
c. myositis ossificans

The correct answer is: myositis ossificans

https://moodle.feu-nrmf.ph/mod/quiz/review.php?attempt=1072772&cmid=86983&showall=1 29/42
1/4/22, 11:29 AM PRELIMINARY EXAMINATION: Attempt review

Question 81

Correct

Mark 1.00 out of 1.00

PRELIMINARY EXAMINATION - CELL INJURY, CELL DEATH AND ADAPTATION

Generally, this is seen in reversible cell injury.

a. apoptosis
b. autophagy
c. cloudy swelling 
d. necrosis

The correct answer is: cloudy swelling

Question 82
Correct

Mark 1.00 out of 1.00

PRELIMINARY EXAMINATION - INFLAMMATION AND REPAIR

Select the statement that is false about inflammation.

a. Inflammation is a protective response of our body


b. Without inflammation wounds would never heal
c. Inflammation serves to get rid of the causes of cell or tissue injury and consequences of the injury
d. Inflammation is a harmful reaction and qualifies as a disease 

The correct answer is: Inflammation is a harmful reaction and qualifies as a disease

Question 83
Incorrect

Mark 0.00 out of 1.00

PRELIMINARY EXAMINATION - CELL INJURY, CELL DEATH AND ADAPTATION

Papillomaviruses result into which of the following changes?

a. hypertrophy of involved muscle


b. epithelial metaplastic change 
c. atrophy
d. epithelial hyperplasia

The correct answer is: epithelial hyperplasia

https://moodle.feu-nrmf.ph/mod/quiz/review.php?attempt=1072772&cmid=86983&showall=1 30/42
1/4/22, 11:49 AM PLATING ON INFLAMMATION AND REPAIR: Attempt review

Question 3

Correct

Mark 1.00 out of 1.00

PLATING ON INFLAMMATION AND REPAIR

A 60 year old male was admitted due to pneumonia complicated by abscess formation. He also developed
pleural effusion which resulted to fibrin deposition on the pleural surface and inflammation. Which pattern of
inflammation is not present in the case?

a. Fibrinous
b. Granulomatous 
c. Purulent
d. Serous

The correct answer is: Granulomatous

Question 4
Correct

Mark 1.00 out of 1.00

PLATING ON INFLAMMATION AND REPAIR

How do leukocytes cause injury to normal tissue?

a. When inflammatory response is directed against a normal tissue


b. When the inflammatory response is against a harmless exogenous substance in the body
c. All of the given options 
d. Prolonged inflammatory response causing collateral damage to tissue

The correct answer is: All of the given options

Question 5
Correct

Mark 1.00 out of 1.00

PLATING ON INFLAMMATION AND REPAIR

The following are the roles of T helper lymphocytes in inflammatory processes, except:

a. Production of immunoglobulins in chronic inflammation 


b. Secretion of cytokines to promote inflammation
c. IFN gamma secretion by Th1 to activate M1 macrophages
d. Cytokine secretion by Th2 for recruitment and activation of eosinophils

The correct answer is: Production of immunoglobulins in chronic inflammation

https://moodle.feu-nrmf.ph/mod/quiz/review.php?attempt=1015739&cmid=84283 2/9
1/4/22, 11:49 AM PLATING ON INFLAMMATION AND REPAIR: Attempt review

Question 6

Incorrect

Mark 0.00 out of 1.00

PLATING ON INFLAMMATION AND REPAIR

A child with recurrent bacterial infections was diagnosed to have Leukocyte Adhesion Deficiency Type 2 due to
absence of sialyl-Lewis X, what process in migration of leukocytes is defective?

a. Margination
b. Rolling
c. Firm adhesion 
d. Transmigration

The correct answer is: Rolling

Question 7
Correct

Mark 1.00 out of 1.00

PLATING ON INFLAMMATION AND REPAIR 

The skin of the right middle finger of a cook was accidentally cut an hour ago with the knife that she is using to
slice vegetables. The knife has bacterial organisms that were introduced into the skin wound. The skin wound is no
longer bleeding and appears to be red and slightly swollen. Which of the following statements is not true about
the cook’s present wound?  

a. Its local signs, such as redness, are prominent


b. Fibrosis associated with the damage is not yet present
c. The inflammation is expected to be confined to the site of damage and infection
d. The cellular infiltrates are predominantly lymphocytes 

The correct answer is: The cellular infiltrates are predominantly lymphocytes

Question 8
Correct

Mark 1.00 out of 1.00

PLATING ON INFLAMMATION AND REPAIR

Repair by regeneration is observed in the following:

a. neural tissue
b. skin 
c. myocardium
d. Any of the other choices

The correct answer is: skin

https://moodle.feu-nrmf.ph/mod/quiz/review.php?attempt=1015739&cmid=84283 3/9
1/4/22, 11:29 AM PRELIMINARY EXAMINATION: Attempt review

Question 90

Correct

Mark 1.00 out of 1.00

PRACTICAL EXAMINATION QUESTION

If this lung belongs to an individual with liver cirrhosis, the cause of this lesion is: 

a. increased vascular permeability due to inflammation


b. decreased plasma oncotic pressure 
c. lymphatic obstruction
d. increased hydrostatic pressure

The correct answer is: decreased plasma oncotic pressure

Question 91
Correct

Mark 1.00 out of 1.00

PRELIMINARY EXAMINATION - INFLAMMATION AND REPAIR

A tissue that is chronically inflamed has which of the following histomorphologic changes?

a. All of the given choices 


b. Mononuclear cell infiltrates
c. Areas with granulation tissue formation
d. Tissue damage

The correct answer is: All of the given choices

https://moodle.feu-nrmf.ph/mod/quiz/review.php?attempt=1072772&cmid=86983&showall=1 33/42
1/4/22, 11:49 AM PLATING ON INFLAMMATION AND REPAIR: Attempt review

Question 15

Correct

Mark 1.00 out of 1.00

PLATING ON INFLAMMATION AND REPAIR

A 45 year old has cough and colds and has palpable cervical lymph nodes associated with the infection. What do
you call this condition?

a. Edema
b. Lymphadenitis 
c. Lymphangitis
d. Lymphoma

The correct answer is: Lymphadenitis

Question 16
Correct

Mark 1.00 out of 1.00

PLATING ON INFLAMMATION AND REPAIR

The following statements are true about fluid accumulation in the thoracic cavity of a patient with pulmonary
tuberculosis:

a. The fluid has high protein and has cell debris


b. The fluid is an ultrafiltrate of plasma due to high hydrostatic pressure
c. It is an exudative fluid 
d. The fluid accumulation is associated with increase in vascular permeability

The correct answers are: It is an exudative fluid, The fluid has high protein and has cell debris, The fluid
accumulation is associated with increase in vascular permeability

Question 17
Incorrect

Mark 0.00 out of 1.00

PLATING ON INFLAMMATION AND REPAIR

A 30 year old male patient underwent excision of a poypoid new growth on his chest. Several weeks after the
procedure, he noticed an enlarging darker brown elevated lesion on the incision site which eventually grew
beyond the incision borders. Which of the following statements is TRUE? The microscopic findings show

a. predominant epithelioid cells with lymphocytic infiltrates


b. numerous vascular channels admixed with fibroblasts 
c. dermal thick connective tissue deposition
d. exuberant neutrophilic infiltration

The correct answer is: dermal thick connective tissue deposition

https://moodle.feu-nrmf.ph/mod/quiz/review.php?attempt=1015739&cmid=84283 6/9
1/4/22, 11:29 AM PRELIMINARY EXAMINATION: Attempt review

Question 95

Incorrect

Mark 0.00 out of 1.00

PRELIMINARY EXAMINATION - CELL INJURY, CELL DEATH AND ADAPTATION

In Diabetes Mellitus Ischemic injury to the foot may occur. If this is bacterially infected the lesion is referred as
______________.

a. foot abscess
b. wet gangrene
c. dry gangrene 

The correct answer is: wet gangrene

Question 96
Correct

Mark 1.00 out of 1.00

PRELIMINARY EXAMINATION - CELL INJURY, CELL DEATH AND ADAPTATION

Caseous necrosis is characteristic of which of the following disease?

a. stroke
b. Tuberculosis 
c. autoimmune disease
d. diabetic foot

The correct answer is: Tuberculosis

Question 97
Correct

Mark 1.00 out of 1.00

PRELIMINARY EXAMINATION - CELL INJURY, CELL DEATH AND ADAPTATION

What is considered a marker of apoptosis?

a. presence of active caspases 


b. cytochrome C
c. formation of apoptotic bodies
d. absence of inflammation

The correct answer is: presence of active caspases

https://moodle.feu-nrmf.ph/mod/quiz/review.php?attempt=1072772&cmid=86983&showall=1 35/42
1/4/22, 11:29 AM PRELIMINARY EXAMINATION: Attempt review

Question 98

Correct

Mark 1.00 out of 1.00

PRELIMINARY EXAMINATION - INFLAMMATION AND REPAIR

This subset of T cell is responsible for the classical activation of macrophages?

a. Th1 
b. Th17
c. Th2
d. CTL

The correct answer is: Th1

Question 99
Correct

Mark 1.00 out of 1.00

PRELIMINARY EXAMINATION - GENETIC DISORDERS (RGB)

A newborn baby is born to a 40 year old, G1P1(1001) mother. The Pediatrician noticed that the baby has a flat facial
features, oblique palpebral fissures and prominent epicanthal folds. Which of the following neuropathologic
complications would you expect to occur?

a. Brain tumor
b. Alzheimer disease 
c. Parkinsonism
d. Meningomyelocele

The correct answer is: Alzheimer disease

Question 100
Correct

Mark 1.00 out of 1.00

PRELIMINARY EXAMINATION - GENETIC DISORDERS (RRA)

A child with Tay-Sachs Disease will have a mutation in which enzyme?

a. Hexosaminidase A 
b. Alpha-1,4-glucosidase
c. Glucocerebrosidase
d. Iduronidase

The correct answer is: Hexosaminidase A

https://moodle.feu-nrmf.ph/mod/quiz/review.php?attempt=1072772&cmid=86983&showall=1 36/42
1/4/22, 11:49 AM PLATING ON INFLAMMATION AND REPAIR: Attempt review

Stay in touch

http://www.feu-nrmf.ph

+632 8983 8338 / +639176272623

https://moodle.feu-nrmf.ph/mod/quiz/review.php?attempt=1015739&cmid=84283 8/9
1/4/22, 11:29 AM PRELIMINARY EXAMINATION: Attempt review

https://moodle.feu-nrmf.ph/mod/quiz/review.php?attempt=1072772&cmid=86983&showall=1 38/42
1/4/22, 11:29 AM PRELIMINARY EXAMINATION: Attempt review

https://moodle.feu-nrmf.ph/mod/quiz/review.php?attempt=1072772&cmid=86983&showall=1 39/42
1/4/22, 11:29 AM PRELIMINARY EXAMINATION: Attempt review

https://moodle.feu-nrmf.ph/mod/quiz/review.php?attempt=1072772&cmid=86983&showall=1 40/42
1/4/22, 11:29 AM PRELIMINARY EXAMINATION: Attempt review

https://moodle.feu-nrmf.ph/mod/quiz/review.php?attempt=1072772&cmid=86983&showall=1 41/42
1/4/22, 11:29 AM PRELIMINARY EXAMINATION: Attempt review

Previous activity
◄ PRE-FINAL PRACTICAL EXAMINATION

Jump to...

Next activity
MIDTERM EXAMINATION ►

Stay in touch

http://www.feu-nrmf.ph

+632 8983 8338 / +639176272623

info@feu-nrmf.ph

 Data retention summary

 Get the mobile app

https://moodle.feu-nrmf.ph/mod/quiz/review.php?attempt=1072772&cmid=86983&showall=1 42/42

You might also like